Введение
Раздел первый. Подготовка математической олимпиады в школе
Раздел второй. Проведение математической олимпиады; проверка, оценка заданий; выявление победителей
Раздел третий. Основные направления по подготовке учащихся к математическим олимпиадам
5 класс
6 класс
II. Внеклассная работа по математике
III. Внешкольная работа по математике
IV. Заочная работа
Раздел четвертый. Примерные тексты школьных олимпиад
6 класс
7 класс
8 класс
9 класс
10 класс
11 класс
Раздел пятый. Особенности проведения математических олимпиад в сельских школах с малой наполняемостью классов
Примерный текст школьной олимпиады для учащихся 7-9 классов
Раздел шестой. Задачи для подготовки к математической олимпиаде
6 класс
7 класс
8 класс
9 класс
10 класс
11 класс
Раздел седьмой. Ответы, указания, решения
Литература
Содержание
Текст
                    О CV'V
ъ
о*|ч
"
ы
А. В. Фарков
МАТЕМАТИЧЕСКИЕ
ОЛИМПИАДЫ
В ШКОЛЕ
классы


А. В. Фарков МАТЕМАТИЧЕСКИЕ ОЛИМПИАДЫ В ШКОЛЕ 5-11 классы 8-е издание, исправленное и дополненное Москва Айрис-пресс 2009
УДК [372.016:51](072) ББК 74.200.58 Ф24 Рецензент: д-р пед. наук, проф., зав. каф. теории и методики обучения математики и физики Арзамас, гос. пед. ин-та им. А. П. Гайдара Л/. И. Зайкин Все права защищены. Никакая часть данной книги не может переиздаваться или распространяться в любой форме и любыми средствами, электронными или механическими, включая фотокопирование, звукозапись, любые запоминающие устройства и системы поиска информации, без письменного разрешения правообладателя. Серийное оформление О. Е. Бауриной Фарков, А. В. Ф24 Математические олимпиады в школе. 5-11 классы/А. В. Фарков. — 8-е изд., испр. и доп. — М.: Айрис-пресс, 2009. — 256 с: ил. — (Школьные олимпиады). ISBN 978-5-8112-3503-2 В пособии приведены примерные тексты школьных математических олимпиад для учащихся 5—11 классов с подробными решениями или указаниями для решения. Книга будет полезна учителям математики, поскольку содержит рекомендации по составлению текстов школьных математических олимпиад и их проведению, в ней рассмотрены различные подходы к проверке и оценке олимпиадных заданий. ББК 74.200.58 УДК[372.016:51](072) © ООО «Издательство ISBN978-5-8112-3503-2 «АЙРИС-пресс», 2002
Предисловие к восьмому изданию (От автора) Со времени выхода первого издания книги «Математические олимпиады в школе. 5-11 классы» в 2002 году прошло более 6 лет. За период подготовки первого издания и прошедшие 6 лет в олим- пиадном движении в России произошли изменения. Автором была продолжена работа в направлении подготовки учащихся к участию в олимпиадах по математике, был подготовлен к печати ряд новых книг. Поэтому появилась потребность в переработке некоторых разделов данной книги, в частности введения, рекомендаций по подготовке учащихся к олимпиадам (данный раздел существенно расширен). В данное издание включен новый раздел, посвященный особенностям проведения математических олимпиад в сельской школе, дополнены и частично переработаны разделы, касающиеся примерных текстов школьных олимпиад (предложены тексты и для учащихся физико-математических профилей, лицейских классов, которые являются более сложными) и задач для подготовки к олимпиадам (особенно для учащихся 9-11 классов). Были учтены пожелания, высказанные учителями математики и коллегами по работе во время выступлений автора на Педагогическом марафоне в г. Москве, международных, всероссийских и региональных конференциях, касающихся проблем олимпиадного движения в России. А. В. Фарков
Введение В последние годы в России проводится много различных математических олимпиад. Кроме традиционных олимпиад, проводятся также дистанционные, устные, заочные, нестандартные и другие виды олимпиад. С каждым годом все большей популярностью среди учащихся и учителей пользуется международная олимпиада- конкурс «Кенгуру». К сожалению, не стали проводиться соросов- ские олимпиады, в которых принимало участие много школьников. В связи с введением ЕГЭ практически прекратили свое существование во многих регионах олимпиады для абитуриентов, проводящиеся в вузах. Традиционные олимпиады проходят, как правило, в пять туров: школьный, районный (городской), областной (краевой, республиканский), зональный и всероссийский. Данный вид олимпиад имеет богатую историю и сегодня остается самым массовым и популярным как среди учащихся, так и среди учителей. Первые олимпиады по математике начали проводиться в Венгрии с 1896 г., назывались они Этвешское соревнование. Сборник задач этих олимпиад был издан на русском языке в 1896 г. С 1894 г. в России выходил журнал «Вестник опытной физики и элементарной математики», где читателям предлагались математические олимпиады на конкурс. Можно сказать, что это были первые заочные олимпиады. Первые математические олимпиады в СССР состоялась в Тбилиси — 3 ноября 1933 г. и в 1934 г. в Ленинграде в России, а с 1935 г. стали проводиться в Москве. В то время основная цель их была в выявлении способных в математическом отношении школьников для организации их дальнейшего обучения, и насколько это возможно, более раннего их привлечения к научной работе. На самых первых олимпиадах даже действовало правило, в соответствии с которым победители олимпиад не имели права принимать участия в последующих олимпиадах. Видимо, родоначальники олимпиадного движения уже тогда понимали, как опасна «профессионализация» соревновательной деятельности. Сегодня по результатам, показанным учащимися на различных этапах Всероссийской олимпиады, зачастую оценивают работу учителей, учебные заведения и органы управления учреждениями образования. И хотя популярность традиционных олимпиад и сегодня высока, но в большинстве регионов все меньше стало проводиться по сравнению с восьмидесятыми годами олимпиад для учащихся 5-8 клас-
сов, хотя учащиеся именно этого возраста наиболее любознательные, желают участвовать в различных соревнованиях. В данном пособии автор на основе личного опыта и изучения опыта, имеющегося в России, приводит требования к подбору заданий, включаемых в тексты школьных олимпиад; описывает методику подготовки учащихся к участию в олимпиадах; методику проведения и оценки заданий олимпиады. Большую часть пособия занимают примерные тексты школьных олимпиад по математике для учащихся разного возраста. Также в пособии приведена большая подборка задач, которые можно использовать как для составления текстов школьных и районных олимпиад, так и с целью подготовки учащихся к олимпиадам. Пособие адресовано в первую очередь учителям математики общеобразовательных учреждений и руководителям математических кружков внешкольных образовательных учреждений. Но оно будет полезно как руководителям общеобразовательных учреждений, так и студентам математических факультетов педвузов. Как показывает опыт, многие учащиеся используют данное пособие для самостоятельной подготовки к школьным и районным математическим олимпиадам различного уровня и добиваются определенных успехов. Поэтому данное пособие может быть рекомендовано и для учащихся. Практически ни одна книга, а особенно посвященная олимпиадам, не может быть идеальной. Автор будет благодарен за все критические замечания, пожелания, которые он постарается учесть в следующих переизданиях. Все замечания по улучшению данного пособия можно высылать на адрес издательства.
Раздел первый ПОДГОТОВКА МАТЕМАТИЧЕСКОЙ ОЛИМПИАДЫ В ШКОЛЕ Математические олимпиады в школе, как правило, проводятся отдельно для каждой параллели классов, начиная с пятого класса. Основными целями школьной олимпиады являются: • расширение кругозора учащихся; • развитие интереса учащихся к изучению математики; • выявление учащихся, проявивших себя по математике, для участия их в районных (городских) олимпиадах и для организации индивидуальной работы с ними. Для проведения олимпиады в школе создается оргкомитет. Как правило, в него входят: заместитель директора — председатель оргкомитета, председатель школьного методического объединения учителей математики — заместитель председателя оргкомитета, а также члены оргкомитета (учителя математики и представители старшеклассников). Для составления, проверки и оценки работ участников олимпиады создается жюри, в состав которого входят председатель и члены жюри. Председателем жюри чаще всего является руководитель школьного методического объединения учителей математики (заведующий кафедрой). Членами жюри могут быть учителя математики и преподаватели вузов, работающие в данной школе; старшеклассники (для проведения олимпиад в младших классах) и студенты педвузов, проходящие практику в школе. Состав оргкомитета, жюри, порядок проведения олимпиад в школе утверждается директором школы. Время проведения школьных олимпиад определяется в соответствии с «Положением о проведении Всероссийской олимпиады в данном учебном году»; как правило, для 8-11 классов это декабрь (ноябрь), а для 5-7 классов — январь-февраль. Возможно и одновременное проведение олимпиады для всех
классов, если в январе (декабре) проводится II тур для 5- 11 классов. Председатель оргкомитета собирает оргкомитет и распределяет обязанности для всех членов: • подготовка текстов олимпиады; • разработка положения о проведении олимпиады, поощрении победителей; • подготовка материалов (бумаги и т. д.); • подготовка объявления и т. д. Наиболее ответственным моментом подготовки олимпиады является составление текста олимпиады. Рассмотрим основные требования к тексту школьной олимпиады по математике: 1. Число задач в тексте олимпиадной работы должно быть от 4 до 7 (при 1-3 заданиях могут возникнуть проблемы с определением победителей и призеров олимпиады, настроиться на решение больше 7 заданий учащимся сложно). 2. Все задачи в тексте работы должны располагаться в порядке возрастания трудности (или сложности). Хотя данные понятия довольно часто встречаются в методической литературе в последние годы, все же остановимся на них подробнее. Сложность — это объективная характеристика задачи, определяемая ее структурой. Сложность задачи зависит от: • объема информации (числа понятий, суждений и т. п.), необходимого для ее решения; • числа данных в задаче; • числа связей между ними; • количества возможных выводов из условия задачи; • количества непосредственных выводов, необходимых для решения задачи; • количества взаимопроникновений при решении задачи; • длины рассуждений при решении задачи; • общего числа шагов решения, привлеченных аргументов и т. д. В. И. Крупич предложил формулу для нахождения сложности задачи: S = т + п + I,
где S — сложность задачи, т — число элементов задачи, п — число явных связей между элементами задачи, / — число видов связи. Рассчитать сложность задачи не очень просто, чаще всего учителя интуитивно распределяют задачи по сложности. Но в тексте олимпиадной работы задания берутся из разных разделов, некоторые из них нестандартные. Поэтому лучше все же применять понятие трудности задания. Трудность — субъективная характеристика задачи, определяемая взаимоотношениями между задачей и решающим ее учеником. Трудность задачи зависит от: • сложности задачи (сложная задача, как правило, является более трудной для учащихся); • времени, прошедшего после изучения материала, который встречается в тексте задачи (задачи на материал, изученный 1-2 года назад, используемые факты, которые уже забылись, более трудны для учащихся); • практики в решении подобного рода задач; • уровня развития ученика (задача, трудная для среднего ученика общеобразовательного класса, может быть легкой для обычного ученика физико-математического класса); • возраста учащегося (задача, трудная для пятиклассника, может быть легкой для восьмиклассника) и т. д. Трудность определяется процентом учеников, решивших задачу из числа ее решавших. Существуют различные формулы для расчета трудности задачи. Рассмотрим, на наш взгляд, наиболее простую из них: КТ = ^ • 100%, где Кт — коэффициент трудности, измеряемый в процентах, п — число учащихся, не решивших задачу, р — число учащихся, решавших задачу, в том числе и не приступивших к ней (общее число участников олимпиады). Пример: 8
Номер задачи п Р 1 2 20 10% 2 6 20 30% 3 10 20 50% 4 12 20 60% 5 16 20 80% 6 19 20 95% Таким образом, из данной таблицы следует, что 6-я задача — наиболее трудная, так как ее решил всего 1 ученик, а 1-я — наиболее легкая, ее решило 18 учеников. 3. В числе первых задач должны быть 1-2 задачи, доступные большинству учащихся, т.е. их трудность должна быть примерно 10-30%. Это могут быть обычные задачи продви нутого уровня, аналогичные задачам из контрольных работ, а также и не изучаемые в школе, но которые должно решить большинство участников. Это необходимо, так как в школьной олимпиаде участвуют все желающие. А участник, не решивший ни одной задачи, теряет уверенность в своих силах, а иногда и интерес к математике. Поэтому должны быть 1-2 доступные почти всем задачи. Но и эти задачи могут содержать «изюминку», благодаря которой более сильный ученик решит ее быстрее и рациональнее. 4. В середине текста олимпиады должно быть 2-3 задачи повышенной трудности. Это могут быть задачи продвинутого уровня из контрольных работ, но с измененными условиями. Их должна решить примерно половина участников, т. е. трудность их будет примерно 40-60%. (Ученик, решивший более третьей части всех задач, уже может получить поощрение.) 5. Последними в тексте олимпиады должно быть 1-2 задания более трудных, их должны решить единицы, значит, и трудность их будет уже примерно 80-95% . Это задания уровня районных (городских) олимпиад. 6. Включаемые задания должны быть из разных разделов школьного курса математики, но, как правило, на материал, изученный в данном учебном году и во втором полугодии предыдущего года. 7. В числе заданий текста олимпиады могут быть занимательные задачи, задачи-шутки, софизмы, задачи прикладного характера.
8. Для заинтересованности учащихся в посещении кружков, факультативов желательно включать задания, аналогичные рассмотренным там. Это могут быть логические задачи, задачи на применение принципа Дирихле, инвариантов, графов, задачи на раскраски, уравнения в целых числах и т. п. Такого рода задачи часто называют специальным термином «олимпиадные», хотя, конечно, не только они должны быть в тексте школьной олимпиады. 9. В качестве одной из задач может быть задача, в условии которой фигурирует год проведения олимпиады. 10. В числе задач не должно быть задач с длительными выкладками, задач на использование трудно запоминающихся формул, на использование справочных таблиц. 11. В текстах олимпиад для разных классов могут быть одинаковые задания. Таковы основные требования к составлению текста работы школьной олимпиады. Кто будет составлять тексты олимпиад — дело оргкомитета. Можно привлечь специалистов в области диагностики из вузов, можно поручить наиболее опытному из учителей. Но, вероятно, лучше, если набираться опыта в составлении текстов будут все учителя. Тем более, что после проведения олимпиады уже можно оценить качество подготовленных материалов. Также трудность некоторых заданий можно оценить, дав аналогичные задания в классе старше. Таким образом, составление текстов по каждой параллели можно поручить 1-2 учителям. Они организуют подбор заданий, причем первоначально заданий необходимо подготовить больше. Окончательные тексты школьных олимпиад желательно утвердить на заседании школьного методического объединения учителей математики, обговорив там число предлагаемых заданий, вариант оценки заданий (возможные варианты оценки будут рассматриваться ниже), распределение членов жюри по классам. Особенно это важно для тех школ, где учащиеся в различных классах обучаются по различным учебникам.
Раздел второй ПРОВЕДЕНИЕ МАТЕМАТИЧЕСКОЙ ОЛИМПИАДЫ; ПРОВЕРКА, ОЦЕНКА ЗАДАНИЙ; ВЫЯВЛЕНИЕ ПОБЕДИТЕЛЕЙ Школьные олимпиады проводятся, как правило, вне уроков. Возможно проведение олимпиад на кружке или факультативе, но для более объективной картины лучше бы проводить олимпиады с утра или после 3-4-го уроков, перенося остальные уроки на другие дни. Есть интересный опыт проведения школьных олимпиад после 4 уроков в последний день первой четверти, когда участники школьной олимпиады решают задачи, а остальные учащиеся школы занимаются генеральной уборкой школы. Проведение школьной олимпиады в выходные дни нецелесообразно. В школьных олимпиадах имеют право принимать участие все желающие. В случае большого числа параллельных классов и, соответственно, огромного числа желающих, возможно проведение сначала классной, а затем школьной олимпиады. Тогда на школьную олимпиаду приглашаются только призеры классных олимпиад или учащиеся, набравшие определенное число баллов (если текст олимпиад ной работы был единый). Но участники классной олимпиады также считаются участниками школьной олимпиады. Также школьные олимпиады можно проводить в 2 тура: заочный и очный. Лучших участников заочного тура обязательно приглашают на очный тур, остальные же учащиеся приходят по желанию. Продолжительность школьной олимпиады рекомендуется: в 5-6 кл. — 1-1,5 ч; в7-8кл. — 1,5-2 ч; в 9-11 кл. — 2-3 ч. В указанное время все участники олимпиады приходят в специально отведенные классы, рассаживаются по местам. Желательно каждому участнику предоставить отдель- и
ный стол. На столах заранее разложена бумага для выполнения работ, тексты олимпиады. Один из членов жюри знакомит участников с текстом олимпиады, числом баллов за каждое задание, временем выполнения работы, правилами оформления заданий. Задания могут быть выполнены в любом порядке. Черновик должен быть подписан и сдан. Особенно это важно для учащихся 5 класса, которые, вполне возможно, впервые участвуют в таких соревнованиях. Также для участников олимпиады можно подготовить и специальные памятки. Памятка участнику олимпиады. 1. Прочитайте все задачи и наметьте, в каком порядке вы будете их решать. Помните, последние задачи обычно более сложные. 2. Если для вас задача решилась слишком легко, то, скорее всего, вы не поняли условие или где-то ошиблись. 3. Если задача не решается — попробуйте упростить ее условие (взять меньшие числа, рассмотреть частные случаи и т. д.) или порешать ее «с конца», «от противного», поставить вместо чисел переменные и т. д. 4. Не зацикливайтесь на одной задаче: иногда отрывайтесь от нее и оценивайте положение. Если есть хоть небольшие успехи, то можно продолжать, а если мысль ходит по кругу, то задачу лучше оставить (хотя бы на время). 5. Почувствовав усталость — сразу отдыхайте (посмотрите в окно, закройте глаза, отвлекитесь). 6. Решив задачу, сразу оформите ее решение. Это поможет проверить рассуждения и освободить мысли для других задач. 7. Перед сдачей работы, проверьте еще раз написанное — поймут ли ваши решения задач члены жюри? После этого участники олимпиады приступают к решению заданий. Консультироваться с товарищами, поворачиваться, использовать какую-то литературу на олимпиаде запрещается. Исключением могут быть лишь справочные материалы, но так как единых у всех учеников их может не быть, лучше задания с использованием справочной литературы не включать в текст. 12
За несколько минут до окончания работы член жюри предупреждает участников об окончании времени выполнения работы и учащиеся начинают сдавать работы, подписав их. После необходимого перерыва (5-10 минут) ученики возвращаются в класс, где один из членов жюри проводит разбор заданий олимпиады. Нецелесообразно отодвигать время разбора на занятие кружка или факультатива. Тем не менее, познакомить членов кружка или факультатива как с предложен ными заданиями на олимпиаде, так и с ее итогами необходимо. После разбора заданий члены жюри по каждому классу приступают к проверке заданий и оценке решений. Желательно, чтобы в каждой параллели было не менее 3 членов. Возможны два варианта проверки: 1) каждый член жюри проверяет только 1-2 задания из текста олимпиады и карандашом оценивает каждое задание, выставляя рядом с заданием определенное число баллов; 2) каждый член жюри проверяет несколько работ участников, оценивая все задания. Оба варианта проверки имеют как плюсы, так и минусы. Поэтому после проверки всех работ надо снова всем членам жюри еще раз обсудить число баллов, выставленное за каждое задание. Работы участников, набравших наибольшее число баллов, рекомендуется проверить еще раз с председателем жюри школьной олимпиады по математике. Самым сложным и ответственным моментом в проведении математической олимпиады является оценка заданий. В зависимости от того, сколько баллов было выставлено за задания, возможны следующие подходы к оцениванию заданий. 1. Подход, применяемый в последние годы чаще всего для городских (районных) олимпиад, при котором все задания оцениваются исходя из 7 баллов. 7 баллов ставится за верное решение; 6 баллов — за верное решение с недочетами; 4-5 баллов — решение в основных чертах верно, но неполно или содержит непринципиальные ошибки; 1-3 балла — решение в целом неверно, но содержит более или менее существенное продвижение в верном направлении; 0 баллов — решение неверно или отсутствует. 13
Решение считается неполным, если оно: • содержит основные идеи, но не доведено до конца; • при верной общей схеме рассуждений содержит пробелы, то есть явно или скрыто опирается на недоказанные утверждения, которые нельзя счесть известными или очевидными. 2. Применяется для заданий, которые все оценены разным числом баллов в зависимости от сложности (трудности) задания. Рассмотрим данный подход подробнее. Пусть некоторое задание оценено 5 баллами. Тогда при безупречно верном решении участнику ставят 5 баллов; при верном решении с недочетами — 4 балла; при неполном решении с негрубыми ошибками — 3 балла; при неверном решении, но с продвижением в верном направлении — 1-2 балла; за отсутствие решения или неверное решение — 0 баллов. Тогда задания, оцененные 3,7, 10 баллами, при аналогичных решениях будут оцениваться с помощью следующей таблицы: Число баллов Безупречное решение Решение с недочетами Неполное решение с негрубыми ошибками Неверное решение, но есть продвижение в верном направлении 5 5 4 3 1-2 3 3 2,5 2 1 7 7 6 4-5 1-3 10 10 9 6-8 1-5 Главное отличие второго подхода состоит в том, что более трудные задания оцениваются большим числом баллов. 3. Подход аналогичный второму, но каждое задание член жюри оценивает значками +, ±, =f, —, 0, которые означают: +: верное решение; ±: верное решение с недочетом; =F: найдена идея решения, но решение не доведено до конца или выполнена лишь часть задания; -: решение неверное, но ученик искал его, хотя и не нашел; 0: отсутствие решения, ученик не приступил к решению. Тогда число баллов за каждое задание выставляется в соответствии с этими значками: 14
Максимальное число баллов за задание + ± - 0 3 3 2-2,5 1,5-2 1 0 4 4 3 2 1 0 5 5 4 2-3 1 0 6 6 5 2-4 1 0 7 7 6 3-5 1-2 0 8 8 7 3-6 1-2 0 9 9 8 3-7 1-2 0 10 10 9 3-8 1-2 0 11 11 9-10 3-8 1-2 0 12 12 10-11 3-9 1-2 0 Главным недостатком третьего подхода к оцениванию может быть большое расхождение у членов жюри при выставлении знака « =f ». Возможны, конечно, и другие подходы. После того, как жюри перепроверило работы всех участников, и особенно набравших наибольшее число баллов, определяются победители и призеры. Они должны быть независимо от того, сколько баллов набрали участники. Если такого не получилось, значит, текст олимпиадной работы составлен с нарушением требований. И виноват составитель текста, а не ученики. Абсурд, если I место не присуждается ни одному ученику школьной олимпиады. Отличием в подведении итогов и определении победителей и призеров от спортивных является то, что победителей и призеров в каждой параллели может быть несколько. Итак, кто же является победителем школьной олимпиады? Ясно, что ученик, набравший наибольшее число баллов. Но так как субъективизм членов жюри может проявиться все равно при оценке заданий, то можно установить специальные границы в процентах от максимального числа баллов. Наиболее подходящим является следующий подход: I место присуждается всем участникам, набравшим больше 75% от максимального числа баллов за все задания олимпиады. (Если все же при неудачном тексте олимпиады никто не набрал данного числа баллов, необходимо опустить число баллов до 70%, 65% и т. д. Олимпиада — это соревнование, а в любом соревновании бывают победители, они должны быть и здесь.) 15
II место присуждается участникам, набравшим от 50 до 75% от максимального числа баллов. III место присуждается набравшим от 33 до 50%. Данные границы участникам олимпиады можно не сообщать. Примерные границы от максимального числа баллов указаны в таблице. Максимальное число баллов I место II место III место 20 15-20 10-14 7-9 25 19-25 13-18 9-12 30 22-30 15-21 10-14 35 26-35 18-25 11-17 40 30-40 20-29 13-19 45 33-45 23 32 15-22 50 37-50 25-36 16-24 В школьном туре победителей и призеров может быть 20- 25% от числа участников, но не будет необычным, если в некоторой параллели больше половины участников станут призерами. Это только повысит интерес учащихся к участию в олимпиадах. На следующий год желающих, думаю, будет больше. После определения победителей заполняется протокол, члены жюри подписывают его. Как правило, в школе апелляции по олимпиадам не рассматриваются. Но учащиеся имеют право ознакомиться с проверенными решениями. После определения победителей и призеров олимпиады по каждой параллели руководство школы совместно с оргкомитетом и жюри олимпиады проводит награждение. Согласно «Положению о Всероссийской олимпиаде школьников» победители всех этапов награждаются грамотами, дипломами и призами. Провести награждение победителей и призеров олимпиады можно на математическом вечере или торжественной линейке. В качестве призов могут быть книги по математике, художественные, научно-популярные книги, денежные призы. Все зависит от конкретных условий школы. Для награждения победителей и призеров школьных олимпиад можно привлечь и средства спонсоров. Иногда на школьных олимпиадах побеждают не те ученики, кто получает на уроках отметки «хорошо» и «отлично», а те, кто получает и «удовлетворительно» по математике, особенно это бывает в 5-8 классах. Поэтому учителю необходима 16
психологическая работа как с учащимися, которые стали победителями, так и с теми школьниками, кто в этот раз не попал в призеры. Некоторые регионы проводят школьные олимпиады по единым текстам, что вряд ли целесообразно применять везде, т. к. отдельные школы региона резко отличаются по уровню развития учащихся и тексты в одной школе могут решить практически все учащиеся параллели, а в другой больше 1-2 задач не решит никто. Также сегодня в связи с большим числом учебников по математике, может оказаться, что некоторые темы изучены по некоторым учебникам, а по другим — нет. Если уж и давать единые тексты, то в качестве заочного тура с целью лучшей подготовки учащихся к участию в олимпиадах.
Раздел третий ОСНОВНЫЕ НАПРАВЛЕНИЯ ПО ПОДГОТОВКЕ УЧАЩИХСЯ К МАТЕМАТИЧЕСКИМ ОЛИМПИАДАМ Наряду с целями школьных олимпиад, указанных в первом разделе, олимпиады по математике готовят учащихся к жизни в современных условиях, в условиях конкуренции. Умение решать задачи, особенно олимпиадные, всегда являлось одним из показателей математической одаренности ученика. Причем главная ценность олимпиад состоит не в выявлении победителей и награждении особо одаренных учащихся, а в общем подъеме математической культуры, интеллектуального уровня учащихся. И для того, чтобы этот подъем культуры и интеллекта действительно произошел, к олимпиадам учащихся надо готовить. Тем более что сегодня по итогам олимпиад оценивают итоги внеклассной и внешкольной работы по математике в школе, районе, области (крае, республике). Школьные, районные, региональные, окружные олимпиады по математике позволяют сравнивать качество математической подготовки, а также и состояние преподавания математики в отдельных классах школы, в отдельных школах района, а также и в различных регионах и округах. Также сегодня, во многом, результаты работы учителя определяются и тем, каких и сколько учащихся — призеров различного рода олимпиад он подготовил. Между тем, природа может распорядиться так, что в данном регионе, в данном месте не окажется таких одаренных детей, и что бы учитель ни предпринимал, все может быть безрезультатно. С другой стороны, учитель математики может не предпринимать никаких особых усилий, а ученик блистает на различных соревнованиях, и прежде всего, на олимпиадах самого высокого уровня. Он добивается этого благодаря своим особым 18
математическим способностям, которые он продолжает развивать, работая с математической литературой самостоятельно, занимаясь на всевозможных математических курсах, в школах при вузах и т. п. Здесь не хотелось бы дискутировать: правильно делает руководство образованием, оценивая только результат, а не то, как достиг этого результата учитель. Для нас важнее то, как учителю математики не только готовить учащихся к олимпиадам, но и сделать все, от него зависящее, для математического развития учащихся. В настоящее время, на основе последней редакции закона «Об образовании», победы учащихся на олимпиадах международного и всероссийского уровней являются достаточным основанием для зачисления в вуз без экзаменов, а выдающиеся результаты, показанные в мероприятиях системы дополнительного образования — для приема в вуз вне конкурса. Также участие в олимпиадах по математике, математических кружках и факультативах планируется учитывать и при отборе учащихся в профильные классы. Школа сегодня уже не является единственным, монопольным источником информации, знаний, умственного развития учащихся. В частности, большой вклад в образование учащихся вносит система дополнительного образования детей. А поэтому результаты, достигаемые учащимися в различных мероприятиях, проводимых в данной системе, должны учитываться при определении перспектив дальнейшего обучения. Так как наибольших успехов в олимпиадах добиваются учащиеся с нестандартным, творческим мышлением, высокими математическими способностями, повышенной обучаемостью математике, то одним из путей подготовки учащихся к олимпиадам является развитие их математических способностей, мышления, интеллекта. Давно известно, что люди, систематически занимающиеся умственным трудом, имеют более высокий показатель интеллекта. А теперь остановимся подробнее на основных направлениях, которые можно выделить в подготовительной работе к олимпиадам любого учителя школы, не только математики. Можно выделить следующие основные направления. 19
I. Работа учителя математики на уроке Глубоко не правы те учителя, которые не уделяют внимания при проведении уроков математики подготовке учащихся к олимпиадам. Чаще победителями олимпиад, начиная с городского (районного) тура, являются учащиеся, которые являются одаренными. Учить же, развивать одаренных детей только вне урока нереально. Всегда можно найти место на уроке, когда вместе с обучающими задачами на уроке можно решать и задачу развития ученика. Например, при изучении темы «Объемы тел» (11 класс), после решения ряда задач по нахождению объема пирамиды, можно предложить учащимся и такую задачу: «Найти объем пирамиды, у которой все боковые ребра образуют между собой углы по 90°, а сами ребра имеют длины соответственно 3, 4, 5 см». Применяя подход, которым решались предыдущие задачи, можно найти стороны основания (по теореме Пифагора), затем площадь основания. Проблема возникнет при нахождении высоты пирамиды. Применив же нестандартный прием: переворачивание пирамиды таким образом, что основанием становится один из прямоугольных треугольников, а высотой — оставшееся третье ребро — мы сразу решим задачу. Подобного рода примеров можно привести не один. Рассмотрим несколько таких задач. Все они тесно связаны с темой урока, тем не менее являются и олимпиад- ными. 1. Решение олимпиад пых задач, тесно связанных с темой урока 1. Вычислите: а) 90 + 89 + 88 + ... + 1 + 0-1-2-...-90-91-92-93; б) 1-2 + 3-4 + 5-6 + ... + 2005-2006. Обе приведенные задачи являются стандартными, но если выполнять действия по порядку, не применяя законов сложения и вычитания, на это потребуется много времени. А время на олимпиадах очень ценно. Поэтому ученик, нашедший более быстрое решение этих и подобных заданий, сэкономит время на решение других задач. На уроке данные задачи мож- 20
но предложить при изучении темы «Сложение и вычитание положительных и отрицательных чисел». 2. При изучении темы «Степень с натуральным показателем» можно предложить для решения учащимся следующие типы задач: а) Сравните: 6523 и 25517. б) На какую цифру оканчивается число 20072010? Решение. а) 6523 > 6423 = (26)23 = 2138. А 25517 < 25617 = (28)17 = = 2136. Так как 6523 > 2138, 2138 > 2136, а 2136 > 25517, то 6523 > 25517. б) Так как последняя цифра числа 20072010 определяется последней цифрой числа 72010, то найдем значения степеней 71, 72, 73, 74, 75 и т.д., и заметим закономерность: последней цифрой являются 7, 9, 3, 1, а далее они повторяются. Так как 2010 = 502 • 4 + 2, то 72010 оканчивается той же цифрой, что и 72, то есть цифрой 9. Тогда и число 20072010 оканчивается на цифру 9. 3. При изучении темы «Алгебраические дроби» можно решить следующую задачу: «Вычислите сумму: 1 j, I j, 1 1 + х + ху 1+y + yz 1+z + zx' если xyz = 1». Решение. Умножим числитель и знаменатель второй дроби на х, а третьей — на ху. Учитывая, что xyz = 1, получим у всех дробей одинаковые знаменатели. Сложим данные три дроби, в итоге получим дробь, у которой числитель и знаменатель равны одному и тому же выражению 1 + х + ху. А, значит, искомая сумма равна 1. 4. При изучении квадратных уравнений, можно наиболее сильным учащимся класса предложить и такую задачу: «Может ли дискриминант квадратного уравнения с целыми коэффициентами равняться 2006? А 2008?» Рассмотрим решение данной задачи. У квадратного уравнения ах2 + Ьх + с = 0, где a,b,c e Z дискриминант D = Ь2—4ас. Так как D = 2006, то найдем целые решения уравнения Ь2-4ас = 2006. Так как правая часть 21
уравнения делится на 2, то и левая часть должна делиться на 2, поэтому Ъ = 2k, тогда 4fe2-4ac = 2006. Разделив обе части уравнения на 2, получим 2k2-2ac = 1003. В левой части уравнения получилось четное число, а в правой — число нечетное. Поэтому уравнение решений в целых числах не имеет. Для числа 2008 имеем Ь2-4ас = 2008, а так как Ъ = 2k, то получим: 4fe2-4ac = 2008. Разделив на 4 обе части уравнения, получим: k2—ac = 502. Данное уравнение имеет решения в целых числах, например: а = 1,с = 27, k = 23. Тогда уравнение х2 + 46* + 27 = 0 имеет D = 2116-4 • 1 • 27 = 2008. Конечно, можно найти и другие решения. На этом же занятии можно сказать, что с методами решения уравнений в целых числах подробно можно познакомиться на занятии факультатива или элективного (если, конечно, факультативы или элективные курсы по математике проводятся в данной школе). 5. При изучении арифметической прогрессии можно рассмотреть задачу: «Докажите, что если в бесконечную арифметическую прогрессию с положительной разностью входят числа 25, 43, 70 (не обязательно стоящие рядом), то в эту прогрессию входит и число 2005». Решение. Так как 25, 43, 70 — члены арифметической прогрессии, то 25 = а\ + kd; 43 = а + nd; 70 = а + md. Из данных трех равенств следует, что 18 = (n-k)d, 27 = (m-n)d. Из данных двух равенств получаем: 9 = (т-2п + k)d. Так как 2005 = 70 + 1935, а 1935 = 215 9 = 215(т-2п + k)d, то 2005 = 70 + 215(т-2п + k)d = ai+md + 215(m-2n + k)d = = ai + (216m-430n + 215k)d или 2005 = ai + Id, где / > 0. 6. При решении текстовых задач в различных классах можно предлагать учащимся решение и задач, которые были на олимпиадах различного уровня, обязательно указывая, сколько учеников их решили. Например: а) Мотоциклист и велосипедист выехали одновременно из пункта А в пункт В. Проехав треть пути, велосипедист остановился и тронулся дальше лишь тогда, когда мотоциклисту 22
оставалось проехать треть пути до В. Мотоциклист, доехав до В, без остановки поехал обратно в А. Кто приедет раньше: мотоциклист в А или велосипедист в В, если велосипедист после первой остановки больше в пути не останавливался? Решение. Так как велосипедист стоял, дожидаясь пока мотоциклисту останется проехать треть пути до В, то на треть всего своего пути велосипедист затратил времени меньше, чем 2 1 мотоциклист на треть своего (-^АВ от 2АВ составляют —). Зна- 3 3 чит и на весь путь велосипедист затратит времени меньше. б) Одну овцу лев съел за 2 дня, волк за 3 дня, собака за 6 дней. За сколько дней они вместе съедят овцу? Решение. 1) Так как лев съел овцу за 2 дня, то за 1 день он съел i овцы. 2) Так как волк съел овцу за 3 дня, то за 1 день он съел - овцы. о 3) Так как собака съела овцу за 6 дней, то за 1 день она съела - овцы, о 4) Вместе лев, волк и собака за 1 день съедят - + - + - = 1, Z 3 о то есть 1 овцу. в) Старинная задача. «Скажи мне, знаменитый Пифагор, сколько учеников посещают твою школу и слушают твои беседы? — Вот сколько, — ответил философ, — половина изучает математику, четверть — музыку, седьмая часть пребывает в молчании и, кроме того, есть три женщины». Решение. Обозначив число учеников Пифагора за х, получим, что jrx — изучает математику, jx — музыку, а ^х — пребывает в молчании. Так как, кроме того, есть еще 3 женщины, то получаем уравнение: Решением данного уравнения будет х = 28. Следовательно, школу Пифагора посещают 28 учеников. 23
Наибольшие трудности у учеников на олимпиадах, как показывает собственный опыт участия в олимпиадах разного уровня, а также проведения школьных и городских олимпиад, вызывают геометрические задачи. Хотя именно геометрия прекрасно развивает нестандартное мышление и выделяет людей, способных заниматься математикой. Данный тип олим- пиадных задач является самым обширным. Это и задачи на разрезания, и на построение, и на нахождение углов. Но чаще всего встречаются задачи, которые используют в своем решении какую-то необычную идею, чаще всего дополнительное построение. Рассмотрим несколько примеров олимпиадных задач по геометрии, которые можно рассмотреть на уроке, связав их решение с темой урока. 7. При изучении геометрических построений можно предложить задачи на построение углов заданной градусной меры через известный угол. Например: «Постройте угол в 5°, если дан угол в 34°». Решение. Отложите угол 34° 5 раз, тогда получится угол 170°. Так как разность развернутого угла и 170° будет равна 10°, то разделим угол в 10° на 2 равных угла и получим угол в 5°. 8. Так как на олимпиадах часто предлагаются задачи, в которых используются дополнительные построения, то подобного рода задачи необходимо рассматривать и на уроках, особо обращая внимание на эти дополнительные построения. Например, рассмотрим такую задачу: «Дан параллелограмм ABCD. К — середина стороны ВС> М — середина стороны CD, АйГ = 6см, AM = Зсм, ZKAM = 60°. Найдите длину стороны AD. Ответ обоснуйте». Решение. Задача имеет множество решений. Рассмотрим наиболее оригинальное (см. рис. 1). Проведем в трапеции AKCD среднюю линию ML. Она будет параллельна AD и КС, причем AL = 3 см. Обозначим AD = 2х> тогда КС = х. Так как треугольник ALM — равнобедренный с углом при вершине 60°, то он — равносторонний, поэтому LM = 3 см. А тогда, ис- 24
пользуя свойство средней линии трапеции, имеем: х = 3, откуда х = 2, а значит, AD = 4 см. Так как решение подобного рода задач требует применения некоторых качеств и приемов мышления, то на уроке необходимо уделять внимание и развитию некоторых качеств ума (прежде всего, гибкости и глубины), так и приемов умственной деятельности (в первую очередь, анализа, так как он чаще всего применяется в олимпиадных задачах, особенно геометрических). 2. Развитие качеств ума и приемов умственной деятельности Для развития гибкости ума на уроке надо: • применять решение упражнений, в которых встречаются взаимно обратные операции; • решать задачи несколькими способами, доказывать теоремы различными методами; • применять различные переформулировки условия задачи; • учить переключению с прямого хода мыслей на обратный; • учить тому, какие знания, умения, навыки и в каком порядке применять в конкретной задаче и т. д. Рассмотрим примеры задач, способствующих развитию данного качества. 25
Упражнения на развитие гибкости ума 1. У двух зрячих один брат слепой, но у слепого нет зрячих братьев. Как это может быть? (Из первой фразы как будто следует, что речь в задаче идет о братьях, тогда как на самом деле зрячими оказываются сестры.) 2. Два ученика подошли одновременно к реке. У берега реки стояла лодка (лишь для одного человека). Тем не менее оба сумели переправиться через речку в одной лодке. Каким образом? (Из первой фразы как будто кажется, что ученики подошли к реке на одном берегу, но для решения задачи получается, что они подошли к реке на разных берегах.) 3. Вам дано 5 спичек. Сложите из них 2 равносторонних треугольника. А если спичек будет 6, то, сколько равносторонних треугольников Вы можете изобразить? (Первая задача решается на плоскости, а вторая — на плоскости (тогда получается 2 равносторонних треугольника) или в пространстве (тогда получается 4 равносторонних треугольника).) 4. Найдите как можно больше способов решения задач: а) Докажите, что треугольник, в котором медиана равна половине стороны, к которой она проведена, является прямоугольным (см. рис. 2). В Решение. Способ № 1. 1) /LA + Z ABD + ZDBC + /LC = 180°. 2) ZA = ZABD, Z.C = ZDBC. 3) Л А + Z ABD + ZDBC + ZC = 2 • Z ABD + 2 • ZDBC = 2(ZABD + ZDBC) = 2 • ZABC = 180°. 26
4) Z ABC = 90°, а значит A ABC — прямоугольный. Вывод: При доказательстве использовались теорема о сумме углов треугольника и свойство углов при основании равнобедренного треугольника. Способ № 2. 1) Рассмотрим треугольник ABD. ZBDC = ZA + ZABD. 2) Рассмотрим треугольник DBC. ZADB = ZC + ZDBC. 3) В треугольнике ABD ZA = ZABD. 4) В треугольнике DBC AC = ZDBC. 5)ZADB + ZBDC = 180°. 6) Z ADB + ZBDC = ZA + ZABD + ZC + ZDBC = = 2 • (Z ABD + ZDBC) = 180°. А значит, Z ABD + ZDBC = ZB = = 90°. Вывод: В данном способе использовались теорема о внешнем угле треугольника, свойство углов при основании равнобедренного треугольника, теорема о смежных углах. Способ № 3. 2) ZA = A ABD, AC = ZDBC. 3) ZFBC = ZABD + ZDBC = Z ABC. 4) ZFBC + ZABC = 180°, ZFBC = ZABC ^ ZABC = 90°. Вывод: Применялись те же суждения, что и во втором способе, но в других комбинациях. б) Высоты треугольника ABC, проведенные из точек А и С, пересекаются в точке М (см. рис. 3). Найдите угол АМС, если ZA = 70°, ZC = 80°. Решение. Способ № 1. Рассмотрим прямоугольные треугольники AKCnANC. Из треугольника АКС находим ZKCA = 90°-70° = = 20°. Из треугольника AN С находим ZNAC = 90°-80° = 10°. Рассмотрим треугольник АМС и найдем ZAMC = 180°-(20° + 10°) = 150°. Способ М 2. Рассмотрим треугольник ABC. ZB = 180°-(80° + 70°) = 30°. 27
к Рис.3 Из треугольника КС В находим ZKCB = 90° -30° = 60°. Из треугольника MNC находим ZNMC = 90°-60° = 30°. Так как углы NMCnAMC — смежные, то ZAMC = 180°-30° = 150°. Способ № 3. Из треугольника ABC находим ZB = 30°. Из треугольника ABN находим ZBAN = 90°-30° = 60°. Из треугольника АКМ находим ZKMA = 90°-60° = 30°. Так как углы КМАиАМС — смежные, то АМС = 150°. Способ № 4. Из треугольника ABC находим ZB = 30°. Так как ZKBN + ZBNM + ZNMK + ZMKB = 360° (данный факт легко доказывается, если провести диагональ в четырехугольнике KBNM), то ZKMN = 360о-90°-90о-30° = 150°. А так как углы KMN и АМС — вертикальные, то ZAMC = 150°. 5. Чему равен угол между биссектрисами вертикальных углов? А смежных углов? (Изменение содержания задачи развивает гибкость ума.) 6. Вычислите: ^ + т;т + |;^ + |;т|. (Первые 3 примера Z 44 о Z о4о предложены одного типа, а четвертый — другого. (Для решения четвертого примера необходимо перестроить умственную деятельность.) 7. Решите задачу: «За 18 дней бригада лесорубов в составе 15 человек заготовила 972 м3 дров. Сколько дров заготовит бригада из 12 человек за 25 дней при такой же производитель- 28
ности труда?». Поставьте новый вопрос к задаче. Измените в соответствии с ним условие исходной задачи и решите новую задачу. Найдите другой способ только что решенной задачи. 8. Измените условие задачи: «Докажите, что каждый из углов равностороннего треугольника равен 60°» таким образом, чтобы в условии задачи были те же понятия, что и в заключении. (Видоизменение задачи развивает гибкость ума.) Для развития глубины ума на уроке надо учить учащихся: • выделять главное отношение в задаче; • выделять существенные признаки понятия; • вычленять ведущие закономерные отношения явлений; • отделять главное от второстепенного, уметь извлекать из текста не только то, что в нем сказано, но и то, что содержится «между строк»; • видеть главные причины происходящего, объяснять их сущность и т. д. Рассмотрим примеры задач, способствующих развитию данного качества. Упражнения на развитие глубины ума 1. Известно, что сложению соответствует одно обратное действие — вычитание; аналогично для умножения обратным действием является деление. Почему же действие возведение в степень имеет два себе обратных: извлечение корня и логарифмирование? (Для возведения числа в степень переместитель- ный закон не действует в отличие от сложения и умножения.) 2. Является ли последовательность вида 3, 3, 3, ... арифметической прогрессией? А геометрической? 3. Подчеркните наиболее общее понятие: медиана, отрезок, хорда, средняя линия треугольника. 4. Выделите основное соотношение в задаче: «Два поезда вышли одновременно навстречу друг другу из двух городов, расстояние между которыми 660 км. Через 4 часа они встретились. Найдите скорость каждого поезда, если скорость одного на 15 км/ч больше скорости другого». 5. Выделите существенные признаки понятий «равнобедренный треугольник », « ромб ». 29
Иногда одна и та же задача может развивать различные качества ума. Упражнения на развитие нескольких качеств ума 1. Вася живет на 5 этаже 12-этажного дома. Он решил покататься на лифте. Сначала он поднялся на 2 этажа, потом опустился на 4 этажа, потом поднялся на 6 этажей, потом опустился на 10 этажей, потом вновь поднялся на 3 этажа. На каком этаже в итоге Вася оказался? (Развитие осознанности и гибкости ума.) Решение. 5 + 2—4 + 6—10 + 3 = 2, но в процессе решения получалось 5 + 2-4 + 6-10= -1. Так как в процессе решения получилось -1, то в задаче есть противоречивые данные. Но, если под «-1 этажом» дома понимать подвал, то все получается. Ведь лифт может опускаться и в подвал иногда! 2. Катеты прямоугольного треугольника равны 3 и 4 см, а высота, проведенная к гипотенузе, равна 2 см. Чему равна гипотенуза треугольника? (Развитие осознанности и глубины ума.) Решение. Так как Sa = i • 3 • 4 = 6 (см2). Но Sa = ^ • 2 • с = с, поэтому с = 6 (см). Но по теореме Пифагора: с = Va2 + Ь2 = 5 (см). Это задача с противоречивыми данными в условии. Противоречие можно получить и другим способом, найдя длины отрезков, на которые основание высоты разбивает гипотенузу. 3. Докажите тождество: а2(х-Ь)(х-с) Ь2(х-с){х-а) с2(х-а)(х-Ь) _ 2 (a-b)(a-c) (b-c)(b-a) (c-a)(c-b) (При оригинальном решении данной задачи (рассмотрев данное тождество, как уравнение относительно переменной х, а это уравнение будет не выше второй степени, мы получим, что три значения: х = а, х = b> x = с удовлетворяют этому «уравнению», а, следовательно, данное равенство является тождеством) развиваются гибкость, глубина, критичность и другие качества ума.) 30
Рассмотренные качества ума: гибкость, глубина и другие — являются основными составляющими такой интеллектуальной особенности, как обучаемость учащихся математике, которую можно развивать как на уроке, так и вне урока. Основной путь развития этой интеллектуальной особенности через применение на уроке различных нестандартных и олим- пиадных задач мы рассмотрели. Важным и необходимым условием повышения уровня обучаемости учащихся математике является и развитие у них приемов умственной деятельности. Рассмотрим основные типы упражнений для развития некоторых приемов. Для развития анализа необходимо: • применять дополнительные построения, нестандартные идеи для решения той или иной задачи; • обучать применению нисходящего и восходящего анализа для решения задач; • обучать нахождению достаточных признаков справедливости заключения, отбирать требуемый признак для решения задачи и т. д. Приведем примеры упражнений для развития этого важного приема умственной деятельности. Упражнения на развитие приема умственной деятельности анализа 1. Можно ли треугольник разбить двумя прямыми на: а) 5 треугольников; б) 8 треугольников? 2. Можно ли разбить равнобедренный треугольник на: а) 4; б) 5; в) 6; г) 7; д) 2008; е) 2009 равнобедренных треугольников? Если можно, то покажите как? 3. Может ли угол при основании равнобедренного треугольника равняться 100°? 4. Каков вид треугольника, если: а) один из его углов больше суммы двух других углов; б) сумма любых его двух углов больше 90°? 5. Что достаточно знать, чтобы утверждать, что на рис. 4 треугольники С АО и BDO были равными? 31
Рис.4 Приведем примеры упражнений, предназначенных для развития других приемов умственной деятельности. Упражнения на развитие приема умственной деятельности классификации 1. Выделите основные типы задач по изученной теме «Проценты». 2. Постройте различные классификации четырехугольников. 3. Вычеркните одно лишнее слово: параллелограмм, ромб, трапеция, квадрат, прямоугольник. 4. Исключите из 5 данных на рис. 5 геометрических объектов лишний. Рис.5 Упражнения на развитие приема умственной деятельности сравнения 1. Сравните параллелограмм и трапецию. 2. Сравните треугольник и тетраэдр. 3. Что общего у прямоугольника и ромба? 4. В чем отличие равностороннего треугольника от квадрата? А чем они похожи? 32
5. Посмотрите на рис. 6 и скажите, что общего у изображенных фигур и в чем их отличие. Рис.6 6. Какая из изображенных фигур на рис. 7 отличается от остальных и чем отличается? Рис.7 Упражнения на развитие приема умственной деятельности абстрагирования 1. Выберите из 5 предложенных математических терминов: прямые, отрезки, лучи, точка, треугольник два, которые бы наиболее точно определяли понятие угол. 2. Выделите существенные признаки понятия «треугольник». Упражнения на развитие приема умственной деятельности аналогии 1. Решите задачу способом аналогичным только что решенной задаче. 2. Найдите четвертое понятие, которое бы так соотносилось с третьим понятием, как первое со вторым: угол — вершина угла; окружность — ? 3. На рис. 8 в верхнем ряду изображены 3 фигуры. Подумайте, как связаны первые две из них, и укажите в наборе (а-г) четвертую фигуру, которая точно так же связана с третьей. зз
Рис.8 Известно, что между приемами умственной деятельности и качествами ума есть связь. Развитие некоторых приемов умственной деятельности способствует и развитию определенных качеств ума. Например, предлагая задачи для развития приемов «анализ» и «синтез» развивается гибкость ума и наоборот — развивая гибкость ума, мы способствуем развитию приемов «анализ» и «синтез». Формирование же приемов абстрагирования и обобщения способствует развитию глубины ума. Есть, конечно, и другие пути развития обучаемости учащихся математике, но это уже отдельный разговор. 3. Другие виды подготовительной работы учителя математики к олимпиадам на уроке Мы рассмотрели два основных вида подготовительной работы учителя математики к олимпиадам на уроке. Также на уроках можно применять и другие приемы. В качестве задач для работы с наиболее сильными учащимися не надо предлагать как слишком простых, так и слишком сложных задач. Они не оказывают существенного влияния на интеллектуальное развитие учащихся. Контрольные работы и зачеты по-прежнему остаются основной формой контроля уровня обученности учащихся. В числе последних заданий текстов контрольных работ (или в качестве дополнительного задания) необходимо предлагать и олим- пиадные задачи. 34
В качестве одного из путей подготовки к олимпиадам необходимо для учащихся в домашние задания включать задачи типа: придумай задачи к такому-то разделу; составь задачу, аналогичную рассмотренной в классе; олимпиадные задачи прошлых лет и т. п. Не будет необычным, если иногда и сильные учащиеся не справятся с домашним заданием. В качестве домашнего задания на неделю, особенно в 5- 6 классах, можно предлагать и домашние олимпиады, включая в их тексты преимущественно олимпиадные задачи. Предложенные учащимся задачи учащиеся решают дома, при этом могут пользоваться имеющейся литературой, а в случае затруднений и советоваться с родителями. За решение предложенных задач учащиеся каждую неделю получают отметку, а по итогам четверти подсчитывается средний балл, который учитывается при выставлении четвертной отметки. Для заинтересованности учащихся в решении олимпиадных задач в конце четверти, года лучшие из учащихся поощряются призами, которыми чаще всего являются интересные и полезные книги по математике. В качестве примера рассмотрим по несколько вариантов домашних олимпиад. 5 класс Вариант 1 1. Как, используя цифру 5 пять раз, знаки арифметических действий и скобки, выразить все натуральные числа от О до 10 включительно? 2. У щенят и цыплят вместе 44 ноги и 17 голов. Сколько щенят и сколько утят? 3. Если школьник купит 11 тетрадей, то у него останется 5 рублей. А на 15 тетрадей у него не хватает 7 рублей. Сколько денег у школьника? 4. Как, имея два сосуда вместимостью 5 л и 7 л, налить из водопроводного крана 6 л? 35
5. Как разрезать прямоугольник, длина которого 16 см, а ширина 9 см, на две равные части, из которых можно составить квадрат? Вариант 2 1. Установите закономерность в последовательности чисел и запишите еще три числа: 7, 8, 12, 21, 37,... 2. Разместите на трех грузовиках 7 полных бочек, 7 бочек, наполненных на половину, и 7 пустых бочек так, чтобы на всех грузовиках был одинаковый по массе груз? 3. На школьной викторине участникам предложили 20 вопросов. За правильный ответ ученику ставилось 12 очков, а за неправильный списывали 10 очков. Сколько правильных ответов дал один из учеников, если он ответил на все вопросы и набрал 86 очков? 4. Сколько прямоугольников изображено на рис. 9? Площадь каждого квадрата равна 1 кв. ед. Рис.9 5. Сколько нулей стоит в конце произведения всех натуральных чисел от 10 до 25? Вариант 3 1. Петя провел три прямые линии и отметил на них 6 точек. Оказалось, что на каждой прямой он отметил 3 точки. Покажите, как он это сделал. 2. Внучке столько месяцев, сколько лет дедушке. Вместе им 91 год. Сколько лет дедушке и сколько лет внучке? 3. В трех мешках находятся крупа, вермишель и сахар. На одном мешке написано «крупа», на другом — «вермишель», на 36
третьем — «крупа или сахар». В каком мешке что находится, если содержимое каждого из них не соответствует записи? 4. Три охотника варили кашу. Один положил 2 кружки крупы, второй — 1 кружку, а у третьего крупы не было. Кашу же они съели все поровну. Третий охотник и говорит: «Спасибо за кашу! В благодарность я даю вам 5 патронов, но как их поделить в соответствии с вашим вкладом в мою порцию каши?» 5. Четверо девочек выбирали водящую с помощью считалки. Тот, на кого падало последнее слово, выходил из круга, и счет повторялся вновь. Считающая девочка каждый круг начинала с себя и в результате стала водящей, причем счет каждый раз кончался перед ней. Какое наименьшее число слов могло быть в считалке? 6 класс Вариант 1 1. Поставьте вместо звездочек цифры: ,59,27 **,45 78,*3 182,1* 2. В ведре вместимостью 6 л находится 4 л молока, а в 7-литровом — 6 л. Пользуясь этими ведрами и пустой 3-литровой банкой, разделите молоко пополам. 3. Можно ли шахматную доску разрезать на прямоугольники размером 3x1? 4. Разместите восемь козлят и девять гусей в пяти хлевах так, чтобы в каждом хлеве были и козлята и гуси, а число их ног равнялось 10. 5. На столе стоят 3 одинаковых ящика, в одном находятся 2 черных шарика, в другом — 1 черный и 1 белый шарик, в третьем — 2 белых шарика. На ящиках написано: «2 белых», «2 черных», «черный и белый». При этом известно, что ни одна из надписей не соответствует действительности. Как, вынув 37
только 1 шарик, определить правильное расположение надписей? Вариант 2 1. Даны числа 1, 2, 3, 4, 5, 6, 7, 8, 9. Расставьте их так, чтобы сумма их на каждой стороне треугольника (см. рис. 10) была равна 20. Рис.10 2. Найдите все дроби со знаменателем 15, которые больше ^ и меньше 1. 3. Переложите одну из семи спичек, изображающих число —, записанное римскими цифрами (т. е. -тт-) так, чтобы полу- Ю .л. а 2 чившаяся дробь равнялась -. о 4. Возраст старика Хоттабыча записывается числом с различными цифрами. Об этом числе известно следующее: • если первую и последнюю цифру зачеркнуть, то получится двузначное число, которое при сумме цифр, равной 13, является наибольшим; • первая цифра больше последней в 4 раза. Сколько лет старику Хоттабычу? 5. Инопланетяне сообщили жителям Земли, что в системе их звезды три планеты А, Б, В. Они живут на второй планете. Далее передача сообщения ухудшилась из-за помех, но было 38
принято еще два сообщения, которые, как установили ученые, оказались оба ложными: а) А — не третья планета от звезды; б) Б — вторая планета. Какими планетами от звезды являются А, Б, В? Вариант 3 1. Вместо звездочек расставьте пропущенные цифры: Х785 * * * * * * 1 * * * * * * ***** 2. Некоторый товар стоил 500 рублей. Затем цену на него увеличили на 10%, а затем уменьшили на 10%. Какой стала цена в итоге? 3. К числу 15 припишите слева и справа по одной цифре так, чтобы полученное число делилось на 15. 4. В летний лагерь приехали отдыхать три друга: Миша, Володя и Петя. Известно, что каждый из них имеет одну из следующих фамилий: Иванов, Семенов, Герасимов. Миша — не Герасимов. Отец Володи — инженер. Володя учится в б классе. Герасимов учится в 5 классе. Отец Иванова — учитель. Какая фамилия у каждого из трех друзей? 5. Найдите площадь изображенного на рис. 11 треугольника, если площадь каждой клетки равна 1 см2. В более старших классах в качестве домашнего задания можно предлагать задания по всей изучаемой теме, в числе последних из которых включать и олимпиадные задачи. Рассмотрим примеры подобного рода задач. 1. Сравните числа: \/2008 + \/2006 и 2\/2007. 2. Сравнить с единицей число 0,99999* '°0001 • 1,000010'99999. 3. Докажите, что если а(а + Ь + с) < 0, то уравнение ах2 + Ьх + с = 0 имеет 2 действительных корня. 39
Рис.11 4. Найти сумму коэффициентов многочлена, получающегося после раскрытия скобок и приведения подобных членов в выражении ч2011 ч2010 5. Из листа бумаги вырезали произвольный треугольник. Можно ли так загнуть три его угла, чтобы оставшаяся часть треугольника оказалась накрытой без просветов и наложений? 6. На доске был нарисован параллелограмм ABCD и отмечены середина Е стороны АВ и середина F стороны CD. Дежурный стер параллелограмм, но оставил точки А, Е, F. Как по этим точкам восстановить параллелограмм? 7. Какой треугольник надо взять, чтобы после проведения в нем одного отрезка получить все известные виды треугольников: равносторонний, равнобедренный, разносторонний, прямоугольный, остроугольный, тупоугольный? 8. Найдите углы треугольника со сторонами а, Ь, с, если его площадь S равна \(а2 + Ь2). 4 9. Пусть а и Ь — катеты прямоугольного треугольника, а с — гипотенуза. Что больше: а3 + Ьг или с3? 10. Какие треугольники можно разрезать на два равнобедренных треугольника? 11. Парус имеет вид четырехугольника ABCD, углы А, С и D которого равны 45° (см. рис. 12). Найдите площадь паруса, если BD = 4 м. 12. На реке расположено два острова А и В. Туристы, отправившись от острова А, желают попасть на остров В, побывав 40
D С Рис.12 поочередно на обоих берегах реки. Как они должны проложить маршрут, чтобы путь имел бы наименьшую длину (берега реки считать прямыми линиями, а острова А и В — точками)? 13. Докажите, что sin 10° — число иррациональное. 14. Решить уравнение: 3* + 4* = Iх. 15. Решить уравнение: sin20 x • cos24 x = 0,0001. Подобного рода задачи можно найти в разделе 6. Но все же работа с сильными учащимися по математике — работа штучная. Поэтому не обойтись без индивидуальной работы как на уроке, так и вне урока. И если в классе есть несколько одаренных детей, которые проявляют себя как раз в решении олимпиадных задач, то с ними необходимо организовать специальную работу, которая будет направлена на развитие их одаренности. Рассмотрим некоторые особенности работы с данными учащимися. Лучшим вариантом для таких детей был бы перевод их на индивидуальное обучение в рамках данного учебного заведения или в учебное заведение повышенного статуса (лицей, школа с углубленным изучением математики), но не каждая школа имеет такую возможность и желание. Поэтому с такими детьми должна быть иначе организована вся система классной и домашней работы. В классе необходимо предлагать другие, более трудные задачи, которые бы несли большую интеллектуальную нагрузку, но при этом не занимали много времени. Акцент в работе с такими учащимися должен был сделан на самостоятельное обучение. Домашние задания следует предлагать в такой форме, которая предполагает собственный выбор не только в отношении трудности и объема выполняемой рабо- 41
ты, но и в отношении самого характера работы. Это могут быть как придумывание задач к разделу, темам, которые являются наиболее интересными, так и решение более трудных олимпи- адных задач. Конечно, таких детей нужно охватить различными формами внеклассной и внешкольной работы, которые бы способствовали их развитию. Итак, переходим к другим направлениям работы учителя математики с целью лучшей подготовки учащихся к математическим олимпиадам. II. Внеклассная работа по математике Под внеклассной работой по математике понимаются необязательные систематические занятия учащихся с преподавателем во внеурочное время. В теории и методике обучения математике различают два типа внеклассной работы. К первому типу относится внеклассная работа с учащимися, отстающими от других в изучении программного материала (дополнительные занятия после уроков). Основной целью ее является своевременная ликвидация (и предупреждение) имеющихся у учащихся пробелов в знаниях и умениях по курсу математики. Вторым типом внеклассной работы является работа с учащимися, проявляющими к изучению математики повышенный, по сравнению с другими, интерес и способности. Последний тип и является собственно внеклассной работой в традиционном понимании этого слова. Как раз этот тип и будет применяться как для подготовки, так и для проведения математических олимпиад. Наиболее важными задачами внеклассной работы на современном этапе развития школы являются следующие: • пробуждение и развитие устойчивого интереса учащихся к математике и ее приложениям; • расширение и углубление знаний учащихся по программному материалу; 42
• развитие математических способностей и мышления у учащихся; • развитие у учащихся умения самостоятельно и творчески работать с учебной и научно-популярной литературой; • создание актива, способного оказать учителю математики помощь в организации эффективного обучения математике всего коллектива данного класса; • расширение и углубление представлений учащихся о практическом значении математики в технике, экономике; • расширение и углубление представлений учащихся о культурно-исторической ценности математики, о роли ведущих ученых-математиков в развитии мировой науки; • осуществление индивидуализации и дифференциации; • разностороннее развитие личности. Внеклассная работа может осуществляться в самых разнообразных видах и формах. Условно можно выделить следующие три основные вида внеклассной работы. 1. Индивидуальная работа — работа с учащимися с целью руководства внеклассным чтением по математике, подготовкой докладов, рефератов, математических сочинений, изготовлением моделей; работа с консультантами; подготовка некоторых учащихся к участию в городской (районной) или областной олимпиаде. 2. Групповая работа — систематическая работа, проводимая с достаточно постоянным коллективом учащихся. К ней можно отнести факультативы, кружки, спецкурсы, элективные курсы. 3. Массовая работа — эпизодическая работа, проводимая с большим детским коллективом. К данному виду относятся вечера, научно-практические конференции, недели математики, олимпиады, конкурсы, соревнования и т. п. На практике все эти три вида внеклассной работы тесно связаны друг с другом. Для подготовки к олимпиадам можно использовать следующие формы внеклассной работы по математике: факультативы, кружки, недели (декады) математики, стенную печать. Кружки (факультативы, спецкурсы) являются основной формой работы с наиболее способными учащимися по мате- 43
матике. Только здесь можно рассмотреть особые типы задач, относящиеся к олимпиадным задачам. В частности, в 5-6 классах можно рассмотреть различные типы логических задач, задачи на применение некоторых инвариантов, математические ребусы, задачи на разрезание, геометрические упражнения со спичками и др. В 7-8 классах — рассмотреть принцип Дирихле, игры, графы, решение более сложных логических задач. А в 9-11 классах — решение уравнений в целых числах, решение нестандартных уравнений. Конечно, будут и другие темы, не предназначенные для изучения специальных методов решения олимпиадных задач, а направленные на реализацию других целей работы кружка (факультатива, спецкурса). Также некоторые занятия кружка (факультатива) можно посвятить и развитию каких-то определенных качеств ума, приемов умственной деятельности, подобрав специальные упражнения, организовав эти занятия в виде практикумов, тренингов и т. п. На занятиях кружков (факультативов и спецкурсов) нужно проводить и математические соревнования, и игры. Они необходимы как для текущего контроля степени усвоения рассмотренного материала, так и для психологической подготовки к будущим олимпиадам. В качестве таких соревнований и игр наиболее часто используются: • брейн-ринг, • математическая регата, • устная олимпиада, • математическая карусель, • математическая драка, • конкурс «Начинающий математик», • математическая игра «Счастливый случай», • игра «Математик — бизнесмен» и др. К сожалению, сегодня не во всех школах для учащихся 5- 8 классов организованы математические кружки. Объяснить это можно различными причинами, в том числе и такими: • мало учащихся, желающих заниматься в кружках; 44
• ряд регионов не проводит районных олимпиад в этих классах, поэтому учителя не видят смысла готовить учащихся к олимпиадам; • учителя математики перегружены, им не оплачивается проведение внеклассной работы и т. п. Несмотря на все причины, вряд ли политика администрации таких школ является правильной. Сегодня в школы введены ставки педагогов дополнительного образования, для награждения учителей грантами Президента России учитывается и проведение внеклассной работы, ее результативность. Но добиться успеха в олимпиадах без этой действенной формы внеклассной работы вряд ли удастся. Тем более что сегодня имеется достаточное количество литературы для проведения кружковых занятий. В качестве примера приведем хотя бы книгу автора: ФарковА. В. Математические кружки в школе. 5-8 классы. — М.: Айрис-пресс, 2005-2007 гг. Подготовка к олимпиадам проводится и при проведении недель (декад) математики: все зависит от плана их проведения. Если в плане недели математики есть конкурсы по решению задач, различные соревнования — это способствует подготовке учащихся к дальнейшим олимпиадам. На математических вечерах, которые иногда завершают недели математики, проводятся разнообразные конкурсы, эстафеты, в число заданий которых часто входят и олимпиадные задачи. Часто в неделю математики проводится и сама школьная олимпиада. Кроме олимпиад, желательно проводить и другие соревнования, получившие широкое распространение в некоторых школах в последние годы. Ведь только соперничество между несколькими более сильными учащимися, нежелание уступать их друг другу в этих соревнованиях, будут способствовать тому, что учащиеся больше будут читать дополнительной литературы, участвовать во внеклассной и внешкольной работе. Тем не менее при проведении математических соревнований необходимо соблюдать меру. Будет вполне приемлемым, если математические соревнования разных видов будут проводиться в школе 3-4 раза в течение года. Например, осенью можно провести для учащихся 5-11 классов традиционные математические олимпиады. Зимой же для учащихся разных клас- 45
сов провести различные математические соревнования, например, турниры Архимеда (4-6 классы), регаты (7-8 классы), карусели (9 классы), бои (10-11 классы). В марте учащиеся 5- 10 классов принимают участие в международной олимпиаде — конкурсе «Кенгуру». А учебный год завершить в мае еще одной олимпиадой, например устной, или каким-то командным соревнованием. Стенная печать также оказывает свое влияние на подготовку учащихся к олимпиадам, если в математических газетах есть «Уголок смекалки», «Подумай» и т. п., в которых помещаются как занимательные задачи, так и софизмы, парадоксы, арифметические ребусы, задачи с различных математических соревнований, а также ответы к этим задачам. Также в газете может быть раздел «Познакомься с методом решения», в который помещаются наиболее интересные задачи из журнала «Математика в школе» или из других источников. III. Внешкольная работа по математике В отличие от внеклассной работы, которая проводится с учащимися одной школы учителями математики (а иногда и родителями учащихся) этой же школы, внешкольная работа по математике организуется с учащимися нескольких школ какого-то города, района или региона. При этом внешкольные занятия с учащимися могут организовываться как на базе какой-то школы, так и на базе вузов, Центров дополнительного образования, Домов творчества и т. п. Внешкольная работа, прежде всего, предназначена для учащихся, уже увлеченных математикой. Основными целями организации внешкольной работы являются: • развитие мышления и математических способностей учащихся; • углубление знаний учащихся по математике. Основными формами внешкольной работы по математике на сегодня являются: 46
• математические кружки и факультативы при опорных школах, вузах, Домах творчества, Центрах дополнительного образования; • летние математические школы; • математические соревнования между школами, городами (различные виды олимпиад, кубок Колмогорова, Уральские турниры ...); • районные и городские научные конференции школьников. Многие из данных форм могут использоваться и для подготовки учащихся как к олимпиадам, так и к другим соревнованиям. Проводят внешкольную работу, как правило, преподаватели и студенты вузов, работники Центров дополнительного образования, Домов творчества, а также и учителя некоторых школ. Задача учителя математики в данном слечае — владеть информацией обо всех формах внешкольной работы, которые могут посещать его ученики. Не каждый учитель может обладать такими качествами, которые позволят ему подготовить призера региональной или всероссийской олимпиады, поэтому иногда без привлечения других специалистов, добиться продвижения ученика будет невозможно. И инициатива в данном случае должна идти от руководителей школ и методистов отделов образования, они должны решать проблемы дополнительного математического образования учащихся своей школы, района. А в качестве таких специалистов могли бы выступить как некоторые учителя математики из близлежащих школ, так и педагоги дополнительного образования и, конечно же, преподаватели вузов. Только совместная работа учителя математики, педагога дополнительного образлвания и преподавателя вуза может принести успех. Итак, рассмотрим еще одно направление работы учителя математики по подготовке к олимпиадам. IV. Заочная работа Одним из направлений для подготовки к олимпиадам является и заочная работа в различных школах при вузах. Одной из 47
таких известных всероссийских школ является школа «Авангард» (г. Москва). Уровень предлагаемых там заданий очень высок, большинство идей в предлагаемых заданиях встречается в различного уровня олимпиадах. И выполнение такого рода заданий будет способствовать, конечно же, подготовке учащихся к олимпиадам. Во многих крупных городах имеются школы одаренных детей, в вузах — факультеты (отделения, центры) довузовской подготовки. В них можно обучаться и по заочной форме. Задача учителя математики будет заключаться в том, чтобы донести информацию о них до своих учеников, убедить некоторых из них в необходимости заочного обучения в данных школах, на данных факультетах. Также часто некоторые журналы, газеты объявляют различные конкурсы для любителей решать разнообразные задачи. Учителю математики необходимо найти время и уделить внимание этим конкурсам. А затем, когда кто-то из его участников примет участие в данных конкурсах, надо не забыть сказать об этом, тем более, если участник покажет хороший результат. Только задействовав все эти четыре направления в подготовке учащихся к олимпиадам (хотя, это для жизни не главное, куда главнее интеллектуальное развитие ученика, подготовка его к современной жизни, где без острой конкуренции уже не обойтись), можно ожидать успеха.
Раздел четвертый ПРИМЕРНЫЕ ТЕКСТЫ ШКОЛЬНЫХ ОЛИМПИАД Составить тексты олимпиад, которые подошли бы каждой конкретной школе, невозможно. Это связано и с разным уровнем способностей учащихся, и с использованием различных учебников. Поэтому в данном разделе предлагается несколько вариантов для каждого класса, с помощью которых учитель математики может самостоятельно подготовить текст школьной или классной олимпиады. Часть предлагаемых текстов составлена автором. Основой других текстов олимпиад являются тексты, составленные учителями школ России, в основном юга Архангельской области, которые переработаны автором. В качестве примера во всех первых вариантах поставлено число баллов, которыми можно оценить правильное решение задачи. 5 класс Вариант 1 1. Расшифруйте два ребуса, в которых одинаковым буквам соответствуют одинаковые цифры, а разным буквам — разные цифры в обоих примерах. (3 б.) ,АБВ VABB ВВ ВВ ААБ , АБВ АБВ АГАВ 2. Летели утки: одна впереди и две позади, одна позади и две впереди, одна между двумя и три в ряд. Сколько всего летело уток? (3 б.) 3. Докажите, что из трех целых чисел всегда можно найти два, сумма которых делится на два. (4 б.) 49
4. Найдите наибольшее целое число, дающее при делении на 13 с остатком частное 17. (5 б.) 5. Определить расстояние АВ и расстояние между точками О и М, если М — середина отрезка АВ, ОА = а, ОВ = Ь. (6 б.) 6. Из числа 12345678910111213...5657585960 вычеркните 100 цифр так, чтобы оставшееся число стало наибольшим. (8 6.) Вариант 2 1. Вычеркните в числе 4000538 пять цифр так, чтобы оставшееся число стало наибольшим. 2. Для того чтобы разрезать металлическую балку на две части, нужно уплатить за работу 5 рублей. Сколько будет стоить работа, если балку нужно разрезать на 10 частей? 3. Парусник отправляется в плавание в понедельник в полдень. Плавание будет продолжаться 100 часов. Назовите день и час его возвращения в порт. 4. Разбейте циферблат часов (см. рис. 13) с помощью отрезков на три части таким образом, чтобы сумма чисел в каждой из этих частей была одной и той же. 5. На улице, став в кружок, беседуют четыре девочки: Аня, Валя, Галя, Надя. Девочка в зеленом платье (не Аня и не Валя) стоит между девочкой в голубом платье и Надей. Девочка в белом платье стоит между девочкой в розовом платье и Валей. Какое платье носит каждая из девочек? 6. Соедините точки А и В (см. рис. 14) линией длиной 19 см так, чтобы она прошла через все точки, изображенные на рисунке (расстояние между двумя соседними точками, расположенными горизонтально или вертикально, равно 1 см). Вариант 3 1. Сколько раз к наибольшему однозначному числу надо прибавить наибольшее двузначное число, чтобы получить наибольшее трехзначное. 50
В Рис.13 Рис.14 2. Расставьте скобки в записи 7-9 + 12 : 3-2 так, чтобы значение полученного выражения было равно а) 23; б) 75. 3. Если Сережа поедет в школу автобусом, а обратно пойдет пешком, то он затратит на весь путь 1 ч 30 мин. Если же в оба конца он поедет автобусом, то затратит всего 30 мин. Сколько времени потратит Сережа на дорогу, если он пойдет пешком и в школу и обратно? 4. Школьный драмкружок, готовясь к постановке отрывка из сказки А. С. Пушкина о царе Салтане, решил распределить роли между участниками. — Я буду Черномором, — сказал Юра. — Нет, Черномором буду я, — заявил Коля. — Ладно, — уступил ему Юра, — я могу сыграть Гвидона. — Ну, я могу стать Салтаном, — тоже проявил уступчивость Коля. — Я же согласен быть только Гвидоном! — произнес Миша. Желания мальчиков были удовлетворены. Как распределились роли? 5. У Ивана имеется деревянный параллелепипед с измерениями 6 см, 12 см, 18 см. Он распиливает его на кубики с ребром 1 см и ставит их один на другой. Сможет ли Иван достроить вышку из этих кубиков, если даже он заберется на трехметровую лестницу. 51
Вариант 4 1. Запишите наибольшее и наименьшее семизначные числа. 2. У щенят и утят вместе 44 ноги и 17 голов. Сколько щенят и сколько утят? 3. Если школьник купит 11 тетрадей, то у него останется 5 рублей. А на 15 тетрадей у него не хватает 7 рублей. Сколько денег у школьника? 4. Как, имея два сосуда вместимостью 5 л и 7 л, налить из водопроводного крана 6 л? 5. Как разрезать прямоугольник, длина которого 16 см, а ширина 9 см, на две равные части, из которых можно составить квадрат? Вариант 5 1. Вычислите: 101101 • 999-101 • 999 999. 2. Разместите на трех грузовиках 7 полных бочек, 7 бочек, наполненных на половину, и 7 пустых бочек так, чтобы на всех грузовиках был одинаковый по массе груз. 3. На школьной викторине участникам предложили 20 вопросов. За правильный ответ ученику ставилось 12 очков, а за неправильный списывали 10 очков. Сколько правильных ответов дал один из учеников, если он ответил на все вопросы и набрал 86 очков? 4. Сколько прямоугольников изображено на рис. 15? Площадь каждого квадрата равна 1 кв. ед. Рис.15 5. Сколько нулей стоит в конце произведения всех натуральных чисел от 10 до 25? 52
Вариант 6 1. Решите уравнение: 2 + 180 : (#-11) = 22. 2. Внучке столько месяцев, сколько лет дедушке. Вместе им 91 год. Сколько лет дедушке и сколько лет внучке? 3. В трех мешках находятся крупа, вермишель и сахар. На одном мешке написано «крупа», на другом — «вермишель», на третьем — «крупа или сахар». В каком мешке что находится, если содержимое каждого из них не соответствует записи? 4. Можно ли треугольник разрезать так, чтобы получилось три четырехугольника? (Если «да», то выполните рисунок.) 5. Даны числа от 1 до 9. Расставьте их в кружки так, чтобы сумма трех чисел вдоль каждой линии (см. рис. 16) была равна 15. Какое число должно быть в центре? Рис.16 Вариант 7 1. В шести кружках, расположенных в форме равностороннего треугольника (см. рис. 17), расставьте числа 31, 32, 33, 34, 35, 36 так, чтобы сумма чисел на всех трех сторонах треугольника была одинаковой и равнялась 100. 2. Чашка и блюдце вместе стоят 25 рублей, а 4 чашки и 3 блюдца стоят 88 рублей. Найдите цену чашки и цену блюдца. 3. На скотном дворе гуляли гуси и поросята. Мальчик сосчитал количество голов, их оказалось 30; а затем он сосчитал 53
количество ног, их оказалось 84. Сколько гусей и сколько поросят было на скотном дворе? 4. Разделите данную фигуру (см. рис. 18) на четыре равные фигуры. Рис. 17 Рис.18 5. Мачеха, уезжая на бал, дала Золушке мешок, в котором были перемешаны мак и просо, и велела перебрать их. Когда Золушка уезжала на бал, она оставила три мешка: в одном — просо, в другом — мак, а в третьем — еще не разобранная смесь. Чтобы не перепутать мешки, Золушка к каждому из них приклеила таблички: «Мак», «Просо», «Смесь». Мачеха вернулась с бала первой и нарочно поменяла местами таблички так, чтобы на каждом мешке оказалась неправильная запись. Ученик Феи успел предупредить Золушку, что теперь ни одна надпись на мешках не соответствует действительности. Тогда Золушка достала только одно-единственное зернышко из одного мешка и, посмотрев на него, сразу догадалась, где что лежит. Как она это сделала? 6. Из 9 монет — одна фальшивая, она легче остальных. Как за два взвешивания на чашечных весах без гирь определить, какая монета фальшивая? 7. Найдите сумму: 1 + 2 + 3 + ... + 111. Вариант 8 1. Найдите среди чисел вида За + 1 первые три числа, которые кратны 5. 54
2. Малыш может съесть 600 г варенья за 6 мин, а Карлсон — в 2 раза быстрее. За какое время они съедят это варенье вместе? 3. Угадайте корни уравнения: 12 : х = 7-х. 4. Квадрат разрезали по ломаной линии, состоящей из трех равных отрезков. Начало разреза в точке А (см. рис. 19). Получили две равные фигуры. Как это сделали? А Рис.19 5. Как с помощью семилитрового ведра и трехлитровой банки налить в кастрюлю ровно 5 литров воды? 6. Догадайся, какие цифры надо поставить вместо звездочек? v**5 Х 4* + 3** *2** 1 * * * * 7. В бутылке, стакане, кувшине и банке находятся молоко, лимонад, квас и вода. Известно, что вода и молоко не в бутылке, сосуд с лимонадом стоит между кувшином и сосудом с квасом, в банке не лимонад и не вода. Стакан стоит около банки и сосуда с молоком. В какой сосуд налита каждая из жидкостей? Вариант 9 1. На прямой линии посажено 10 кустов так, что расстояние между любыми соседними кустами одно и то же. Найди- 55
те это расстояние, если расстояние между крайними кустами 90 дм. 2. Выразите х из формулы а = (х + 8): 9. о 3. Когда велосипедист проехал § пути, лопнула шина. На о остальной путь пешком он затратил вдвое больше времени, чем на велосипедную езду. Во сколько раз велосипедист ехал быстрее, чем шел? 4. В записи 1*2*3*4*5 замените «*» знаками действий и расставьте скобки так, чтобы получилось выражение, значение которого равно 100. 5. Было 9 листов бумаги. Некоторые из них разрезали на три части. Всего стало 15 листов. Сколько листов бумаги разрезали? 6. Для нумерации страниц книги потребовалось всего 1392 цифры. Сколько страниц в этой книге? Вариант 10 1. Угадайте корень уравнения у • у + 5 = 21 и выполните проверку. 2. Попрыгунья Стрекоза половину времени каждых суток красного лета спала, третью часть времени каждых суток танцевала, шестую часть — пела. Остальное время она решила посвятить подготовке к зиме. Сколько часов в сутки Стрекоза готовилась к зиме? 3. Найдите значение выражения: 26-25-25-24 + 24-23-23-22+ 22-21-21-20 +20-19-19 18+ + 18•17-17 - 16 + 16 - 15-15•14. 4. Восстановите запись: х*2*3 * * + ***87 ***** 2*004* 5. В семье четверо детей, им 5, 8, 13 и 15 лет. Детей зовут Аня, Боря, Вера, Галя. Сколько лет каждому ребенку, если 56
одна девочка ходит в детский сад, Аня старше Бори и сумма лет Ани и Веры делится на 3? 6. Приехало 100 туристов. Из них 10 человек не знали ни немецкого, ни французского языка, 75 знали немецкий язык и 83 знали французский. Сколько туристов знали французский и немецкий языки? Вариант 11 210 1. Решите уравнение 3 + —^ = 33. х—о 2. Вычислите площадь фигуры, изображенной на рис. 20. Рис. 20 3. Из 18 одинаковых кубиков сложили прямоугольный параллелепипед высотой в три кубика. Найдите площадь поверхности параллелепипеда, если площадь поверхности одного кубика равна 19 см2. 4. Сумма уменьшаемого, вычитаемого и разности равна 26. Найдите уменьшаемое. Вариант 12 1. 3 ученика делают 3 самолетика за 3 минуты. Сколько учеников сделают 9 самолетиков за 9 минут? 2. Рыбаки поймали 19 рыбин массой 100 г, 200 г,..., 1900 г. Можно ли весь улов поделить поровну между 10 рыбаками? Если можно, то как? Если нет, то почему? 3. Средний возраст 11 игроков футбольной команды 22 года. Когда одного игрока удалили с поля, средний возраст оставшихся игроков стал 21 год. Сколько лет удаленному игроку? 57
4. Цена билета на стадион была 150 руб. После снижения цены билета количество посетителей увеличилось на 50%, а сбор увеличился на 25%. На сколько снизили цену билета? 5. Напишите в строку пять чисел так, чтобы сумма любых двух соседних чисел была отрицательной, а сумма всех чисел положительной. Вариант 13 1. Внуку столько же месяцев, сколько лет бабушке. Бабушке с внуком вместе 52 года. Сколько лет бабушке и сколько лет внуку? 2. Петя провел три прямые линии и отметил на них 6 точек. Оказалось, что на каждой прямой он отметил 3 точки. Покажите, как он это сделал. 3. Три охотника варили кашу. Один положил 2 кружки крупы, второй — 1 кружку, а у третьего крупы не было. Кашу же они съели все поровну. Третий охотник и говорит: «Спасибо за кашу! В благодарность я даю вам 5 патронов, но как их поделить в соответствии с вашим вкладом в мою порцию каши?» 4. Четверо девочек выбирали водящую с помощью считалки. Тот, на кого падало последнее слово, выходил из круга, и счет повторялся вновь. Считающая девочка каждый круг начинала с себя и в результате стала водящей, причем счет каждый раз кончался перед ней. Какое наименьшее число слов могло быть в считалке? 6 класс Вариант 1 1. Решите уравнение: 5(х + 2,6) = 3(2* + 5,2). (3 б.) 2. Дан прямоугольник ABCD, где А(-4;-1), В(3;-1), С(3; 5), D(-4; 5). Задайте с помощью двойного неравенства: а) множество абсцисс всех точек прямоугольника; б) множество ординат всех точек прямоугольника. (4 б.) 58
3. В записи 52*2* замените звездочки цифрами так, чтобы полученное число делилось на 36. Укажите все возможные решения. (5 б.) 4. Сколько воды надо добавить к 600 г жидкости, содержащей 40% соли, чтобы получился 12%-ый раствор этой соли? (8 6.) 5. Ученик вышел из дома в школу в 8 ч утра. В какое время он придет в школу, если до нее 1 км? (9 б.) 6. Олег, Игорь и Аня учатся в 6 классе. Среди них есть лучший математик, лучший шахматист и лучший художник. Известно, что: а) лучший художник не нарисовал своего портрета, но нарисовал портрет Игоря; б) Аня никогда не проигрывала мальчикам в шахматы. Кто в классе лучший математик, лучший шахматист и лучший художник? (10 б.) Вариант 2 1. Поставьте вместо звездочек цифры: ,59,27 **,45 78,*3 182,1* 2. Выразите число 16 с помощью четырех пятерок, соединяя их знаками действий. 3. Найдите два корня уравнения: |-0,63|: |*| = |-0,9|. 4. Разместите восемь козлят и девять гусей в пяти хлевах так, чтобы в каждом хлеве были и козлята и гуси, а число их ног равнялось 10. 5. На столе стоят три одинаковых ящика, в одном находятся 2 черных шарика, в другом — 1 черный и 1 белый шарик, в третьем — 2 белых шарика. На ящиках написано: «2 белых», «2 черных», «черный и белый». При этом известно, что ни одна из надписей не соответствует действительности. Как, вынув 59
только 1 шарик, определить правильное расположение надписей? 1. Решите уравнение: 0,5 • (х + 3) = § • (И-*). о Вариант 3 0,5- (х + 3) 2. Найдите все дроби со знаменателем 15, которые больше - и меньше 1. У 3. Переложите одну из семи спичек, изображающих число —, записанное римскими цифрами (т. е. -==-) так, чтобы полу- ю -X. чившаяся дробь равнялась -. о 4. Возраст старика Хоттабыча записывается числом с различными цифрами. Об этом числе известно следующее: • если первую и последнюю цифру зачеркнуть, то получится двузначное число, которое при сумме цифр, равной 13, является наибольшим; • первая цифра больше последней в 4 раза. Сколько лет старику Хоттабычу? 5. Древнегреческая задача. — Скажи мне, знаменитый Пифагор, сколько учеников посещают твою школу и слушают твои беседы? — Вот сколько, — ответил Пифагор, — половина изучает математику, четверть — природу, седьмая часть проводит время в размышлении и, кроме того, есть еще три женщины. Сколько всего учеников посещают школу Пифагора? Вариант 4 п 1. Решите уравнение: -^ : 3,1 = х : 9,3. У 2. Вместо звездочек расставьте пропущенные цифры: Х785 * * * * * * 1 * * * 60
3. Некоторый товар стоил 500 рублей. Затем цену на него увеличили на 10%, а затем уменьшили на 10%. Какой стала цена в итоге? 4. К числу 15 припишите слева и справа по одной цифре так, чтобы полученное число делилось на 15. 5. В летний лагерь приехали отдыхать три друга: Миша, Володя и Петя. Известно, что каждый из них имеет одну из следующих фамилий: Иванов, Семенов, Герасимов. Миша — не Герасимов. Отец Володи — инженер. Володя учится в 6 классе. Герасимов учится в 5 классе. Отец Иванова — учитель. Какая фамилия у каждого из трех друзей? Вариант 5 1. Даны числа 1, 2, 3, 4, 5, б, 7, 8, 9. Расставьте их так, чтобы сумма их на каждой стороне треугольника (см. рис. 21) была равна 20. Рис. 21 2. Найдите наиболее рациональным способом значение выражения: 25-| • 7 + (1211-41) • 25 + 125 • 357 • 0,008. 7 V Zo о/ 3. Решите уравнение: \х—4\ = 3. 4. Школьник прочитал книгу за три дня. В первый день он прочитал 0,2 всей книги и еще 16 страниц, во второй день — 0,3 остатка и еще 20 страниц. В третий день — 0,75 остатка и последние 30 страниц книги. Сколько страниц в книге? 61
5. Инопланетяне сообщили жителям Земли, что в системе их звезды три планеты А, Б, В. Они живут на второй планете. Далее передача сообщения ухудшилась из-за помех, но было принято еще два сообщения, которые, как установили ученые, оказались оба ложными: а) А — не третья планета от звезды; б) Б — вторая планета. Какими планетами от звезды являются А, Б, В? Вариант 6 1. Выполните действия: 15,81 : (24-23,66)-18 : 37,5. 2. Решите уравнение: |jc—3| = 7. 3. Расшифруйте запись. Одинаковыми буквами обозначены одинаковые цифры, разными буквами — разные цифры. +УДАР УДАР ДРАМА 4. В шестизначном числе первая цифра совпадает с четвертой, вторая — с пятой, третья — с шестой. Докажите, что это число кратно 7, 11, 13. 5. В школьной математической олимпиаде принимали участие 9 учеников шестого класса. За каждую решенную задачу ученик получал 2 очка, а за каждую нерешенную задачу с него списывалось 1 очко. Всего было предложено 10 задач. Докажите, что среди участников олимпиады из шестого класса было, по крайней мере, два ученика, набравших одинаковое число очков. (Считается, что ученик, набравший больше штрафных очков, чем зачетных, набрал ноль очков.) Вариант 7 1. Расшифруйте запись. Одинаковыми буквами обозначены одинаковые цифры, разными буквами — разные цифры. «КОКА КОЛА ВОДА 62
2. Какая часть квадрата (см. рис. 22) закрашена? Рис. 22 3. Один купец прошел через три города, и взыскали с него в первом городе пошлины половину и треть имущества, и во втором городе половину и треть (с того, что осталось), и в третьем городе снова взыскали половину и треть (с того, что у него было); и когда он прибыл домой, у него осталось имущества на 1000 денежных единиц. Узнайте, какова была стоимость имущества у купца? 9 10 11 12 4. Расставьте числа —; —; —; — в порядке убывания. 10 11 \2л а. о 5. Ни у кого из тысячи пиратов Не наберется тысячи дукатов. Но даже самый маленький пират Имеет все же хоть один дукат. Так можно ли сказать о тех пиратах, Что среди них — безусых и усатых, Косматых, безбородых, бородатых — Есть двое одинаково богатых? 6. По кругу написано 2009 натуральных чисел. Докажите, что найдутся два соседних числа, сумма которых четна. Вариант 8 1. Масса бидона с молоком 32 кг, без молока — 2 кг. Какова масса бидона, заполненного молоком наполовину? 63
2. Расшифруйте запись. Одинаковыми буквами обозначены одинаковые цифры, разными буквами — разные цифры. х МИНУС МИНУС * * * *с ***** у * * * *н * * * * и МИНУС ********* 3. Три подруги вышли в белом, синем, зеленом платьях и туфлях таких же цветов. Известно, что только у Ани цвет платья и туфель совпадает. Ни платье, ни туфли Вали не были белыми. Наташа была в зеленых туфлях. Определить цвет платья и туфель каждой подруги. 4. В классе 35 учеников. Из них: 20 школьников занимаются в математическом кружке, 11 — в экологическом, 10 ребят не посещают эти кружки. Сколько экологов увлекается математикой? 5. В школе 33 класса, 1150 учеников. Найдется ли класс, в котором меньше 35 учеников? Вариант 9 1. Первый раз Дима на рыбалку поехал на велосипеде. Рыбы поймал много, поэтому обратно шел пешком. На весь путь он затратил 40 минут. Во второй раз он до реки туда и обратно ехал на велосипеде и затратил 20 минут. Сколько времени Диме потребуется, чтобы пройти путь в оба конца пешком? 2. Разрежьте клетчатый прямоугольник размером 5 х 8 на фигурки из четырех клеток вида (рис. 23). Рис. 23 64
3. На окраску куба размерами 2x2x2 требуется 2 грамма краски. Сколько краски потребуется на покраску куба размером 6x6x6? 4. Маша купила в магазине тетради по 13 рублей и блокноты по 15 рублей. За всю покупку она заплатила ровно 239 рублей. Сколько тетрадей и блокнотов купила Маша? 1. Решите уравнение: Вариант 10 12,3 *- 2,324 46,48* 2. Произведение двух взаимно простых чисел равно 3232. Чему равно наименьшее общее кратное этих чисел? Найдите эти числа. 3. Сравните числа х и г/, если 13,5% числа х равны 12,5% числа у. 4. Прямоугольник разделен двумя отрезками на четыре прямоугольника, площади трех из которых 2 см2, 4 см2, 6 см2 (рис. 24). Найдите площадь прямоугольника. 2 6 4 Рис. 24 Вариант 11 1. В стаде 8 овец. Первая съест копну сена за 1 день, вторая — за 2 дня, третья — за 3 дня,..., восьмая — за 8 дней. Кто быстрее съест копну сена: две первые овцы или все остальные вместе? 2. В начале забега на 1000 м вперед вырвался Андрей, вторым шел Борис, а третьим — Виктор. За время бега Андрей и Борис менялись местами 6 раз, Борис и Виктор — 5 раз, Андрей 65
и Виктор — 4 раза. В каком порядке прибежали спортсмены? Почему? 3. В классе девочек, которым нравится математика, столько же, сколько и мальчиков, которым не нравится математика. Кого в классе больше: учеников, которым нравится математика или мальчиков? 4. Придумайте натуральное число, которое делится на 2004 и сумма его цифр также делится на 2004. 7 класс Вариант 1 1. При каких значениях с уравнение сх = 9: а) имеет корень, равный —9; 0; —; о б) не имеет корней; в) имеет положительный корень? (3 б.) 2. Среди перечисленных выражений укажите такие, которые: а) тождественно равны а2; (-а)2; -(-а)2; -а2; б) тождественно равны а3; (-а)3; -(-а)3; -а3. (4 б.) 3. На сколько процентов увеличится площадь прямоугольника, если его длину увеличить на 20%, а ширину — на 10% ? (5 6.) 4. Постройте график уравнения: а) (х-2)(у + 3) = 0; б) х2 + ху = 0. (6 б.) 5. Вместо «*» расставьте пропущенные цифры: х8* * * * * * * * ***8 (76.) 6. 5 школьников приехали из 5 различных городов в Архангельск на областную математическую олимпиаду. «Откуда вы, ребята?» — спросили их хозяева. Вот что ответил каждый из них. 66
Андреев: «Я приехал из Онеги, а Григорьев живет в Каргополе». Борисов: «В Каргополе живет Васильев. Я же прибыл из Коряжмы». Васильев: «Я прибыл из Онеги, а Борисов — из Котласа». Григорьев: «Я прибыл из Каргополя, а Данилов из Вельска». Данилов: «Да, я действительно из Вельска, Андреев же живет в Коряжме». Хозяева очень удивились противоречивости ответов приехавших гостей. Ребята объяснили им, что каждый из них высказал одно утверждение правильное, а другое ложное. Но по их ответам вполне можно установить, кто откуда приехал. Откуда приехал каждый школьник? (10 б.) Вариант 2 1. Решите уравнение: 2(3-2*) = Зл:-4(1 + Зх). 2. Число 56 разложите на два слагаемых так, чтобы ^ пер- о вого слагаемого была равна j второго. 3. Решите уравнение: \7—х\ = 9,3. 4. Вычислите: „ 46-95 + 69120 84-312-6п 5. Имеется 9 пластинок и двухчашечные весы без гирь. По виду все пластинки одинаковые, но одна из них легче других. Как с помощью двух взвешиваний найти более легкую пластинку? Вариант 3 1. Выразите 10 пятью девятками. Укажите как можно больше способов. 2. Докажите, что при любых значениях букв верно равенство: (х-у)(х + у)-(а-х + у)(а-х-у)-а(2х-а) = 0. 3. Найти все значения х и у, для которых х • у + 1 = х + у. 67
4. Двоим друзьям потребовалось вычислить 42-32. Они заметили, что результат — число 7 — равен сумме оснований квадратов чисел 4 и 3. Проверив свое открытие на числах 10 и 11, друзья установили, что оно подтверждается: 112-102 = = 21 = 11 + 10. После этого друзья нашли все пары (а; Ъ) натуральных чисел а > Ь, для которых разность а2-Ь2 равна сумме а + Ь. Как друзьям удалось найти все такие числа (а; Ь)? 5. Как разрезать квадрат 5x5 прямыми линиями так, чтобы из полученных частей можно было составить 50 равных квадратов? Не разрешается оставлять неиспользованные части, а также накладывать их друг на друга. Вариант 4 1. Трактористы вспахали поле за три дня. В первый день о они вспахали ^ всего поля, во второй день — 40% поля, а в третий день — остальные 72 га. Найдите площадь поля? 2. Решите уравнение: |5л:| • |-1,5| = 12. 3. В некотором месяце три четверга пришлись на четные числа. Какой день недели был 26-го числа этого месяца? 4. Бочка наполнена бензином. Как перелить из нее в мотоцикл 6 л бензина с помощью 9-литрового ведра и 5-литрового бидона? 5. Расставьте знаки действий « + », « —», «х», «:» и скобки, чтобы получить верное равенство: а) 19 99 = 0, б) 1999 = 1, в) 1999 = 3, г) 1999 = 9, д) 19 99 = 10. Вариант 5 1. Из корзины яиц взяли половину всего количества яиц, потом еще половину остатка, затем половину нового остатка и, наконец, половину следующего остатка. В итоге в корзине осталось 10 яиц. Сколько яиц первоначально было в корзине? 2. Вычислите значение выражения: 273-45 55•24 26 - З4 б8 104 б4 68
3. Найдите значение выражения: /810 + 675^ /810 675\ V162 225/ " V162 225/ * 4. Через точку В проведены четыре прямые так, что АВ _1_ ±BD, BE _1_ ВС у и проведена прямая АС, пересекающая данные прямые так, что АВ = ВС. Прямая АС пересекает BD в точке D, АС пересекает BE в точке Е. Докажите, что A ABE = ABCD. 5. Сколько бабушек и прабабушек было у ваших прабабушек и прадедушек? Вариант 6 1. Длину каждой стороны квадрата увеличили на 20% . На сколько процентов увеличилась площадь квадрата? 2. Найдите двузначное число, равное сумме его цифр, увеличенной в 6 раз. 3. Проказница Мартышка, Осел, Козел да Косолапый Мишка, затеявши играть квартет, испробовали все способы усесться на 4 пенька на поляне, прежде чем поверили Соловью, который, как известно, сказал им: «А вы, друзья, как ни садитесь, все в музыканты не годитесь!» Сколько раз им пришлось пересаживаться? 4. Цифру 9, с которой начинается трехзначное число, перенесли в конец числа. В результате получилось число на 216 меньше данного. Какое число было первоначально? 1. Сократите дробь: Вариант 7 521 + 521 + 521 + 521 + 521 524 2. Молодой рыбак положил в уху мало соли. Если бы он положил в уху соли вдвое больше, то досаливать пришлось бы вдвое меньше. Какую долю от нужного количества положил в уху рыбак? 3. Дробь ' ' р ' л с q H равна целому числу, разные буквы обозначают разные цифры, а между ними стоит знак умножения. Чему равна дробь? Ответ обоснуйте. 69
4. Разрежьте треугольник на 2 треугольника, четырехугольник и пятиугольник, проведя 2 прямые линии. 5. В ящике 24 кг гвоздей. Как на чашечных весах без гирь и без стрелки отмерить 9 кг? Вариант 8 1. Двум братьям вместе 35 лет. Сколько лет каждому, если половина лет одного равна трети лет другого? 2. Из 40 т железной руды выплавляют 20 т стали, содержащей 6% примесей. Каков процент примесей в руде? 3. Один фонтан наполняет бассейн за 2,5 ч, а другой — за 3,75 ч. За какое время наполнят бассейн оба фонтана? 4. Постройте график функции у = х + \х\. 5. Решить уравнение: \х + 4| + \х—1\ = 6. 6. На сторонах АВ, ВСиАС равностороннего треугольника ABC взяты соответственно точки Z), Ey Fy так что AD = BE = = CF. Каков вид треугольника DEF1 Докажите. Вариант 9 1. Сколько существует двузначных чисел, в записи которых не употребляется цифра 1? 2. На свои деньги Петя мог бы купить 8 бубликов и 7 пирожных либо 5 бубликов и 8 пирожных. Сколько он смог бы купить одних бубликов? 3. На доске написано число 321321321321. Какие цифры необходимо стереть, чтобы получить возможное наибольшее число, делящееся на 9? 4. Страницы книги пронумерованы подряд с первой до последней. Хулиган Вася вырвал из разных мест книги 25 листов и сложил номера всех пятидесяти вырванных страниц. У него получилось число 2002. Когда об этом узнал отличник Коля, то он заявил, что при счете Вася ошибся. Объясните, почему Коля прав. 5. У звезды ACEBD (см. рис. 25) равны углы при вершинах А и В, углы при вершинах £ и С, а также длины отрезков АС и BE. Докажите, что AD = BD. 70
\D El Рис. 25 Вариант 10 1. Докажите, что выражение (а + Ь)х + (а-Ь)х-2ах тождественно равно нулю. 2. Решите уравнение: 9) (0,3*-12) = 0. 3. Участок под клубнику прямоугольной формы, длина которого в 3 раза больше ширины, окружен оградой, отстоящей от сторон участка на 2 м. Площадь, ограниченная оградой, на 128 м2 больше площади самого участка. Определите длину участка. 4. Число а составляет 80% числа Ь, а число с составляет 140% числа Ь. Найдите числа а, Ъ> с, если известно, что с больше а на 72. 5. Докажите, что всякое натуральное число, запись которого при любом основании системы счисления, превосходящим 2, есть 121, является полным квадратом. Вариант 11 1. Вася в течение суток тратит - часть своего времени на о сон, - — на занятия в школе, - — на встречи с друзьями, ^ 4 5 о своего времени слушает музыку, ^ — играет на компьютере. Можно ли так жить, если каждым из перечисленных дел он занимается отдельно? 71
о 2. Как от куска материи длиной ^ метра отрезать полметра, не имея под руками метра? 3. Подряд записаны числа 1,2,..., 2001, 2002. Каких цифр при записи этих чисел использовано больше: двоек или единиц? На сколько? 4. За весну пес Аргус потерял — своего веса. Затем за лето 4 прибавил в весе - часть. За осень похудел на ^ веса, а за зиму о 1и прибавил ^ веса. Похудел или поправился Аргус за прошед- о ший год? 5. Найдите угол между часовой и минутной стрелкой в 7 ч 38 мин. Вариант 12 1. Две каменные лестницы имеют одинаковую высоту 5 м и одинаковое основание 4 м. Они покрыты ковровыми дорожками. Первая лестница имеет 6 ступенек, а вторая — 8. Хватит ли дорожки, покрывающей ступеньки первой лестницы, для покрытия второй? 2. Поезд проходит мимо светофора за 5 с, а мимо платформы длиной 150 метров за 15 с. Найдите длину поезда и его скорость. 3. При делении двузначного числа на сумму его цифр в частном получается 6, а в остатке 4. Найдите это число. 4. Какой угол образуют стрелки часов в 12 часов 20 минут? Вариант 13 1. Какие цифры надо поставить вместо букв А и Б, чтобы получилось верное равенство: АБ • А • Б = БББ? (Здесь АБ — двузначное число, БББ — трехзначное число). 2. На рыбалке Петя и Ваня вместе поймали рыбы в 3 раза больше, чем Толя. А Ваня и Толя — в 5 раз больше, чем Петя. Кто больше поймал рыбы: Петя и Толя вместе или Ваня? 3. В вершинах пятиугольника стоят рядом 3 фишки. Любую фишку разрешается сдвинуть вдоль диагонали на любое свободное поле. Можно ли получить такую позицию, в которой 72
одна фишка осталась бы на месте, а две другие поменялись бы местами? 4. Отрезки АС и BD пересекаются и АВ = ВС = CD = AD. Докажите, что отрезки АС и BD перпендикулярны. Вычислите площадь фигуры ABCD, если АС = 10 см, BD = 15 см. Вариант 14 1. Фигура, изображенная на рисунке 26, состоит из 7 одинаковых квадратов. Ее периметр равен 16 см. Найдите площадь фигуры. Рис. 26 2. Сосчитайте: 1 + 2-3-4 + 5 + 6-7-8 + 9 + 10-... + 2002-2003-2004 + 2005. 3. Старший брат говорит младшему: «Когда мне было столько лет, сколько тебе сейчас, то я был втрое старше тебя, а когда тебе будет столько лет, сколько мне сейчас, то нам вместе будет 60 лет». Сколько лет братьям? 4. Докажите, что при а = Ь + 1 выполняется следующее тождество: (а + Ь)(а2 + Ь2)(а4 + Ь4)...(а16 + Ь16) = а32-Ь32. 5. В коробке имеются карандаши разного цвета, разной длины и разной толщины. Придумайте такой набор карандашей, чтобы у любых 2 из них совпадал ровно один признак (цвет, толщина или длина). 73
8 класс Вариант 1 1. Упростите выражение: I -^— + ^r^1— I -r- . (3 6.) \J/2—9 3-УУ 5 2. Зная, что ^ = ^, найдите значение выражения: ———. (4 6.) 3. Пассажир едет в поезде, который идет со скоростью 60 км/ч, и видит, что мимо окна проходит встречный поезд в течение 4 с. Какова скорость встречного поезда, если его длина равна 120 м? (5 б.) 4. Постройте график функции: у = \х-3\. (5 б.) 5. Восстановите математическую запись примера: «АННА ВАЛЯ 4809 Здесь разные буквы обозначают разные цифры, а одинаковые буквы — одинаковые цифры. (6 6.) 6. Докажите, что биссектрисы внешних углов прямоугольника, пересекаясь, образуют квадрат. (8 б.) Вариант 2 1. Поставьте знаки модуля так, чтобы равенство 1-2-4-8-16=19 стало верным. 2. Постройте график уравнения (х-1)2 • у = 0. 1 , 1 3. Докажите, что значение выражения есть число рациональное. 4. Постройте треугольник по данной высоте, углу при основании и медиане, проведенной из этого угла. 5. В школе 30 классов и 1000 учащихся. Докажите, что есть класс, в котором не менее 34 учеников. 74
Вариант 3 1. Расшифруйте пример на сложение, где одинаковыми буквами обозначены одинаковые цифры, а разными — разные: ,АБВГ АБВГ ВГДБГ 2. Разложите многочлен xs + jc4 + 1 на три множителя. 3. Вершину А прямоугольника ABCD соединили с серединами сторон ВС и CD. Мог ли один из этих отрезков оказаться вдвое длиннее другого? 4. 2002 человека выстроены в шеренгу. Всегда ли можно расставить их по росту, если разрешается переставлять любых двух людей, стоящих только через одного? Вариант 4 1. Постройте график функции: У х2-1 х+1 2. Разложите на множители: Ца2 + b2) + 21Ь2-20аЬ-36. 3. Сколькими нулями оканчивается произведение всех целых чисел от 1 до 100 включительно. 4. Принцип Дирихле гласит: «Пусть в п клетках сидит не менее чем п + 1 кроликов. Тогда найдется клетка, в которой сидит не менее двух кроликов». Попробуйте применить этот принцип к следующей задаче: «Шесть школьников съели семь конфет. а) Докажите, что один из них съел не менее двух конфет. б) Верно ли, что кто-то съел ровно две конфеты?» 5. Из листа бумаги вырезали произвольный треугольник. Можно ли так загнуть три его угла, чтобы оставшаяся часть треугольника оказалась накрытой без просветов и наложений? 75
Вариант 5 1. Представьте в виде рациональной дроби: 2. Отгадайте ребус: *** |**8** _** ** * * * * * * О 3. Одну овцу лев съедает за 2 дня, волк — за 3 дня, а собака — за 6 дней. За сколько дней они вместе съедят овцу? 4. Постройте график уравнения: х + Зу =1 х + 2у + 1 5. Проверьте равенство: + ... + -1—— = 9. л/2 + 1 л/3 + л/2 \/Ш) + \/99 6. На доске был нарисован параллелограмм ABCD и отмечены середина Е стороны АВ и середина F стороны CD. Дежурный стер параллелограмм, но оставил точки А, £, F. Как по этим точкам восстановить параллелограмм? Вариант 6 1. Вычислите: 20 32 42 56 72 90 110 132* 2. В Л ABC проведены биссектрисы углов Аи В, угол между ними равен 125°. Найдите угол С. 3. Постройте график функции: у = \fx^ + х. 4. Решите уравнение: у 1 + у/2 + у/х = 2. 76
5. Найдите значения а и Ь, при которых равенство Ъх + 31 а + b 2) *-5 л: + 2 выполняется при всех допустимых значениях переменной х. Вариант 7 1. У Димы было 20 рублей. Он купил в магазине х карандашей по 2 рубля каждый. После покупки у Димы осталось у рублей сдачи. Нарисуйте график зависимости у(х). 2. Найдите все такие целые с, при которых дробь — с—4 является целым числом. 3. Найдите все пары натуральных чисел, удовлетворяющих уравнению х2-у2 = 69. 4. Какой треугольник надо взять, чтобы после проведения в нем одного отрезка получить все известные виды треугольников: равносторонний, равнобедренный, разносторонний, прямоугольный, остроугольный, тупоугольный? 5. Разложите на множители выражение л:4 + х2 + 1. 9 класс Вариант 1 1. Запишите число 10 с помощью семи «4», знаков арифметических действий и запятой. (4 б.) 2. Как с помощью циркуля и линейки разделить угол величиной в 19° на 19 равных частей? (6 б.) 3. Расстояние между пунктами А и В — 60 км. Из А в В выходит автомобиль, а из В в том же направлении одновременно с первым автомобилем выходит второй. Если скорость первого автомобиля увеличить на 10 км/ч, а второго — на 8 км/ч, то первый автомобиль догонит второй в том же месте, но на час раньше. Какова скорость каждого автомобиля? (6 б.) 4. 2009*** делится на 2007. Сколько способов существует заменить *** цифрами? (7 б.) 77
5. Докажите, что среди любых шести человек найдутся трое знакомых или трое незнакомых между собой людей. (106.) Вариант 2 1. Найдите значение выражения: при а = 2009. 2. Сократите дробь: хг + 5*2-4jr-20 *2 + 3*-10 3. Решите систему уравнений: = 2(х-у). 4. Постройте график функции: 5. Четыре школьника сделали в магазине покупки: первый купил пенал и ластик, заплатив 40 руб.; второй купил ластик и карандаш, заплатив 12 руб.; третий купил пенал, карандаш и две тетради, заплатив 50 руб.; четвертый купил пенал и тетрадь. Сколько заплатил четвертый школьник? Вариант 3 1. Решите числовой ребус, если каждая цифра встречается всего один раз: *_*_ | * * 2. Запишите числа от 1 до 12 так, чтобы три суммы чисел, записанных в вершинах каждого из 2 прямоугольников и 1 квадрата (см. рис. 27), были одинаковыми. 78
3. Натуральное число х возвели в третью степень. Докажите, что хотя бы одно из чисел jc3 + х или хг—х делится на 10. оо о о Рис. 27 4. Листок календаря частично закрыт предыдущим листком (см. рис. 28). Какая его часть больше — закрытая или открытая? 5. Найдите действительные решения уравнения Вариант 4 1. Антон, Борис и Владимир занимаются различными видами спорта: футболом, плаванием и теннисом. Кто из них каким видом спорта занимается, если известно, что Борис и Владимир не пловцы, а Борис — не теннисист? 2. Задача Безу. Некто купил лошадь и спустя некоторое время продал ее за 24 пистоля. При этой продаже он теряет столько процентов, сколько стоила его лошадь. Спрашивается, за какую сумму он ее купил? 3. Докажите, что если сумма (х2 + у2) делится на 3 и х, у — целые, то х и у делятся на 3. 4. Диагонали выпуклого четырехугольника делят его на 4 треугольника. Докажите, что произведение площадей двух 79
противоположных треугольников равно произведению площадей двух других треугольников. 5. При каких значениях а квадратные трехчлены х2 +ах+1 и х2 + х + а имеют общий корень? Вариант 5 1. Решите уравнение: X2 2. Найдите сумму: 3. Найдите углы треугольника со сторонами а, Ь, с, если его площадь равна S = -(а2 + Ь2). 4. Сколькими способами число 100 можно представить в виде суммы нескольких последовательных натуральных чисел? 5. Можно ли разменять купюру в 50 рублей 15 монетами достоинством 1 и 5 рублей? Вариант 6 1. Решите уравнение: (х2-х-1)2-хг = 5. 2. Автомобиль проехал 600 км. Первую половину пути он двигался со скоростью 100 км/ч, а вторую — 60 км/ч. Найдите среднюю скорость движения автомобиля. 3. Равнобокая трапеция ABCD разбивается диагональю АС на 2 равнобедренных треугольника. Определите углы трапеции. 4. Сколько цифр содержит число 45 • 513? 5. Решите уравнение в целых числах: х + у = ху. 80
10 класс Вариант 1 1. Найдите производную функции: у = (х-6) • хг. (2 б.) 2. Постройте график функции: у = у/2 + sin4 х- cos 2х + л/2 + cos4 х + cos 2*. (4 6.) 3. Заполните пустые клетки таблицы так, чтобы числа в каждой строке и каждом столбце составляли геометрическую прогрессию. (6 б) 27 6 36 8 4. Верно ли, что 2•2009 1 = 2010? 1+2 1+2+3 1 + 2 + 3 + ... + 2009 (б б.) 5. Длина стороны квадрата ABCD равна б см. Точка М удалена от каждой вершины на 17 см. Найдите расстояние от середины отрезка МА до середины каждой из сторон квадрата. (8 6.) Вариант 2 1. Решите неравенство: *4-4л;3 + 12*2-24л; + 24 < 0. 2. Решите уравнение: sin х • cos x - cos 2x • cos 8jc = \ sin 12*. 4 3. Решите уравнение в целых числах: ху = х—у. 81
4. Решите систему уравнений: y + z)=72, y + z) = 120, 5. В окружность радиуса R вписан треугольник, вершины которого делят окружность в отношении 2:5:17. Найдите площадь треугольника. Вариант 3 1. Решите уравнение 2 cos2 x + cos x = 1. 2. Двое рабочих могут выполнить некоторую работу за 7 дней при условии, что второй приступит к ней на 2 дня позже первого. Если бы ту же работу каждый выполнял в одиночку, то первому потребовалось бы на 4 дня больше, чем второму. За сколько дней каждый рабочий мог бы выполнить эту работу? 3. Постройте график функции у = 2х2 + 3|jc| + 2. 4. При каком минимальном натуральном k > 2008 число | 1 | y/k2-2k будет рациональным? 5. Решите в целых числах уравнение х^—Ъху + 2yz = 7. Вариант 4 1. Решите неравенство: (х2-4:Х)2 ^ 16. 2. Известно, что х + — = 5. Найдите х2 + —- 3. При каком целом ft неравенство х2' п <\ верно при любом действительном х? 82
4. Докажите, что уравнение ху = 2009(х+у) имеет решения в целых числах. 5. Можно ли разделить равносторонний треугольник на 2010 равносторонних треугольников? Если да, то как? Если нет, то почему? Вариант 5 н _ l-2|sin;dsin:x; 1. Решите уравнение ' — = 0. /xip—x) 2. Расстояние между серединами сторон АВ и CD выпуклого четырехугольника ABCD равно расстоянию между серединами его диагоналей. Найдите угол, образуемый прямыми ВС и AD при их пересечении. 3. Докажите, что на графике функции у = хг + Зх2 + Зл;-3 есть точка, которая является центром симметрии графика. 4. В настоящее время есть монеты 1, 2, 5 и 10 рублей. Укажите все денежные суммы, которые можно уплатить как четным, так и нечетным числом монет. (Можно использовать одинаковые монеты.) 5. Четыре одинаковые банки с четырьмя разными красками наполнены на три четверти. Имеется возможность переливать любую часть жидкости из одной банки в другую. Можно ли во всех банках сделать одинаковую смесь? (Другой посуды нет и выливать краску нельзя.) Вариант 6 1. Древнеегипетская задача. В одной из задач для первого члена убывающей арифметической прогрессии дается выражение, представляющееся в современной символике следующей формулой: а = — -(п-1)-. Найдите S(n). 2. Найдите sin4 a- cos4 а, если tg ^ = 3. 3. Каким должно быть число Ь, чтобы уравнения хг + Ьх + 1 = 0 и х4 + Ьх2 + 1 = 0 имели общий корень? 83
4. Найдите площадь равнобедренной трапеции, у которой диагональ равна 8 см, а угол между диагоналями равен 45°. 5. Решите в целых числах уравнение: ху + х-Ъу = -6. Вариант 7 l.He выполняя вычислений, докажите, что число 17 не является корнем уравнения 2л:4-Зл:3 + 6х2-5х = 41. 2. В квадрате ABCD на серединах сторон ВС и CD отмечены точки MhJV, которые соединены с вершиной А. Найдите угол MAN. 3. Задача Гипсикла. Докажите, что в арифметической прогрессии с четным числом членов сумма членов второй половины превышает сумму членов первой на число, кратное квадрату половины числа членов. Г* Г* £* £* 4. Расположите числа sin -, cos -, tg -, ctg - в порядке о о о о возрастания. Обоснуйте ответ. 5. Найдите все пары целых чисел (х, у)> для которых 2л:2 + у2 = 2ху + 4л:. Вариант 8 1. Постройте график функции у = (cos л:)0 \А-зт2л:. 2. Решите уравнение ^125-^2^ = 51,2. 3. Около круга описан прямоугольный треугольник с острым углом 30° и противолежащим катетом длиной 4 см. Найдите площадь круга. 4. Докажите, что если величины углов выпуклого пятиугольника составляют арифметическую прогрессию, то каждая из них больше 36°. 5. Докажите, что для графика у = х4 + ахг + Ьл:2 + сх + d точка (0; 0) не является центром симметрии. Вариант 9 1. Решите уравнение f1/!^ + (\/Ъ) = ( 84
2. Один из углов треугольника равен 30°. Докажите, что радиус описанной около этого треугольника окружности меньше половины его периметра. 3. Два положительных числа а и Ь удовлетворяют неравенству а < 1. Докажите, что одно из этих чисел больше 1, а а + о второе меньше 1. 4. Докажите, что при любом натуральном п выражение /I3 + 23я делится нацело на 6. 5. Предприятие предполагает использовать 2000000 рублей на путевки в дома отдыха для своих сотрудников. Имеются путевки на 15, 27, 45 дней стоимостью соответственно 24000, 40000, 60000 р. Сколько и каких путевок надо купить, чтобы общее число дней отдыха было наибольшим? Вариант 10 1. Решите уравнение х-х2-2хг = ^. о 2. Пусть а + Ь + с<0и уравнение ах2 + Ьх + с = 0 не имеет корней. Какой знак имеет число с? 3. Дана система уравнений {cos a + cos /3=1, sin a + sin/3 = а. Докажите, что а ^ л/3. 4. На продолжении СЕ стороны АС равностороннего треугольника ABC за вершину С построен произвольный равносторонний треугольник CDE, Доказать, что треугольник СМР, гдеМиР — середины отрезковAD и BE соответственно, — тоже равносторонний. 5. Перед Бабой Ягой и Кащеем Бессмертным лежат две кучи мухоморов, в одной 100 штук, а в другой 150 штук. Эти персонажи по очереди берут грибы из куч, за один раз можно взять любое ненулевое число грибов из одной из куч. Пропускать ход нельзя, выигрывает тот, после хода которого, грибов не останется. Первой ходит Баба Яга. Кто из игроков выиграет при правильной игре? 85
11 класс Вариант 1 1. Постройте график функции: г/ = л/4 sin4 х-2 cos 2х + 3 + л/4 cos4 # + 2 cos 2jc + 3. (3 6.) 2. Определите а так, чтобы сумма квадратов корней уравнения х2 + (2-а)х-а-3 = 0 была наименьшей. (5 б.) 3. ABCDA\BiC\D\ — куб с ребром 2 см. Паук находится в центре грани АВА\В\. Какую наименьшую длину может иметь путь паука по поверхности куба в вершину С? (5 б.) 4. Докажите, что 2а + -^ > 3 при 0 < а < 1. (7 б.) а2 5. Непараллельные стороны трапеции продолжены до взаимного пересечения и через полученную точку проведена прямая, параллельная основаниям трапеции. Найти отрезок ее, ограниченный продолжениями диагоналей, если основания равны аиЬ. (10 6.) Вариант 2 1. Решите неравенство: 2. Найдите наименьшее и наибольшее значения функции: f(x) = хг-х2-х + 2 на отрезке [-1; |1. 3. Постройте график функции у = y/tgx • y/ctgx. 4. Существует ли в пространстве фигура, состоящая из многоугольников и содержащая точки А, В, С, D, для которой выполняются следующие соотношения: АВ = CD = 8 см; АС = = BD = 10 см; АВ + ВС = 13 см? 5. На доске написано несколько плюсов и минусов. Разрешается стереть любые два знака и написать вместо них плюс, если они одинаковы, и минус в противном случае. Докажите, что последний оставшийся на доске знак не зависит от порядка, в котором производились стирания. 86
Вариант 3 1. Решите уравнение: sin 9* + | sin (Зх + |) = ^ sin(3* + тг). 2. Вычислите площадь фигуры, ограниченной линиями: г/=±*2 + * и z/ 3. Решите систему уравнений: 4. В равнобедренной трапеции даны основания а = 21 см, = 9 см и высота Л = 8 см. Найдите радиус описанного круга. 5. Найдите все решения уравнения: х2 + Ъу2 + 4*1/ + 2у + 1 = 0. Вариант 4 1. Решите уравнение: sin ( ^тг • costt*) = тт. in ( ^тг 2. Покажите, что куб можно пересечь плоскостью так, чтобы в сечении получился правильный шестиугольник. 3. Дана функция у = рЧ. Постройте график данной функ- \х\ ции. 4. Представьте числа от 1 до 10 с помощью числа тг, используя скобки, знаки сложения, вычитания, умножения, деления, извлечения квадратного корня, а также символ функции [х]9 ([х] — целая часть числа х). Например: 11 = [(тг • тг) + у/к]. 5. Найти сумму 6 + 66 + 666 + ... + 666...6. (Последнее слагаемое содержит п шестерок.) 87
Вариант 5 1. Дана функция У N Найдите: а) нули функции; б) промежутки монотонности; в) экстремумы. Постройте схематически график функции. 2. Докажите неравенство log2 3 > \/2 без использования таблиц и микрокалькулятора. 3. Найдите максимум ab> если а + 2Ь = 1. 4. Решите уравнение: |#—1|—1#—2| = 1. 5. Найдите все действительные решения уравнения: х2 + 2х sin(xy) + 1 = 0. Вариант 6 1. Решите уравнение: х/яТТ + \/х—\ = \/2. 2. В четырехугольнике ABCD диагонали пересекаются в точке М. Известно, что AM = 1, ВМ = 2, СМ = 4. При каких значениях DM четырехугольник ABCD является трапецией? 3. Найдите экстремумы функции у = -Зх-2е~х. 4. Докажите, что число (l + | + | + ... + оТщ) •2-3-...-2004 а) целое; б) делится на 2005. 5. Вася из бумаги склеил многогранник, затем разрезал его по ребрам на отдельные грани, сложил в конверт и послал Саше. Верно ли, что Саша склеит такой же многогранник, какой был у Васи? 88
Вариант 7 1. Докажите, что функции ( 1 I sin-, xfOy * = 0 VO, * = 0 ~* ' [0, не имеют производных в точке х = 0. х = 0 2. Решите уравнение л/1 + л: + х2 + \/1-л; + л;2 = 2. з 3. Вычислите -з 4. Найдите все пары целых чисел (jc, у), для которых 6 f [\\x\-2\dx. 2х2 + у2 = 2ху + Зу. 5. Сравните 20102010 и 20112009. Вариант 8 1. Вычислите площадь фигуры, изображенной на рис. 29, если у = -х2. Рис. 25 2. Решите систему неравенств: х2 < 16*, 4>*2. 89
jc2-4 1 3. Решите уравнение: 2 \х\ In x + г-: = —=Ц—. 1*1 + 2 cosf 4. В усеченный конус вписан шар. Сумма диаметров верхнего и нижнего оснований конуса в 5 раз больше радиуса шара. Найдите угол между образующей усеченного конуса и плоскостью основания. 5. Найдите все трехзначные числа, кратные 13, сумма цифр которых также кратна 13. Вариант 9 1. Постройте график уравнения: |л:| + \у\ = 1. 1 2. Вычислите о 3. На координатной плоскости задан четырехугольник с вершинами в точках (0;6), (8; 12), (11; 8) и (3;2). Вычислите площади фигур, на которые разбивает его прямая, заданная уравнением х + 7у—67 = 0. 4. Решите уравнение: (х2 + х)2 + \х2 + х\ -2 = 0. 5. В кубе со стороной 1 м находятся 2009 тараканов. Докажите, что хотя бы трех из них можно поймать сферой радиу! са са Вариант 10 1. Найдите ошибку в вычислении: 7Г 7Г ftg2 J tg2xdx= [ (—\ l)dx = (tgx-x) J z = —7Г. 0 о о 2. Сколько действительных корней имеет уравнение х5 + х3 + 1 = О? 3. Решите систему уравнений: + У _ х + 2 = У + г 2 3 4 ' (х + у)2 + (х + г)2 + (у + г)2 = 29. 90
4. Окружность, радиус которой равен 1 см, а центр расположен на оси Оу> касается параболы, заданной уравнением у = х2. Найдите координаты точки касания и центра окружности. 5. Решите в натуральных числах уравнение: 1+х + х2 + х3 = 2у.
Раздел пятый ОСОБЕННОСТИ ПРОВЕДЕНИЯ МАТЕМАТИЧЕСКИХ ОЛИМПИАД В СЕЛЬСКИХ ШКОЛАХ С МАЛОЙ НАПОЛНЯЕМОСТЬЮ КЛАССОВ Большинство сельских школ сегодня имеют по одному классу в параллели, при этом часто в этом классе учится всего несколько учеников. При таком числе учащихся в классах учителя математики часто не проводят классных олимпиад. Причины этого могут быть разные: учителя и без проведения олимпиады знают, кого можно направить на городскую (районную) олимпиаду; мало желающих участвовать в олимпиаде и др. В результате учащиеся или совсем не участвуют в районных олимпиадах или приезжают туда без опыта участия в них. А поэтому часто показывают низкие результаты. Одним из возможных подходов решения данной проблемы является изменение методики проведения олимпиад в части требований к текстам олимпиад; в требованиях к оценке выполненных заданий и определении победителей. Рассмотрим данные изменения подробнее: 1. Текст школьной олимпиады должен составляться для учащихся нескольких классов, например, 5-6 классов, 7- 9 классов, 10-11 классов. 2. Задания, включаемые в текст, должны состоять из двух блоков. В первый блок включаются задания, учитывающие класс, в котором учатся участники. Они должны быть, так как ученики учатся в разных классах, поэтому отличаются друг от друга по уровню обученности. Включение заданий, аналогичным рассмотренным на уроке объясняется и тем, что участники олимпиад должны решить верно хотя бы по одной из предложенных задач. Во второй блок включаются олимпиадные задачи, предназначенные, в первую очередь, для проверки уровня разви- 92
тия мышления учащихся. Они являются одинаковыми для нескольких классов. 3. Задания лучше все оценивать, исходя из максимальной оценки за одно задание: 5 б или 7 б. Для выставления числа баллов за каждую задачу можно использовать следующую таблицу: Максимальное число баллов Безупречное решение Решение с недочетами Неполное решение с негрубыми ошибками Неверное решение, но есть продвижение в верном направлении 5 5 4 3 1-2 7 7 6 4-5 1-3 4. Победителей и призеров определять по набранной сумме баллов также для 2 (или 3) классов. 5. В члены жюри наряду с учителями математики можно включить учителей физики, химии, начальных классов, а также и старшеклассников (для проведения олимпиад в младших классах). К оценке работ учащихся старшего возраста можно подойти строже. В качестве примера приведем несколько возможных вариантов текстов школьных олимпиад. Примерные тексты школьных олимпиад для учащихся 5—6 классов Вариант 1 Вариативная часть 5 класс 1. Запишите наибольшее и наименьшее семизначное число. 2. Вычислите: 101101 • 999-101 • 999999. 6 класс 1. Найдите все дроби со знаменателем 15, которые больше о 5 и меньше 1. 93
2. Решите уравнение: -^ : 3,1 = х : 9,3. у Инвариантная часть 3. Масса бидона с молоком 32 кг, без молока — 2 кг. Какова масса бидона, заполненного молоком наполовину? 4. Вместо звездочек расставьте пропущенные цифры: Х785 * * * *** 1** * * * * ***** 5. Три подруги вышли в белом, синем, зеленом платьях и туфлях таких же цветов. Известно, что только у Ани цвет платья и туфель совпадает. Ни платье, ни туфли Вали не были белыми. Наташа была в зеленых туфлях. Определите цвет платья и туфель каждой подруги. Вариант 2 Вариативная часть 5 класс 1. Найдите среди чисел вида За + 1 первые три числа, которые кратны 5. 2. Для нумерации страниц книги потребовалось всего 1392 цифры. Сколько страниц в этой книге? 6 класс 1. Сколько воды надо добавить к 600 г жидкости, содержащей 40% соли, чтобы получился 12-процентный раствор этой соли? 2. Попрыгунья Стрекоза половину времени каждых суток красного лета спала, третью часть времени каждых суток танцевала, шестую часть — пела. Остальное время она решила посвятить подготовке к зиме. Сколько часов в сутки Стрекоза готовилась к зиме? 94
Инвариантная часть 3. Расшифруйте запись. Одинаковыми буквами обозначены одинаковые цифры, разными буквами — разные цифры. «КОКА КОЛА ВОДА 4. В летний лагерь приехали отдыхать три друга: Миша, Володя и Петя. Известно, что каждый из них имеет одну из следующих фамилий: Иванов, Семенов, Герасимов. Миша — не Герасимов. Отец Володи — инженер. Володя учится в 6 классе. Герасимов учится в 5 классе. Отец Иванова — учитель. Какая фамилия у каждого из трех друзей. 5. В школе 33 класса, 1150 учеников. Найдется ли класс, в котором меньше 35 учеников? Примерный текст школьной олимпиады для учащихся 7—9 классов Вариативная часть 7 класс 1. Среди перечисленных выражений указать такие, которые: а) тождественно равны а2: (-а)2, -(-а)2, -а2; б) тождественно равны а3: (-а)3, -(-а)3, -а3. 2. Двоим друзьям, потребовалось вычислить 42- З2. Они заметили, что результат: число 7 равен сумме оснований квадратов чисел 4 и 3. Проверив свое открытие на числах 10 и 11, друзья установили, что оно подтверждается и в случае II2—102 = 21 = 11 + 10. После этого друзья нашли все пары (а,Ь) натуральных чисел а > Ь, для которых разность а2-Ь2 равна сумме а + Ь. Как друзьям удалось найти все такие числа (а,Ь)? 3. Как разрезать квадрат 5x5 прямыми линиями так, чтобы из полученных частей можно было составить 50 равных квадратов? Не разрешается оставлять неиспользованные части, а также накладывать их друг на друга. 95
8 класс i Л7 ( 6 1 \ J/2 + 6г/ + 9 1. Упростите выражение: —^— + ^— -г- . V-9 3-yJ 5 2. Докажите, что значение выражения —-— + 1-Зл/З 1 + 3\/3 есть число рациональное. 3. Докажите, что биссектрисы внешних углов прямоугольника, пересекаясь, образуют квадрат. 9 класс 1. Сократите дробь: ** +25*2-4*"20. 2. Равнобокая трапеция ABCD разбивается диагональю АС на 2 равнобедренных треугольника. Определите углы трапеции. 3. Сколько цифр содержит число 45 • 513? Инвариантная часть 4. Сколькими нулями оканчивается произведение всех целых чисел от 1 до 100 включительно? 5. Решить уравнение в целых числах: х + у = ху. 6. 5 школьников приехали из 5 различных городов в Архангельск на областную математическую олимпиаду. «Откуда вы, ребята?» — спросили их хозяева. Вот что ответил каждый из них: Андреев: «Я приехал из Онеги, а Григорьев живет в Каргополе». Борисов: «В Каргополе живет Васильев. Я же прибыл из Коряжмы». Васильев: «Я прибыл из Онеги, а Борисов — из Котласа». Григорьев: «Я прибыл из Каргополя, а Данилов из Вельска». Данилов: «Да, я действительно из Вельска, Андреев же живет в Коряжме». Хозяева очень удивились противоречивости ответов приехавших гостей. Ребята объяснили им, что каждый из них высказал одно утверждение правильное, а другое ложное. Но по 96
их ответам вполне можно установить, кто откуда приехал. Откуда приехал каждый школьник? Используя выше приведенные тексты из раздела 4, учитель может сам по аналогии составить тексты для двух или трех классов. Для учащихся 5-6 классов можно сделать и единые тексты школьных олимпиад, больше учитывая содержание материала, изученного в начальной школе и начале 5 класса, а также включая задачи на материал, не изучаемый на уроках, но рассматриваемый во внеклассной работе. Например: Вариант 1 1. Выразите х из формулы: а = (х + 8): 9. 2. Найдите значение выражения: 26-25-25-24 + 24-23-23-22 + 22-21-21-20 + 20-19-19 18+ + 18- 17-17-16 + 16- 15-15- 14. 3. Расшифруйте два ребуса, в которых одинаковым буквам соответствуют одинаковые цифры, а разным буквам — разные цифры в обоих примерах. ,АБВ VABB ВВ ВВ ААБ . АБВ АБВ АГАВ 4. Парусник отправляется в плавание в понедельник в полдень. Плавание будет продолжаться 100 часов. Назовите день и час его возвращения в порт. 5. Из числа 12345678910111213...5657585960вычеркните 100 цифр так, чтобы оставшееся число стало наибольшим. 6. В бутылке, стакане, кувшине и банке находятся молоко, лимонад, квас и вода. Известно, что вода и молоко не в бутылке, сосуд с лимонадом стоит между кувшином и сосудом с квасом, в банке не лимонад и не вода. Стакан стоит около банки и сосуда с молоком. В какой сосуд налита каждая из жидкостей? 97
Вариант 2 1. Догадайся, какие цифры надо поставить вместо звездочек: 4* + 3** *2** 1 * * * * 2. У Ивана имеется деревянный параллелепипед с измерениями б см, 12 см, 18 см. Он распиливает его на кубики с ребром 1 см и ставит их один на другой. Сможет ли Иван достроить вышку из этих кубиков, если даже он заберется на трехметровую лестницу? 3. Можно ли треугольник разрезать так, чтобы получилось три четырехугольника? (Если «да», то выполните рисунок). 4. Школьный драмкружок, готовясь к постановке отрывка из сказки А. С. Пушкина о царе Салтане, решил распределить роли между участниками: — Я буду Черномором, — сказал Юра. — Нет, Черномором буду я, — заявил Коля. — Ладно, — уступил ему Юра, — я могу сыграть Гвидона. — Ну, я могу стать Салтаном, — тоже проявил уступчивость Коля. — Я же согласен быть только Гвидоном! — произнес Миша. Желания мальчиков были удовлетворены. Как распределились роли? 5. В трех мешках находятся крупа, вермишель и сахар. На одном мешке написано «крупа», на другом— «вермишель», на третьем — «крупа или сахар». В каком мешке что находится, если содержимое каждого из них не соответствует записи? 6. Мачеха, уезжая на бал, дала Золушке мешок, в котором были перемешаны мак и просо, и велела перебрать их. Когда Золушка уезжала на бал, она оставила три мешка: в одном — просо, в другом — мак, а в третьем — еще не разобранная смесь. Чтобы не перепутать мешки, Золушка к каждому из них приклеила таблички: «Мак», «Просо», «Смесь». Мачеха вернулась с бала первой и нарочно поменяла местами таблички так, чтобы на каждом мешке оказалась неправильная запись. 98
Ученик Феи успел предупредить Золушку, что теперь ни одна надпись на мешках не соответствует действительности. Тогда Золушка достала только одно-единственное зернышко из одного мешка и, посмотрев на него, сразу догадалась, где что лежит. Как она это сделала? Решения всех предложенных заданий в текстах данных олимпиад можно найти в приведенных ответах к задачам раздела 4.
Раздел шестой ЗАДАЧИ ДЛЯ ПОДГОТОВКИ К МАТЕМАТИЧЕСКОЙ ОЛИМПИАДЕ В этом разделе содержатся задачи, которые можно предложить учащимся для подготовки к олимпиаде в школе, городе (районе). Часть данных задач может быть применена в качестве заданий школьных олимпиад. Некоторые из задач 5 класса могут быть использованы в 6, из 6 класса — в 7 и т. д. В случае, если некоторый материал окажется неизученным на момент решения предлагаемых задач, их можно решить и позже. Задачи по классам распределены в соответствии с действующими на сегодня наиболее распространенными в общеобразовательных учреждениях учебниками математики, геометрии, алгебры и начал анализа. Особенностью данной подборки является то, что задачи не сгруппированы по разделам, темам. Этой цели служат следующие пособия автора, в частности «Математические кружки в школе. 5-8 классы», «Учимся решать олимпиадные задачи. Геометрия. 5-11 классы» и др. Многие из предлагаемых задач заимствованы из различных сборников (большинство из них указано в списке литературы), часть задач переработана, а некоторые из задач составлены автором. 5 класс 5.1. Напишите число 100: а) шестью одинаковыми цифрами; б) девятью различными цифрами. (Можно использовать знаки: +, :.) 5.2. Ваня задумал число, прибавил к нему 5, потом разделил сумму на 9, умножил на 4, отнял 6, разделил на 7 и получил 2. Какое число задумал Ваня? 100
5.3. Говорил дед внукам: «Вот вам 130 орехов, разделите их на 2 части так, чтобы меньшая часть, увеличенная в 4 раза, равнялась бы большей части, уменьшенной в 3 раза». Как разделить орехи? 5.4. Запишите подряд 20 пятерок: 55...5. Поставьте между некоторыми цифрами знак сложения так, чтобы сумма равнялась 1000. 5.5. Крестьянин попросил взять у царя 1 яблоко из его сада. Царь разрешил. Пошел крестьянин к саду и видит: весь сад огорожен тройным забором, причем каждый забор имеет одни ворота, вход в которые охраняет сторож. Подошел крестьянин к первому сторожу и говорит: «Царь разрешил мне взять одно яблоко из сада». На что сторож ему сказал: «Возьми, но при выходе отдашь мне половину тех яблок, что возьмешь, и еще одно». Эти же слова повторили крестьянину 2 и 3 сторожа, охранявшие другие ворота. Сколько яблок должен взять крестьянин, чтобы после того, как он отдаст положенную часть 3 сторожам, у него осталось одно яблоко? 5.6. Запишите число, состоящее из суммы 11 тысяч, 11 сотен и 11 единиц. 5.7. Объясните, как с помощью спичек можно получить на столе угол в 60°, 90°, 120°. Изобразите получившиеся фигуры с этим углом. 5.8. Сколько существует двузначных чисел, у которых цифра десятков больше цифры единиц? 5.9. Школьники Петя и Вася взвесили свои портфели на весах. Весы показали 3 кг и 2 кг. Когда же они поставили на весы оба портфеля, весы показали 6 кг. «Как же так? — удивился Петя. — Ведь 2 + 3 не равняется 6». На что Вася ответил: «Разве ты не видишь, что у весов сдвинута стрелка?» Сколько же весят портфели на самом деле? 5.10. На сколько сумма всех четных чисел первой сотни больше суммы всех нечетных чисел этой сотни? 5.11. Имеются двое песочных часов: на 3 минуты и на 7 минут. Яйцо варится 11 минут. Как отмерить это время при помощи имеющихся часов? 101
5.12. В ящике лежат шары: 5 красных, 7 синих и 1 зеленый. Сколько шаров надо вынуть, чтобы достать 2 шара одного цвета? 5.13. Какое наименьшее число разрезов нужно сделать, чтобы разрезать куб с ребром 3 см на 27 единичных кубиков? 5.14. Николай с сыном и Иван с сыном были на рыбалке. Николай поймал столько же рыб, сколько и его сын, а Иван — втрое больше, чем его сын. Всего было поймано 35 рыб. Сколько рыб поймал Иван и как звали его сына? 5.15. Автору данной книги в 1989 году исполнилось столько лет, сколько сумма цифр года его рождения. В каком году он родился? 5.16. Петя и Ваня ехали вниз по эскалатору. На середине эскалатора хулиган Витя сорвал с Пети шапку и бросил ее на встречный эскалатор. Пострадавший Петя побежал обратно вверх по эскалатору, чтобы затем спуститься вниз и вернуть шапку. Хитрый Ваня побежал по эскалатору вниз, чтобы затем подняться вверх и успеть раньше Пети. Кто успел первый, если скорости ребят относительно эскалатора постоянны и не зависят от направления движения? 5.17. Король нанял 2 рабочих для рытья подземного хода из своего дворца. Один рабочий за 1 час может прокопать вдвое больше, чем другой, а платит им король каждому одинаково за каждый час работы. Что обойдется дешевле — совместная работа землекопов с 2 сторон до встречи или поочередное рытье половины подземного хода каждым из землекопов. 5.18. Продавец получил для продажи несколько пачек конвертов по 100 конвертов в каждой. 10 конвертов он отсчитывает за 10 с. За сколько секунд сообразительный продавец может отсчитать 70 конвертов? 90 конвертов? 5.19. Найдите ошибку в следующих рассуждениях. Имеем числовое равенство: 4:4 = 5:5. Вынесем за скобки в каждой части общий множитель. Получим: 4(1 : 1) = 5(1 : 1). Числа в скобках равны, поэтому 4 = 5, или 2x2 = 5. 5.20. Петя сказал друзьям: «Позавчера мне было 10 лет, а в будущем году исполнится 13». Мог ли Петя не соврать? 5.21. В ряд выписано 12 девяток: 99999999999 9. Поставьте между ними знак: +,-,:, х, чтобы получилось 2000. 102
5.22. Трое учеников пошли на рыбалку, взяв с собой лодку, выдерживающую нагрузку до 100 кг. Как перебраться ученикам с берега реки на остров, если их массы равны 40 кг, 50 кг, 70 кг? 5.23. У поросят Ниф-Нифа и Нуф-Нуфа было соответственно по 4 и 8 пирогов. К ним пришел Наф-Наф и попросил угостить его пирожками. Пироги были разделены поровну. После того, как все пироги были съедены, Наф-Наф поблагодарил поросят и дал им 6 рублей. Как разделить эти деньги между Ниф-Нифом и Нуф-Нуфом справедливо? 5.24. Средний возраст 11 игроков футбольной команды 22 года. Во время матча один из игроков за грубое нарушение правил был удален с поля до конца игры. Средний возраст оставшихся на поле игроков стал равен 21. Сколько лет футболисту, удаленному с поля? 5.25. Расшифруйте пример, если одинаковым буквам соответствуют одинаковые цифры, а разным буквам — разные: ССС 5.26. Митя, Толя, Сеня, Юра и Костя пришли в музей и встали в очередь. Если бы Митя встал посередине очереди, то он оказался бы между Сеней и Костей, а если бы Митя встал в конце очереди, то рядом с ним мог быть Юра, но Митя встал впереди всех своих товарищей. Кто за кем стоит? 5.27. На школьной олимпиаде по математике участникам было предложено решить 6 задач. За каждую решенную задачу засчитывалось по 7 очков, а за каждую нерешенную списывалось 3 очка. Сколько задач решил участник, если он набрал 12 очков? 2 очка? 32 очка? 5.28. Сколько существует натуральных чисел, меньших 100, в записи которых цифра 5 использована хотя бы 1 раз? 5.29. В семье 3 детей: 2 мальчика и девочка. Их имена начинаются с букв А, В, Г. Среди А и В есть начальная буква имени одного мальчика, а среди В и Г — начальная буква имени другого мальчика. С какой буквы начинается имя девочки? юз
5.30. Четыре человека обменялись рукопожатиями. Сколько всего было рукопожатий? 5.31. Четыре брата Юра, Петя, Вова, Коля учатся в 1, 2, 3, 4 классах. Петя — отличник, младшие братья стараются брать с него пример. Вова учится в 4 классе. Юра помогает решать задачи брату. Кто из них в каком классе учится? 5.32. Винни-Пуху подарили на день рождения бочонок с медом массой 7 кг. Когда Винни-Пух съел половину меда, то бочонок с оставшимся медом стал иметь массу 4 кг. Сколько килограммов меда было первоначально в бочонке? 5.33. Три одинаковых арбуза надо разделить поровну между четырьмя детьми. Как это сделать, выполнив наименьшее число разрезаний? 5.34. Дочери в настоящее время 8 лет, а матери 38 лет. Через сколько лет мать будет втрое старше дочери? 5.35. Илья Муромец, Добрыня Никитич и Алеша Попович вступили в бой с несколькими великанами. Получив по 3 удара богатырскими палицами, великаны обратились в бегство. Больше всего ударов нанес Илья Муромец: 7, меньше всего — Алеша Попович: 3. Сколько всего было великанов? 5.36. Возраст дедушки выражается наименьшим трехзначным числом, которое записывается различными цифрами. Сколько лет дедушке? 5.37. Три поросенка построили три домика из соломы, из прутьев и из камней. Каждый из них получил один домик: Ниф-Ниф — не из камней и не из прутьев; Нуф-Нуф — не из камней. Объясните, какой домик достался Наф-Нафу? 5.38. Дядька Черномор и 33 богатыря охраняют остров Буян. Наряд из 6 богатырей дежурит в течение суток. Каким образом дядька Черномор может организовать дежурство в течение 11 суток так, чтобы каждый богатырь отдежурил 2 суток? 5.39. В двузначном числе первую цифру 4зачеркнули. Получилось число в 9 раз меньше первоначального. Каким может быть двузначное число? 5.40. Гриша, Люда, Зина, Петя родились 12 февраля, 6 апреля, 12 июня, 26 июня. Интересно, что Петя и Люда родились в одном месяце, а Зина и Петя родились в один и тот же день разных месяцев. Когда родился Гриша? 104
5.41. Победителей олимпиады выстроили в ряд на сцене. Директор школы, поздравляя их, заметил, что пятым справа стоял Коля, выступивший лучше всех. Учитель математики же обратил внимание на то, что Коля стоял девятым слева. Сколько всего учеников стояло на сцене? 5.42. В феврале 2000 года 2 февраля было средой. Сколько вторников было в феврале 2000 года? 5.43. Мальвина дала Буратино лист бумаги, на котором были нарисованы квадрат и треугольник. Буратино поставил внутри квадрата 3 точки, а внутри треугольника — 2. Всего получилось 4 точки, причем ни одна из них не располагалась на сторонах квадрата или треугольника. Покажите, как были нарисованы квадрат и треугольник и как Буратино поставил точки. 5.44. Закрасьте 8 одинаковых клеток так, чтобы каждая из них имела по две соседние закрашенные клетки. 5.45. У всех 25 учеников на родительское собрание пришли папы и мамы. Мам было 20, а пап — 10. У скольких учеников на родительское собрание пришли и папы, и мамы? 5.46. Сколько всего квадратов на рис. 30? 5.47. Малыш и Карлсон сидели на крыше и наблюдали за голубями. На крыше сидело несколько голубей. Когда на крышу село еще 15 голубей, а улетело 18 голубей, то на крыше осталось 16 голубей. Сколько голубей первоначально наблюдали Малыш и Карлсон? 5.48. Космические корабли-спутники летают на высоте до 300 км. Хватит ли кубиков объемом в 1 куб. мм, содержащихся в 1 куб. м, чтобы сложить из них башню такой высоты? 5.49. Разделите изображенную на рис. 31 фигуру на четыре равные части. 5.50. Запишите число 1 000 000, имея только цифру 3, знаки арифметических действий и скобки, если они необходимы. Можно ли в этой записи обойтись без действия деления? 5.51. Решите уравнение: % — % + ^ = -£г. 5.52. Какой фигурой может быть пересечение треугольника и четырехугольника? Покажите на рисунках. 105
Рис. 30 Рис. 31 5.53. Почтальон Печкин разнес почту во все дома деревни, после чего зашел к дяде Федору выпить молока. На рис. 32 показаны все тропинки, которые проходил Печкин, причем, как оказалось, ни по одной из них он не проходил дважды. Каков мог быть маршрут почтальона Печкина? В каком доме живет дядя Федор? Рис. 32 5.54. Найдите площадь прямоугольника, если его длина на 5 см больше ширины, а половина периметра равна 19 см. 5.55. Начертите три тупых угла так, чтобы два из них не имели общих точек, а стороны третьего пересекали бы лишь одну сторону каждого из первых двух углов. 5.56. Во сколько раз лестница с первого этажа на шестнадцатый длиннее лестницы с первого на четвертый этаж дома? 106
5.57. Восстановите запись: *** Х *8 + *:* ***5 *** * О 5.58. На координатном луче отмечено несколько точек, координаты которых являются натуральными числами. Известно также, что сумма этих чисел равна 75. Если мы каждую точку переместим вправо на три единичных отрезка, то сумма координат новых точек будет уже равняться 99. Сколько чисел было отмечено на координатном луче? 5.59. Сколько всего имеется пятизначных чисел, сумма цифр в которых равняется трем? Причем в записи каждого числа цифра 1 может встречаться не более одного раза. 5.60. Разрежьте прямоугольник 3 х 4 на 2 равные части. Найдите как можно больше способов. Разрезать можно лишь по стороне квадрата 1 х 1 и способы считаются разными, если полученные фигуры не будут равными при каждом способе. 5.61. Поставьте в выражении скобки так, чтобы получилось верное равенство: 270 4-120 4- 390 : 3 • 5 = 1120. 5.62. Из трех одинаковых кубов с ребром 8 см составили прямоугольный параллелепипед. Найдите его объем и площадь поверхности. 5.63. Найдите все натуральные значения х> при которых верно неравенство: 5.64. Малыш проехал на самокате некоторое расстояние за 15 минут. За какое время он проедет на велосипеде расстояние в 3 раза большее? Скорость малыша на велосипеде в 5 раз больше, чем на самокате. 5.65. Из скольких кубиков, поставленных один на другой, может состоять башня, показанная на рис. 33? 5.66. В кабинет математики для консультации собрались трое учеников: Аня, Боря и Света. Для ответа на вопросы Ане 107
Вид спереди Вид сбоку Рис. 33 требуется 5 минут, Боре — 2 минуты, а Свете — 7 минут. Как учителю правильнее построить проведение консультации, чтобы ученики находились в кабинете как можно меньше времени? 5.67. Катя и Юра купили лотерейные билеты с номерами: 625517 и 322324, и обнаружили, что в каждом из номеров можно расставить знаки арифметических действий и скобки так, что в каждом случае результат будет равняться 100. Как это можно сделать? 5.68. Сколько всего пятизначных чисел можно составить из цифр 0 и 1? Цифры могут повторяться. Запишите эти числа. 5.69. Как с помощью двух бидонов 5 л и 8 л отлить из молочной цистерны 7 л молока? Молоко разрешается выливать обратно в цистерну. 5.70. Старший брат идет от школы до дома 30 минут, а младший — 40 минут. Через какое время старший брат догонит младшего, если тот вышел из школы на 5 минут раньше? 5.71. Сколько сантиметров проволоки потребуется для изготовления каркаса куба с ребром 6 см? 5.72. На сковородке помещается 2 кусочка хлеба. На поджаривание куска с одной стороны требуется 1 минута. Как за три минуты поджарить 3 куска с обеих сторон? 5.73. Деревянный окрашенный куб с ребром 4 см распилили на кубические сантиметры. Сколько среди них оказалось кубиков, окрашенных с трех сторон? 5.74. Муравей направился в гости в соседний муравейник. Туда он шел пешком, а обратно ехал: первую половину пути на гусенице — со скоростью в 2 раза меньше, чем пешком; а 108
другую половину — на кузнечике, со скоростью, в 5 раз большей, чем пешком. На какой путь муравей затратил времени меньше: в гости или обратно и почему? 5.75. Сергей приехал в гости к своей сестре Кате. Гуляя по городу, они остановились перед больницей. «Я навещу своего племянника», — сказал Сергей. «Хорошо, — сказала Катя. — Тут у меня нет больного племянника, да я его уже проведывала сегодня. Пойду, зайду в магазин». Каковы родственные отношения Кати и больного племянника? 5.76. Запишите число 100 с помощью: а) пяти единиц и знаков действий; б) пяти пятерок и знаков действий; в) пяти троек и знаков действий. 5.77. Ваня шепнул на уроке Марине: «Ижаксдоп тевто!». На каком «языке» говорил Коля? (Оказывается, торговцы вразнос на Руси также знали этот «язык».) 6 класс 6.1. Лошадь может съесть воз сена за 1 месяц, коза — за 2 месяца, а овца — за 3 месяца. За какое время лошадь, коза и овца вместе съедят такой же воз сена? 6.2. Найдите дробь со знаменателем 19, которая больше -> но меньше -. 6.3. Сколькими нулями оканчивается произведение: 1 • 2 • 3 • ... 2001 • 2002 • 2003? 6.4. 12 человек несут 12 буханок хлеба. Каждый мужчина несет по 2 буханки хлеба, женщина — по ^ буханки, а ребенок по 2« Сколько было мужчин, женщин, детей? 6.5. На озере расцвела 1 лилия. Каждый день число цветков удваивалось, и на 10-й день все озеро покрылось цветами. На какой день покрылась цветами половина озера? 109
6.6. Как раз делить круг тремя прямыми на 4,5,6, 7 частей? 6.7. У Ивана-царевича был волшебный ковер-самолет размером 9 х 12 м2, Змей Горыныч подкрался и отрезал от ковра маленький коврик размером 1 х 8 м2. Иван-царевич очень расстроился, т. к. волшебный ковер-самолет мог летать, лишь имея прямоугольную форму. Поэтому он решил еще отрезать кусок 1 х 4 м2, чтобы получился прямоугольник 8 х 12 м2, но Василиса Премудрая предложила поступить по-другому. Она разрезала ковер на три части, из которых волшебными нитками сшила квадратный ковер-самолет размером 10 х 10м2. Как переделала ковер Василиса Премудрая? 6.8. Когда Гулливер попал в Лилипутию, то обнаружил, что там всё ровно в 12 раз короче, чем на его родине. Сможете ли вы сказать, сколько лилипутских спичечных коробков помещается в спичечной коробке Гулливера? 6.9. На мачте пиратского корабля развевается двухцветный прямоугольный флаг, состоящий из чередующихся черных и белых вертикальных полос одинаковой ширины. Общее число полос равно числу пленных, находившихся в данный момент на корабле. Сначала на корабле было 12 пленных, а на флаге 12 полос, затем 2 пленных сбежали. Как разрезать флаг на 2 части, а затем сшить их, чтобы площадь флага и ширина полос не изменилась, а число полос стало равно 10? 6.10. Имеются чашечные весы без гирь и 3 одинаковые по внешнему виду монеты. Одна из монет фальшивая, причем неизвестно, легче она настоящих монет или тяжелее (одинаковы монеты одного веса). Сколько надо сделать взвешиваний, чтобы определить фальшивую монету? Решите эту же задачу в случаях, когда имеется 4 монеты и 9 монет. 6.11. Плоскость раскрашена в 2 цвета. Докажите, что найдутся 2 точки на расстоянии 1 метр друг от друга, окрашенные в одинаковый цвет. 6.12. Имеется 5 закрытых чемоданов и 5 ключей к ним. При этом неизвестно, к какому чемодану подходит какой ключ. Какое наименьшее число попыток надо сделать, чтобы наверняка определить, какой ключ подходит к какому чемодану? но
6.13. Баба Яга в своей избушке на курьих ножках завела сказочных животных. Все они, кроме двух, — Говорящие Коты. Все, кроме двух, — Мудрые Совы; остальные — Усатые Тараканы. Сколько обитателей в избушке у Бабы Яги? 6.14. На волшебной яблоне выросли 3 банана и 4 апельсина. Если сорвать один из плодов — вырастет такой же; если одновременно 2 одинаковых плода — вырастет апельсин, а если одновременно сорвать 2 разных плода — вырастет банан. В каком порядке надо срывать плоды, чтобы на яблоне остался ровно один? Можно ли определить, какой это будет плод? Можно ли срывать плоды так, чтобы на яблоне ничего не осталось? 6.15. Ребята пришли с рыбалки с уловом. Все вместе они поймали 121 рыбку, причем количество рыбок у каждого оказалось одинаковым. Сколько ребят ходило на рыбалку? 6.16. Проехав половину всего пути, пассажир лег спать и спал до тех пор, пока не осталось проехать половину того пути, который он проспал. Какую часть всего пути пассажир проехал бодрствующим? 6.17. Карлсон очень любил сладкое. Налив себе в стакан сметаны, он добавил туда варенья из банки, но как только он перемешал сметану и варенье, то понял, что хочет есть одно варенье. Недолго думая, он перелил в банку столько варенья со сметаной, сколько взял из банки варенья. После перемешивания Карлсон задумался: чего же получилось больше: сметаны в банке с вареньем или варенья в стакане со сметаной? А как думаете вы? 6.18. Отличник Вася решил купить себе в магазине одну ручку за 1 руб 80 коп и 6 стержней. Продавец потребовал с Васи 5 руб, на что Вася ответил тому, что он ошибся. Прав ли Вася и почему? 6.19. В классе меньше 30 учеников. За контрольную работу по математике пятая часть учеников получила пятерки, четвертая часть — тройки, а половина — четверки. Остальные получили двойки. Сколько учеников было в классе? Сколько из них получили двойку? 6.20. Делится ли число 11 • 21 - 31 • 41 • ... • 91-1 на 10? Почему? in
6.21. Витя попросил одноклассниц порешать для него задачи. Для того чтобы задачи быстрее решались, он сказал девочкам, что за каждую решенную задачу девочка, решившая ее первой, получает 3 конфеты, решившая второй — две, решившая последней — одну. После решения всех задач Витя обнаружил, что у всех девочек на столе по 11 конфет. Девочки сказали Вите, что они брали согласно уговору. Каждая девочка решила все задачи, и одновременно ни одной из задач они не решили. Правильно ли девочки брали конфеты и почему? 6.22. В 9 часов утра со станции А отправился пассажирский поезд, а вслед за ним в 11 ч с той же станции отправился скорый поезд. На каком расстоянии от станции А пассажирскому поезду надо будет пропустить скорый, если скорость пассажирского поезда 54 км/ч, а скорого — 72 км/ч? 6.23. Дима с собакой пошел встречать папу. Когда собака увидела папу, она побежала к нему со скоростью 5 м/с. Добежав до него, она сразу же побежала обратно к Диме. Добежав до Димы, собака снова побежала к папе и т. д. Какое расстояние пробежала собака, если Дима и папа двигались со скоростью 1,5 м/с, а первоначальное расстояние между ними было равно 300 м? 6.24. Три школьных товарища купили в буфете 14 пирожков. Коля купил в 2 раза меньше Вити, а Женя — больше Коли, но меньше Вити. Сколько пирожков купил каждый товарищ? 6.25. Незнайка подбросил кубик (см. рис. 34а) так, что он упал, как показано на рис. 346. Заполните пустые видимые грани куба. 6.26. Вычислите: 666 666-666 666 1+2+3+4+5+6+5+4+3+2+1 777 777-777 777 1+2+3+4+5+6+7+6+5+4+3+2+1* 6.27. Если треть числа разделить на его семнадцатую часть, в остатке будет 100. Найдите это число. 6.28. Сумма 2008 натуральных чисел — нечетное число. Каким числом, четным или нечетным, является произведение этих чисел? 112
ф А Рис. 34 6.29. В ковре 4x4 метра моль проела 15 дырок. Докажите, что из него можно вырезать коврик размером 1x1 метр, не содержащий внутри себя дырок. (Дырки считать точечными.) 6.30. На школьной олимпиаде по шахматам выступило 6 команд, в каждой команде было по 5 учеников. Сколько всего партий было сыграно на олимпиаде, если каждая команда играла с каждой по одной игре? 6.31. В парламенте одной из стран 150 депутатов. По крайней мере, один из них честен. В каждой паре депутатов хотя бы один продажен. Сколько всего честных депутатов в парламенте данной страны? 6.32. Вычислите: -90-89-88-...-1 + 0 + 1 + 2 + ... + 98 + 99 + 100. 6.33.3а первый день бригада скосила 15 га, а за второй день — 20% оставшейся площади. Всего за 2 дня было скошено 36% всех лугов. Найдите площадь всех лугов. 6.34. Из города Котлас в город Коряжма автомобиль ехал со скоростью 40 км/ч в течение 1 часа. Обратно автомобиль двигался уже со скоростью 60 км/ч. Найдите среднюю скорость автомобиля. из
6.35. Разделите семь яблок поровну на 12 человек, не разрезая яблоки более чем на 4 части. 6.36. В какой системе счисления справедливо равенство: 4-13 = 100? 6.37. Несколько одинаковых по численности бригад сторожей спали одинаковое число ночей. Каждый сторож проспал больше ночей, чем сторожей в бригаде, но меньше, чем число бригад. Сколько сторожей в бригаде, если все сторожа вместе проспали 1001 человеко-ночь? 6.38. После того как Маша съела половину яблок из банки, уровень компота понизился на одну треть. На какую часть (от полученного уровня) понизится уровень компота, если съесть половину оставшихся яблок? 6.39. Разрежьте изображенную на рис. 35 фигуру на две одинаковые части. Рис. 35 Рис. 36 6.40. Прямоугольник составлен из шести квадратов (см. рис. 36). Найдите сторону самого большого квадрата, если сторона самого маленького равна 1. 6.41. В поединке двух борцов разной силы всегда побеждает сильнейший. Можно ли разбить 9 борцов по трое на три команды так, чтобы во встречах команд по системе «каждый с каждым» по числу побед первая команда одержала верх над второй, вторая — над третьей, а третья — над первой? 114
7 класс 7.1. В классе 25 учеников, а сумма их возрастов составляет 270 лет. Найдутся ли в классе 20 учащихся, сумма возрастов которых больше 260? 7.2. Функция у = \х\ определяется так: . | J jc, если х ^ 0, [-#, еслил:<0. Как определить следующие функции: а) у = \х\ + х; б) у = \x\-x. Постройте их графики. 7.3. Изменится ли частное и остаток, если делимое и делитель увеличить в 3 раза? 7.4. Улитка ползает по столбу высотой 10 м. За день она поднимается на 5 м, а за ночь опускается на 4 м. За какое время улитка доберется от подножия до вершины столба? 7.5. Сколько раз в сутки часовая и минутная стрелки образуют прямой угол? 7.6. Переписывая с доски числовое выражение а5 • Ь2, ученик допустил ошибку и был очень удивлен, когда выяснилось, что записанное им число аЪЪ2 является значением заданного выражения. Какими могут быть цифры аиЬ? 7.7. На соревнованиях по фигурному катанию все 9 судей поставили за технику исполнения и артистизм спортсмена только оценки 5,6 и 5,7. Сумма всех оценок оказалась равной 101,9. Сколько оценок 5,7 получил спортсмен? 7.8. Для проведения водопровода в дом нужно 167 м труб. В наличии имеются трубы лишь длиной 5 м и 7 м. Сколько надо взять тех и других труб, чтобы сделать наименьшее число соединений? 7.9. Жители трех домов решили вырыть общий колодец так, чтобы от каждого дома до колодца было одинаковое расстояние. Покажите на рис. 37, где нужно расположить колодец? 7.10. Пришел Иван-царевич в подземелье к Кощею Бессмертному Василису Прекрасную освобождать. В подземелье 115
Рис. 37 три темницы. В одной из них томится Василиса, в другой расположился Змей Горыныч, а третья темница — пустая. На дверях есть надписи, но все они ложные. На первой темнице написано: «Здесь Василиса Прекрасная»; на второй темнице: «Темница № 3 не пустая»; на третьей темнице написано: «Здесь Змей Горыныч». В какой же темнице Василиса? 7.11. Докажите, что 13 +132 +133 +134 +... +132009 +132010 делится нацело на 7. 7.12. Расставьте в записи 4 • 12 + 18 : 6 + 3 скобки таким образом, чтобы получилось: а) число 50; б) возможное наименьшее число; в) возможное наибольшее число. 7.13. Решите числовые ребусы: а) х*76 б) Х2*9 * * * * ,18** ,*5* * * * * ** * * **920 ***06 7.14. 101 лошадь разместили в 15 конюшнях. Почему хотя бы в одной конюшне будет обязательно нечетное число лоша- дей? 7.15. Из куба 3x3x3 удалили центральный кубик и восемь угловых кубиков. Можно ли оставшуюся фигуру из 18 кубиков составить из 6 брусков 3x1x1? 7.16. За два года предприятие снизило объем выпускаемой продукции на 51%. При этом каждый год объем продукции снижался на одно и то же число процентов. На сколько? 7.17. Во всех подъездах дома одинаковое число этажей, и на каждом этаже одинаковое число квартир. При этом число квартир на этаже меньше числа этажей, но больше числа подъездов, а число подъездов больше одного. Сколько в доме этажей, если всего в доме 60 квартир? не
7.18. Натуральное число умножили на каждую из его цифр. Получилось 1995. Найдите исходное число. 7.19. Один сапфир и два топаза ценней, чем изумруд, в три раза. А семь сапфиров и топаз его ценнее в восемь раз. Определить мы просим Вас, сапфир ценнее иль топаз? 7.20. Фигура на рис. 38 состоит из одних квадратов. Найдите сторону левого нижнего, если сторона самого маленького равна 1. Рис. 38 7.21. Из натурального числа вычли сумму его цифр, из полученного числа снова вычли сумму его (полученного числа) цифр и т. д. после одиннадцати таких вычитаний впервые получился нуль. С какого числа начали? 7.22. Существует ли такое натуральное число я, что число 26 4- 29 4- 2п является полным квадратом? 7.23. В треугольнике ABC угол А равен 40°, угол В равен 20°, slAB-BC = 4. Найдите длину биссектрисы угла С. 7.24. Одновременно были зажжены две свечи одинаковой длины: одна потолще (сгорающая за 4 часа), другая потоньше (сгорающая за 2 часа). Через некоторое время обе свечи были потушены. Оказалось, что огарок толстой свечи в 3 раза длиннее огарка тонкой свечи. Сколько времени горели свечи? 7.25. В некотором государстве имеется несколько городов. Из каждого города выходит хотя бы одна дорога, и между лю- 117
быми двумя городами не больше одной дороги. Сколько городов может быть в этом государстве, если дорог всего 7? 7.26. Ученики 7 класса Саша, Миша, Гриша, Дима и Витя соревновались в беге, в прыжках в высоту и в длину. Каждый ученик был в майке какого-то одного цвета: красного, синего или зеленого. Удивительно, но каждый раз на первом месте был ученик в красной майке, на втором — в синей, на третьем — в зеленой. Также известно, что последние места в беге занял Саша, в прыжках в длину — Гриша, а в прыжках в высоту — Витя. Могут ли у Миши и Димы быть майки одного цвета? 7.27. В вершинах треугольника записано по натуральному числу, на каждой стороне — произведение чисел, записанных в ее концах, а внутри треугольника — произведение чисел, записанных в его вершинах. Сумма всех семи чисел равна 1000. Какие числа записаны в вершинах треугольника? 7.28. Углы, прилежащие к одной из сторон треугольника, равны 15° и 30°. Какой угол образует с этой стороной приведенная к ней медиана? 7.29. В равенстве: AM ЭМ = МЭ- МА разные буквы обозначают различные цифры. Докажите, что М = А Э М' 7.30. На плоскости изображены 4 равных треугольника так, что любые два имеют ровно 2 общие вершины. Верно ли, что все они имеют общую вершину? 7.31. Математический бой между 7-А и 7-Б начался между 10 и 11 часами, когда часовая и минутная стрелки были направлены в противоположные стороны, а закончился между 16 и 17 часами, когда стрелки совпали. Сколько времени продолжался математический бой? 7.32. Вася забыл 2 последние цифры в телефонном номере своего одноклассника, который имеет 6 цифр. Тем не менее, Вася помнит, что цифры «0» среди них не было. Сколько различных звонков должен сделать Вася в самом неудачном случае, чтобы дозвониться до одноклассника? 118
7.33. О натуральном числе, большем 2, были высказаны следующие утверждения: 1) оно четно; 2) это число 7; 3) это число простое. Найдите это число, если два из приведенных утверждений истинны, а одно — ложно. 7.34. На столе лежат 60 конфет. Два семиклассника по очереди берут не более 4 конфет за один раз. Выигравшим считается тот, кто возьмет со стола последнюю конфету. Кто выиграет при правильной игре? 7.35. Сравните числа: 100100 и 5050 • 15050. 8 класс 8.1. Постройте треугольник по трем медианам. 8.2. Постройте биссектрису острого угла треугольника, вершина которого недоступна. 8.3. Какой знак следует поставить между дробями г и з> о а чтобы в результате получить дробь ^—£ равную каждой из о + а дробей г и з? о а 8.4. Можно ли разрезать разносторонний треугольник на два равных треугольника? 8.5. Сколькими способами можно разрезать равносторонний треугольник на 2 равных треугольника? 8.6. Сравните числа: 8.7. Часы показывают полдень. Через какое время часовая и минутная стрелки снова совпадут? 8.8. Равны ли 2 треугольника, если они имеют по 3 равных угла и по 2 равных стороны? 8.9. 4 черные коровы и 3 рыжие дают за 5 дней столько молока, сколько 3 черные коровы и 3 рыжих за 4 дня. У каких коров удои больше — у черных или у рыжих? 8.10. На какую цифру оканчивается число 20092011? 119
8.11. За круглым столом сидят 20 человек, причем более половины юношей. Докажите, что какие-то два юноши сидят напротив друг друга. 8.12. В классе 37 учеников. Докажите, что среди них найдутся 4 ученика, отмечающих день рождения в один месяц. 8.13. Три подруги-ученицы: отличница Белова, хорошист- ка Чернова и троечница Рыжова собирались на дискотеку. Вдруг черноволосая заметила: «Как интересно, одна из нас — имеет белые волосы, другая — черноволосая, а третья — рыжая. Но ни у кого из нас цвет волос не совпадает с фамилией». «Да, ты права», — поддержала отличница. Какого цвета волосы были у хорошистки? 8.14. Сосчитайте: л iWi-iWi-iWi-^V .fi—Ч V А) V 9 У V 167 V 25У '" V 225Г 8.15. Найдите последнюю цифру числа 82011. 8.16. Как построить угол в 1°, имея шаблон угла величиной в 19°? 8.17. Кощей Бессмертный зарыл клад на глубину 1 м. Этого ему показалось недостаточно, он откопал клад, углубил колодец до 2 м и снова зарыл. Этого ему снова показалось мало, он отрыл клад, углубил яму до 3 м и зарыл. Затем он проделал то же, углубив колодец до 4 м, потом до 5 м, до 6 м и т. д. Известно, что колодец глубиной п метров Кощей выкапывает за п2 дней. Известно также, что на 1001-й день Кощей умер от непосильной работы. На какой глубине остался клад? (Временем, необходимым для закапывания колодца, пренебречь.) 8.18. Докажите, что п2 + п + 1 при любом натуральном п: а) есть число нечетное; б) не является квадратом никакого другого натурального числа. 8.19. Среди 81 монеты имеется 1 фальшивая (более легкая). Как ее найти, используя всего 4 взвешивания? 8.20. Среди 79 монет имеется 1 фальшивая (более легкая) монета. Как ее найти, используя не более 4 взвешиваний? 8.21. Пусть а,Ь — катеты прямоугольного треугольника, а с — гипотенуза. Что больше: а3 4- Ьг или с3? 120
8.22. Упростите выражение: (a-b)s + (ft-c)3 4- (а-с)г. 8.23. Квадрат ABCD со стороной 2 см и квадрат DEFK со стороной 1 см стоят рядом на верхней стороне АК квадрата AKLM со стороной 3 см. Между парами точек А и Е> В и Fy С hK,DhL натянуты паутинки. Паук поднимается снизу вверх по маршруту AEFB и спускается по маршруту CKDL (рис. 39). Какой маршрут короче? 8.24. Можно ли в центры клеток шахматной доски 8x8 вбить 16 гвоздей так, чтобы никакие три гвоздя не лежали на одной прямой? 8.25. Зная, что число 2011 — простое, выясните, существуют ли натуральные числа хиу такие, что: а) х2-у2 = б) *3-у3 = в) х4-у4 = 8.26. Решите уравнение: 1993 1 + 8: (l+8: (1-8: (1 + 4 : (1-4 : (1-8*))))). 8.27. В результате измерения четырех сторон и одной из диагоналей некоторого четырехугольника получились числа 2; 4; 5,5; 10; 15. Чему равна длина измеренной диагонали? х + у— 8.28. Сократите дробь: 8.29. Бригада лесорубов решила вырубить сосновый лес, но экологи запротестовали. Тогда бригадир успокоил экологов, сказав им: «В нашем лесу сосны составляют 99% от всего леса. После рубки леса сосны будут составлять 98% всех деревьев». Какую часть леса вырубит бригада? 8.30. В квадрате ABCD проведены отрезки СЕ и CF, где Е — середина АВ, F — середина AD (рис. 40). Докажите, что СЕ и CF делят отрезок BD на три равные части. 8.31. Докажите, что любое четырехзначное число больше произведения его цифр. 8.32. М. В. Ломоносов тратил одну денежку на хлеб и квас. Когда цены выросли на 20%, на ту же денежку он приобретал полхлеба и квас. Хватит ли той же денежки хотя бы на квас, если цены еще вырастут на 20% ? 121
в А Е С \ d\ м F В К Рис. 39 8.33. Докажите, что все числа вида 1007,10017,100117,... делятся на 53. 8.34. Первая слева цифра четырехзначного числа 7. Если эту цифру перенести на последнее место, то число уменьшится на 864. Найдите четырехзначное число. 8.35. Найдите наименьший угол между часовой и минутной стрелкой, если часы показывают 12 ч 35 мин. 9 класс 9.1. Решите систему уравнений: /(х-2)2 = 2-х. 9.2. Найдите сумму коэффициентов многочлена, получающегося после раскрытия скобок и приведения подобных членов в выражении (4-5* 4- *2)2010(4 4- Ъх 4 х2)2011. 9.3. Иван сказал в этом году: «Мне будет п лет, когда пойдет п2 год». В каком году родился Иван? 9.4. Решите уравнение: х2 4 ху 4 у2-2х 4 2у 4 4 = 0. 9.5. Дома Винни-Пуха, Пятачка, Кролика и Совы расположены на окружности. Где нужно построить дом ослику Иа, чтобы сумма расстояний от него до домов остальных была наименьшей? 122
9.6. Цены снизили на 20%. На сколько процентов больше можно купить товаров на те же деньги? 9.7. По кругу расположены 9 шестеренок так, что первая сцеплена со второй, вторая с третьей и т. д., девятая с первой. Могут ли эти шестеренки вращаться? 9.8. Разрежьте квадрат на 5 прямоугольников так, чтобы у соседних прямоугольников стороны не совпадали. 9.9. Известно, что а + b + с = 5, ab + ас + be = 5. Чему может равняться а2 + Ь2 + с2? 9.10. Четверо ребят — Алексей, Борис, Владимир и Григорий участвовали в лыжных гонках. На следующий день, на вопрос кто какое место занял, они ответили так: Алексей: «Я не был ни первым и ни последним». Борис: «Я не был последним». Владимир: «Я был первым». Григорий: «Я был последним». Известно, что три из этих ответов были правдивыми, а один — ложью. Кто сказал правду? Кто был первым? 9.11. В очереди в школьный буфет стоят Вика, Соня, Боря, Денис и Алла. Вика стоит впереди Сони, но после Аллы; Боря и Алла не стоят рядом; Денис не находится рядом ни с Аллой, ни с Викой, ни с Борей. В каком порядке стоят ребята? 9.12. Верно ли, что УэПп/б- \/9-4\/5 = 1? 9.13. Сравните числа без помощи калькуляторов, таблиц: a) \/66 и л/7 + \/Зб; б) л/б + \/17 и 2>/3 + \/10. 9.14. Существуют ли натуральные числа тип, для которых верно равенство: т2 + 2010 = п2? 9.15. «Докажем», что 2 = 3: 4-10+^ = 9-15 + ^, 4 4 У-(НУ- Извлекая квадратные корни из обеих частей равенства, полу- 5 5 чаем 2—— = 3—— ,то есть 2 = 3. Найдите ошибку. Li Li 9.16. Освободитесь от иррациональности в знаменателе дроби: 123
9.17. Директор школы беседует с 4 учениками школы, подозреваемыми в хищении классного журнала из учительской. Александр сказал, что журнал похитил Борис. Борис утверждал, что виноват Григорий. Григорий заверил директора, что Борис врет. Виктор настаивал на том, что журнал взял не он. Директору школы удалось установить, что один из учащихся сказал все же правду. Кто похитил журнал? 9.18. Найдите такие числа аиЬ, что а + Ъ = аЪ = ^. о 9.19. Какое из 2 чисел: 2\/2 + ^18 или 4, больше? 9.20. Постройте график: \у\ = х. 9.21. Найдите двузначное число, равное удвоенному произведению его цифр. 9.22. Разложите на множители выражение я4 + 4. 9.23. Решите уравнение: (х2—х—1)2-хг = 5. 9.24. На занятии математического кружка 10 школьников получили 42 жетона за правильное решение задач. Докажите, что по крайней мере 2 ученика получили жетонов поровну (возможно и по нулю). 9.25. Докажите, что число ababab делится на 21. 9.26. Две стороны треугольника равны 49 см и 99 см, а угол между ними 59°. Каким является этот треугольник: остроугольным, прямоугольным или тупоугольным? 9.27. Малыш и Карлсон разделили круглый торт двумя перпендикулярными разрезами на 4 части. Карлсон взял себе одну наименьшую часть и одну наибольшую часть, а остальные две отдал Малышу. Кому торта досталось не меньше половины? 9.28. Докажите неравенство ^a2 + 3ab + 2b2 ^ 0, гдеаиЬ — действительные числа. 9.29. На диагонали прямоугольника выбрали точку и провели через нее прямые, параллельные сторонам. По разные стороны от диагонали образовались два прямоугольника. Докажите, что их площади равны. 9.30. На сторонах АВ и ВС параллелограмма ABCD вне его построены равносторонние треугольники АВМ и BCN. Докажите, что треугольник DMN — равносторонний. 9.31. Докажите, что если а ^ 0, то а3 4- За2 4-15 > 13а. 124
9.32. Найдите все пары целых чисел (ху у), удовлетворяющие уравнению 2ху 4 Зу2 = 24. 9.33. Узнайте, через сколько минут после того, как часы показали ровно 4 часа, минутная стрелка догонит часовую. 9.34. Докажите, что уравнение 2009#2-2010i/2 = 2011 не имеет решений в целых числах. 9.35. Верно ли, что при любом четном числе х число Xs 4 + 9х5 + 8*2 делится на 288? 10 класс 10.1. Из двух равных трапеций сложите параллелограмм. Как применить данный результат к выводу формулы площади трапеции? 10.2. Упростите: ^1-27^26 + 9v^262 + s/26. 10.3. Прямая, параллельная основанию треугольника, делит его площадь пополам. В каком отношении она делит боковые стороны треугольника? 10.4. Сколько диагоналей в правильном семиугольнике? 10.5. Каждая сторона одного треугольника больше каждой стороны другого треугольника. Верно ли, что площадь первого обязательно больше площади второго? 10.6. В уравнении (х2 4 ... )(х 4 1) = (*4 4- 1)(* 4- 2) одно число стерто и заменено точками. Найдите стертое число, если известно, что один из корней этого уравнения равен 1. 10.7. Что больше: \/2 4 \/3 4 \/2-Щ или 2л/2? 10.8. Хулиган Витя вырезал из школьной стенгазеты в форме квадрата все то, что ему не понравилось. В итоге остался кусок в форме правильного восьмиугольника. Можно ли по этому восьмиугольнику узнать размеры школьной стенгазеты, если отрезанных кусков было 5 и они имели форму многоугольника? 10.9. Квадрат можно легко разрезать на 2 равных треугольника или 2 равных четырехугольника. А можно ли разрезать квадрат на 2 равных пятиугольника или 2 равных шестиугольника? 125
10.10. Вычислите: a) sin (arccos ~z J; 6) arccos(cos 10). 10.11. Найдите четырехзначное простое число, цифры которого образуют арифметическую прогрессию. 10.12. Могут ли числа 1; \[2\ \[Ъ быть последовательными членами одной арифметической прогрессии? А геометрической? 10.13. От 2 кусков сплавов с разным содержанием свинца массой 6 кг и 12 кг отрезали по куску равной массы. Каждый из отрезанных кусков сплавили с остатком другого сплава, после чего процентное содержание свинца в обоих сплавах стало одинаковым. Каковы массы отрезанных кусков? 10.14. Дан круг. Постройте круг, площадь которого была бы больше площади данного круга в 10 раз. 10.15. Можно ли расположить по кругу цифры 1,2,3,..., 9 так, чтобы сумма никаких 2 соседних чисел не делилась ни на 3, ни на 5, ни на 7? 10.16. При каких значениях а разность корней уравнения ах2 + х-2 = 0 равна 3? 10.17. В магазине продается краска, расфасованная в банки по 3 кг и 5 кг. Докажите, что в этом магазине покупатель всегда может купить нужное, больше 7, число килограммов краски. 10.18. По горизонтальной дороге мотоциклист ехал со скоростью 60 км/ч. На пути ему встретился подъем протяженностью 2 км, за которым следовал спуск протяженностью также 2 км. На подъеме мотоциклист ехал со скоростью 30 км/ч. С какой скоростью мотоциклист должен ехать на спуске, чтобы средняя скорость на подъеме и на спуске составляла 60 км/ч? 10.19. Докажите, что tg -^ = V*"1 . 12 л/3 + 1 10.20. Найдите все решения в простых числах уравнения: х2-2у2 = 1. 10.21. Докажите, что уравнение sin x = ах не может иметь 2008 решений. 10.22. Может ли сумма нечетных чисел 1+3+5+...+(2п-1) при каком-нибудь нечетном п оканчиваться на 2007? 126
10.23. Докажите, что sin6 x + cos6 x ^ 0,25. 10.24. При каких значениях параметра а уравнение х4 + -I- 2л:2 -I- 8 = а не имеет корней? 10.25. Имеется 101 монета, 50 из них — фальшивые. Каждая фальшивая монета тяжелее настоящей монеты на 1 г. Вес монет можно определить с помощью двухчашечных весов со стрелкой, показывающей разность весов на чашках. Петя взял монету и за одно взвешивание хочет определить, фальшивая ли она. Сможет ли он это сделать? 10.26. Бизнесмен каждый месяц фиксировал свой доход и расход. Может ли быть так, что за любые пять подряд идущих месяца его общий расход превышает доход за эти месяцы, а в целом за год доход превышает расход? 10.27. В равнобедренном треугольнике с основанием, равным а и боковыми сторонами, равными Ь, угол при вершине треугольника равен 20°. Докажите, что а3 4- Ьг = ЗаЬ2. 10.28. Первый член последовательности равен 1, второй ее член равен 2000, а каждый следующий член, начиная с третьего, равен разности двух предыдущих. Найдите 2010-й член этой последовательности. 10.29. Докажите, что 131-11! кратно 31. 10.30. Докажите, что число 47 4- 716 — составное. 10.31. Докажите, что 10.32. На окружности отмечены 4 точки —А, В, С, £>, причем дуга АВ содержит 50°, а дуга CD содержит 30°. Чему может быть равен угол между прямыми АС и BD, если указанные 4 точки являются вершинами трапеции? 11 класс 11.1. Решите уравнение: a) arcsin(lgx) = 0; б) lg(arcsinx) = 0. 11.2. Постройте графики функций: а) у = sin(arcsinx); б) у = arccos|a;|. 127
11.3. Определите число решений уравнения \х\ = х-а в зависимости от параметра а. 11.4. Решите уравнение: х2-4 = у/х + 4. 11.5. Решите уравнение в целых числах: 2х2-\ = 2ху. 11.6. Решите уравнение: tg2 х = -— l 11.7. Решите уравнение в натуральных числах: 2п + 7 = А:2. 11.8. Найдите четырехзначное число, которое в 4 раза меньше числа, записанного теми же цифрами, но в обратном порядке. 11.9. Решите уравнение: 2х + 3* = |. о 11.10. Первая цифра некоторого шестизначного числа равна 1. Если эту цифру переставить в конец числа, оставив остальные цифры без изменения, то полученное число окажется втрое больше исходного. Найдите исходное число. 11.11. Десять машин выпускают одинаковые резиновые мячи массой по 10 г каждый. Одна из машин испортилась и стала выпускать мячи массой по 5 г. Как найти испортившуюся машину с помощью одного взвешивания мячей? 11.12. Без помощи таблиц вычислите, какое из чисел log3 7 или Iog7 27 больше? 11.13. Решите систему уравнений: * + i = У+ 1 = 11.14. Почему крупную картошку чистить быстрее, чем мелкую? 11.15. «Докажем», что 2 > 3: значит, 2 > 3. Найдите ошибку. 11.16. Докажите, что число lg 3 — иррациональное. 11.17. На рынке продают два арбуза разных размеров: один арбуз в обхвате на четверть больше другого, зато в полтора раза дороже. Какой арбуз выгоднее купить? 128
11.18. Сколько положительных членов есть среди 2009 членов последовательности: sin 1°, sin 10°, sin 100°, sin 1000°, ...? 11.19. Решите уравнение 2л:2 4- 5ху—12у2 = 28 в натуральных числах. 11.20. Три квадрата расположены, как на рис. 41. Найдите величину угла между прямыми АС и BD. В Рис. 41 11.21. Антиквар приобрел 99 одинаковых по виду старинных монет. Ему сообщили, что одна из этих монет — фальшивая. Она легче настоящих монет, которые весят одинаково. Как, используя чашечные весы без гирь, за 7 взвешиваний выявить фальшивую монету, если антиквар не разрешает никакую монету взвешивать более двух раз? 11.22. На острове Невезения, где живут только рыцари, всегда говорящие правду, и лжецы, которые всегда лгут, существует три политические партии, причем любой житель состоит в одной из них. Каждому жителю задали три вопроса: состоите ли Вы в первой, состоите ли Вы во второй, состоите ли Вы в третьей партии. На эти вопросы были получены 60%, 50%, 40% утвердительных ответов соответственно. Кого во второй партии больше: рыцарей или лжецов? 11.23. Задача Озанама (Жак Озанам (1640-1717) — французский математик, автор «Курса математики» и сборника задач «Математические и физические развлечения»). Семеро друзей собрались к обеду, но между ними возник церемонный спор, кому и с кем садиться. Чтобы прекратить пререкания, кто-то из присутствующих предложил всем сесть за стол как придется, но с условием, чтобы вновь собраться на другой день и затем в следующие дни обедать вместе, причем каждый раз садиться по-разному до тех пор, пока не бу- 129
дут испробованы все возможные комбинации. Спрашивается, сколько раз придется им обедать вместе для этой цели? 11.24. Докажите, что пъ—п делится на 5. 11.25. Решите уравнение: sin(|sin(|sin(|sin(|,))))=O. 11.26. Три боковых ребра четырехугольной пирамиды равны 1 см, 6 см и 11 см. Может ли основанием пирамиды быть квадрат? Ответ обоснуйте. г2 г 11.27. Решите уравнение: -^—т + 2 • * +3 = 0. 11.28. Решите уравнение: 4 + х2 = 8~sln * 2 + ЛГ 11.29. Решите уравнение: sin х • cos x • cos 2x • cos 8x = ^ sin 12л:. 4 11.30. Постройте график функции: 1-х у(х) = arctg х + arctg -т——.
Раздел седьмой ОТВЕТЫ, УКАЗАНИЯ, РЕШЕНИЯ Примерные тексты школьных олимпиад (4 раздел) 5 класс Вариант 1 1. ,321 V321 A = 3 11 11 Б = 2 332 ,321 В=1 321 Г = 5 3531 2.3. 3. Из трех чисел как минимум два являются одинаковой четности, значит, их сумма делится на 2. 4. а = 13 17 +12 = 233. 5. Точки А и В могут лежать по одну или по разные стороны от точки О. Рассмотрим первый случай: Аи В лежат по одну сторону от точки О (см. рис. 42). 1) а < Ъ. Тогда АВ = ОВ-ОА = b-а, ОМ = ОВ-МВ = = &-|(&-а)=±(а + &). 2) а > Ъ, тогда А и В меняются местами и АВ = ОА—ОВ = = a-b, ОМ = ОА-МА = а-\(а-Ъ)-\а + \b= ha + Ъ). Li Li Ct Ct 3) a = ft, тогда точки А и В совпадут и ОМ = ОА = a = b. Рассмотрим второй случай: точки Аи В лежат по разные стороны от точки О (см. рис. 43). II i i i 1 1 1 О А М В А ОМ В Рис. 42 Рис. 43 131
Рассуждая аналогично, получаем: ОМ = ^(Ь-а)у если Ъ > а, ОМ = ^(а-6), если Ъ < а, ОМ = 0, если Ъ = а. 6. Надо вычеркнуть 100 цифр, причем оставить как можно больше цифр «9» впереди. Тогда до первой цифры «9» вычеркнем 8 цифр, до второй — 19, до третьей — 19, до четвертой — 19, до пятой — 19. Таким образом, мы вычеркнем 19 • 4 + 8 = 84 цифры. Останется вычеркнуть 16 цифр из оставшегося числа 999995051525354555657585960. Вычеркнем теперь 15 цифр, стоящих перед семеркой. Остается число 999997585960. Осталось вычеркнуть одну пятерку. Таким образом, останется число 99999785960. Вариант 2 1.58. 2. 45 рублей, так как распилов надо сделать 9. 3. В сутках 24 часа, поэтому 100 ч = 4-24ч+4ч = 4сут+4ч. Поэтому парусник вернется в пятницу в 16 ч. 4. См. рис. 44. 5. Из второго предложения ясно, что Аня и Валя не в зеленом платье, Надя — не в зеленом и не в голубом. Из третьего предложения следует, что Валя не в розовом и не в белом платье. Тогда Валя будет в голубом платье, а Галя в зеленом. Используя первое предложение, изобразив девочек по кругу, получим, что Галя будет стоять между Валей и Надей. Тогда Аня в белом, а Надя в розовом платье. Ответ: Валя, Аня и Надя соответственно в голубом, белом и розовом платьях. 6. См. рис. 45. Вариант 3 1. 9 + п -99 = 999, п = 10. Ответ: 10 раз. 132
2. а) (7-9 +12): 3-2 = 23, б) (7-9+12): (3-2) = 75. 3. 30 мин : 2 = 15 мин — Сережа едет в школу автобусом в одну сторону. 1 ч 30 мин—15 мин = 1 ч 15 мин — Сережа идет пешком в одну сторону. 1 ч 15 мин 4-1 ч 15 мин = 2 ч 30 мин — пешком в оба конца. Ответ: 2 ч 30 мин. Рис. 44 ч? Рис. 45 4. Для решения задачи применим графы (см. рис. 46). Салтан v^ ^ Юра Гвидон Черномор Коля Миша Рис. 46 Так как к Салтану идет лишь одна стрелка, то Коля будет играть Салтана. Тогда Коля не будет Черномором, а значит, Черномором будет Юра и Миша — Гвидоном. 5. 6-12-18 = 1536 (см3) — объем параллелепипеда. При постановке кубиков объемом 1 см3 друг на друга получим вышку высотой 15 м 36 см. Так как лестница всего длиной 3 м, то рост мальчика с вытянутой рукой должен быть 15 м 36 см—3 м = = 12 м 36 см, чего не может быть. 133
Вариант 4 1. 9999999 — наибольшее и 1000000 — наименьшее. 2. 5 щенят и 12 утят. 3. 38 рублей. 4. 1) Наполняем семилитровый сосуд, переливаем из него 5 л в пятилитровый, затем 5 л выливаем, а оставшиеся 2 л в семилитровом сосуде выливаем вновь в пятилитровый сосуд. 2) Снова наполняем семилитровый сосуд, отливаем из него 3 л в пятилитровый сосуд. Тогда в семилитровом остается 4 л. Выливаем все из пятилитрового сосуда и выливаем в него 4 л из семилитрового сосуда. 3) Наполняем вновь семилитровый сосуд, отливаем из него 1 л в пятилитровый сосуд. Таким образом, в семилитровом сосуде получаем 6 л. 5. См. рис. 47. ч. *ч, ч^ ч ч ч ч ч ч ч ч Ч ч ч, *ч, Ч Ч, Ч Ч., X, Ч Ч \ ■ч. 4j \ ч, ч \, Ч^ ч ч ч ч. ч, ч^ ч^ ч ч. ч Ч ч. % \ \ ч ч. ^ч. ^ч ч ч ч ч ч ч. ч ч, %, ч ч ч "ч, ч ч \ ч ч ч ч ч^ ч ч, 4j ч^ \, Ч, Ч Ч ч ч ч ч ч. Ч ч Ч Ч ч ч ч ч \ ч ч Рис.47 Вариант 5 1. 101101 • 999-101 • 999999 = 101 • 1001 • 999-101 - 999х хЮ01 = 0. 2. На первый грузовик поместить 3 полных бочки, 1 наполненную наполовину, 3 пустых бочки; на второй грузовик — 3 полных, 1 наполненную наполовину и 3 пустых бочки; на третий грузовик — 1 полную, 5 наполненных наполовину, 1 пустую. 134
3. Изобразим таблицу набранных очков соответственно при верных 20, 19 и т. д. вопросах: Верных ответов Набрано очков 20 240 19 218 18 196 17 174 16 152 15 130 14 108 13 86 12 64 Из таблицы видно, что ученик ответил верно на 13 вопросов. Можно было заметить закономерность, что каждый раз число набранных очков уменьшается на 22. Ответ: 13. 4. С площадью по 1 кв. ед. будет 9 прямоугольников; 12 — с площадью по 2 кв. ед.; 6 прямоугольников — по 3 кв. ед.; 4 прямоугольника имеют площадь по 4 кв. ед. и 4 — по 6 кв. ед. и 1 — 9 кв. ед. Ответ: 36 прямоугольников. 5. В произведении содержится 5 «пятерок»: по одной дают разложения 10, 15 и 20 на простые множители; а 25 = 5 х 5. Произведение каждой «пятерки» на четный множитель дает нуль, поэтому произведение оканчивается 5 нулями. Ответ: 5 нулей. Вариант 6 1.* = 20. 2. Внучке 7 лет, дедушке 84 года. 3. Используем таблицу. ^^^--^^^ Содержимое ^^^^-^^^ мешка № мешка —-^^^ 1 2 3 Вермишель + Крупа + Сахар + Так как в первом мешке не крупа, то ставим в соответствующей клетке « — ». Аналогично, во второй строке ставим « — » — против вермишели. Так как в третьем мешке — не крупа и не сахар, то ставим «минусы» в столбцах с надписями «крупа» и «сахар». Тогда из таблицы получаем, что в третьем мешке — вермишель, во втором — крупа (крупы нет в 1 и 3 мешках), значит, сахар — в 1 мешке. 135
Ответ: В мешке с надписью «крупа» находится сахар, с надписью «вермишель» — крупа, с надписью «крупа или сахар» — вермишель. 4. Да, возможный вариант изображен на рис. 48. 5.10 =1 + 9 = 2 + 8 = 3 + 7 = 4 + 6. Разместим «5» в центре. Тогда возможный вариант может быть такой (см. рис. 49). Рис. 48 Рис. 49 Вариант 7 1. См. рис. 50. 2. Одна чашка и одно блюдце вместе стоят 25 рублей, поэтому 4 чашки и 4 блюдца будут стоить 100 рублей. Так как по условию задачи 4 чашки и 3 блюдца стоят 88 рублей, то одно блюдце стоит 12 рублей. Тогда одна чашка будет стоить (25-12) рублей = 13 рублей. Ответ: Цена чашки 13 рублей, цена блюдца 12 рублей. 3. Если бы все поросята встали на задние ноги, то на земле оказалось бы 30 • 2 ног. Тогда вверху будет 84-60 = 24 (ноги). Так как каждый поросенок вверх поднял по две ноги, то поросят будет 24 : 2 = 12. Тогда гусей будет 30-12 = 18. Ответ: 12 поросят и 18 гусей. 4. См. рис. 51. 5. Золушка взяла зернышко из мешка с надписью «смесь»; так как ни одна табличка не соответствовала содержимому 136
мешка, то там был мак или просо. Если взятое Золушкой зернышко — мак, то в мешке с надписью «смесь» — мак, тогда в мешке с надписью «мак» — просо, а в мешке с надписью «просо» — смесь. Аналогично, если взятое зернышко — просо, то в мешке с надписью «смесь» —просо. Тогда в мешке с надписью «мак» — смесь, а в мешке с надписью «просо» — мак. Рис. 50 Рис. 51 6. Разделим 9 монет на три кучки по 3 монеты. Произведем первое взвешивание: положим 2 кучки по 3 монеты на каждую чашку весов. Возможны 2 случая: а) весы находятся в равновесии, тогда на весах находятся настоящие монеты; фальшивая монета находится среди тех монет, которые не взвешивались; б) равновесия на весах нет, тогда фальшивая монета среди тех монет, где кучка легче. Определив таким образом кучку с фальшивой монетой, выполним с ней второе взвешивание. Возьмем из трех монет любые две и положим их на чашки весов. Снова возможны 2 случая: а) весы находятся в равновесии, тогда фальшивая монета оставшаяся; б) равновесия нет, в этом случае фальшивая монета там, где вес меньше. 137
7. Напишем искомую сумму дважды: S = 1 + 2 + 3 + ... + 109 + 110+111. S = 111 + 110 + 109 + ... + 3 + 2 + 1. Сложим почленно: 2S = (1 + 111)+ (2+ 110)+... + (110+ 2)+ (111 + 1) = 112-111. Тогда5 = 112-111 : 2 = 6216. Вариант 8 1.10,25,40. 2. 600 : 6 = 100 (г) — съест Малыш за 1 минуту, 6:2 = 3 (мин) — за такое время Карлсон съест все варенье, 600 : 3 = 200 (г) — съест варенье Карлсон за 1 минуту, 100 + 200 = 300 (г) — могут съесть вместе варенье Малыш и Карлсон, 600 : 300 = 2 (мин) — за такое время съедят варенье вместе Малыш и Карлсон. Ответ: 2 мин. 3. х = 3 или х = 4. 4. См. рис. 52. А Рис. 52 5. С помощью трехлитровой банки нальем 6 л воды в ведро. Еще раз нальем 3 л воды в банку и наполним семилитровое ведро доверху. Тогда в банке останется 2 л воды, которую выльем 138
в кастрюлю. Добавим к ним 3 л воды с помощью банки, получим всего 5 л воды. Возможны и другие варианты решения. 6. 315 Х 41 , 315 1260 12915 7. Молоко в кувшине, лимонад в бутылке, квас в банке, вода в стакане. Вариант 9 1. Так как посажено 10 кустов, то промежутков между ними будет 9. Поэтому расстояние между соседними кустами будет 90 : 9 = 10 (дм). 2. х = 9а-8. 3. Велосипедист прошел пешком - пути, то есть в 2 ра- о за меньше, чем проехал на велосипеде. Времени же затратил вдвое больше. Поэтому он ехал в 4 раза быстрее, чем шел. 4.1 • (2 + 3) • 4 • 5 = 100. 5. При разрезании каждого листа на 3 части число листов увеличивается на 2. Добавилось: 15—9 = 6 (листов). Значит, 6:2 = 3 (листа) бумаги разрезали. 6. На первые девять страниц потребуется 9 цифр, на каждые следующие 90 страниц надо по 2 цифры на каждую страницу, а значит, надо 2 • 90 цифр. Пусть в книге х страниц, тогда страниц с тремя цифрами будет #-99, а цифр на них — 3 • (лг-99). Получаем уравнение: 9 + 2-90 + 3(*-99)=1392. Решая его, получаем х = 500. О т в е т: В книге 500 страниц. Вариант 10 1. у = 4. Проверка: 4-4 + 5 = 21. 2. В сутках 24 ч, из них Стрекоза спала 24 : 2 = 12 (ч), танцевала 24 : 3 = 8 (ч), пела 24 : 6 = 4 (ч). Всего на эти дела 139
она потратила 12 + 8 + 4 = 24 (ч), поэтому на подготовку к зиме времени у нее не осталось. 3. 26-25-25-24+24-23-23-22+22-21-2120+2019-1918+ + 18•17-17 • 16 + 16• 15-15 • 14 = 25 • (26-24) + 23 • (24-22) + + 21 • (22-20) + 19 • (20-18) + 17 • (18-16) + 15 • (16-14) = = 2(25 + 23 + 21 +19 + 17 + 15) = 2(40 + 40 + 40) = 2-120 = 240. 4. ^7243 * 29 , 65187 14486 210047 5. Так как девочка ходит в детский сад, то Боре не 5 лет. Так как Аня старше Бори, то Ане 13 или 15 лет. Но сумма лет Ани и Веры делится на 3, поэтому Ане 13 лет, тогда Вере 5 лет. Тогда так как Аня старше Бори, то Боре 8 лет. Гале остается 15 лет. Ответ: Вере 5 лет, Боре 8 лет, Ане 13 лет, Гале 15 лет. 6. 100-10 = 90 (чел.) — знали немецкий или французский языки; 90-75 = 15 (чел.) — не знали немецкого языка; 90-83 = 7 (чел.) — не знали французского языка; 90-(15 + 7) = 68 (чел.) — знали и французский и немецкий языки. Ответ: 68 туристов знали и французский и немецкий языки. Вариант 11 1. Можно решить устно: перенести 3 в правую часть и получить 30, затем поделить обе части на 30 и получить в числителе 7. Так как числитель равен знаменателю, то х-3 = 7, откуда находим х = 10. 2. Площадь фигуры равна 131. 3. Возможны 2 варианта параллелепипеда, построенного из 18 кубиков высотой 3 кубика: 3x3x2 или 3x6x1. Площадь поверхности данных параллелепипедов будет равна 42 и 54 площадей 1 грани. Учитывая, что площадь грани равна — см2, о получим площадь поверхности: 133 см2 или 171 см2. 140
4. Так как вычитаемое и разность в сумме дают уменьшаемое, то два уменьшаемых будут равны 26, а, значит, уменьшаемое будет равно 13. Вариант 12 1. Так как 3 ученика делают за 3 минуты 3 самолетика, то за 9 минут они сделают 9 самолетиков. О т в е т: 3 ученика. 2. Так как масса всей рыбы будет равна (1900 4- 100) • 9 + 4- 1000 = 19000 (г), то каждому рыбаку должно достаться по 1900 г. Значит, разделить рыбу можно следующим образом: 1900 г; 100 г и 1800 г; ... 900 г и 1000 г. 3. Сумма возрастов всех футболистов была равна 11 • 22 = = 242, а после удаления стала 10 • 21 = 210. Значит, возраст удаленного футболиста 32 года. 4. Обозначим за х и у — соответственно первоначальное число посетителей и новую цену билета. Тогда, после снижения цены, посетителей будет 1,5л:, а сбор денег 1,5ху. Так как первоначально денег собрали 150л:, а сбор увеличился на 25%, то получаем уравнение 1,5ху—150л: = 0,25 • 150л:. Решая его, находим у = 125 (руб.), то есть цену снизили на 25 руб. 5. Задача имеет много решений, например: (4,-5,4, -5, 4); (5, -6, 5, -6, 5) и т. д. Вариант 13 1. 48 лети 4 года. 2. Возможный вариант показан на рис. 53. Рис. 53 141
3. Второй охотник съел столько каши, сколько положил крупы, поэтому третий охотник от него ничего не получил. Поэтому все патроны надо отдать первому охотнику. 4. В первом круге число слов должно делиться на 4, во втором — на 3, а в последнем — на 2. Наименьшее число, делящееся на 2; 3; 4, будет 12. Значит, наименьшее число слов в считалке будет равно 12. 6 класс Вариант 1 1.* = -2,6. 2. -4^*^ 3;-1 ^ у^ 5. 3. Число делится на 36, если оно делится и на 4 и на 9. Так как сумма цифр 5, 2, 2 равна 9, то сумма двух недостающих цифр должна равняться 0, 9 или 18. Учитывая, что число должно делиться на 4, а предпоследняя цифра равна 2, то последняя цифра может быть лишь 0 или 4 или 8. Тогда ответами будут числа: 52524, 52128, 52020, 52920. 4. 600 • 40 : 100 = 240 (г) — содержится соли в 600 г жидкости; 240 : 12 • 100 = 2000 (г) — будет 12% -й жидкости; 2000-600 = 1400 (г) — воды надо добавить. Ответ: 1400 г. 5. Так как скорость ученика не может превышать 10 км/ч, то время на дорогу будет не менее — ч, то есть не менее 6 мин. Поэтому ответ может быть таким: ученик придет в школу не раньше 8 часов 6 минут. Возможны и другие варианты ответа. Например, ученик придет в школу между 8 ч 6 мин и 8 ч 20 мин. 6. Так как Аня не проигрывала мальчикам в шахматы, то она — лучший шахматист. Так как художник не нарисовал своего портрета, а нарисовал портрет Игоря, то Игорь — лучший математик, а Олег — лучший художник. Ответ: Олег — лучший художник, Аня — лучший шахматист, Игорь — лучший математик. 142
Вариант 2 1. 59,27 ,44,45 78,43 182,15 2. 55: 5 + 5 = 16. 4. Обозначим число гусей в одном хлеве за х> а число козлят за у, тогда, учитывая, что ног в одном хлеве должно быть 10, получим уравнение: 2х+4у = 10. Из данного уравнения имеем, что число козлят может быть только 1 или 2, соответственно гусей будет 3 или 1. Тогда размещение будет такое: в двух хлевах будет по 1 козленку и 3 гусям, в трех хлевах — по 2 козленка и 1 гусю. 5. Необходимо вынуть шарик из ящика с надписью «черный или белый». Если вынутый шарик окажется белым, значит, в этом ящике 2 белых, в ящике с надписью «2 белых» будет 2 черных, а с надписью «2 черных» будут черный и белый. Аналогично рассуждаем, если вынутый шарик — черный. Вариант 3 о 2. Числа q и 1 представим в виде дробей со знаменателем, кратным 15. Тогда ^ = tf, 1 = т?. Между числами ^ и 1 лежат У 45 45 У дроби тг, т4, tf, 4^. Условию удовлетворяет лишь tf = т1- 45 45 45 45 45 15 Ответ: —. 15 Q VI = 6 = 2 *'1Х 9 3' 4. Так как после зачеркивания получается наибольшее число с суммой цифр 13, то вторая и третья цифры равны 9 и 4. Так как первая цифра больше последней в 4 раза и все цифры различны, то первая цифра будет 8, а последняя 2. В результате получаем число 8942. Ответ: старику Хоттабычу 8942 года. 143
5. Решается с помощью уравнения: jrX 4 ^х 4 j=x 4 3 = х. О т в е т: 28 учеников. Вариант 4 2. 785 Х121 785 ,1570 785 94985 3. 550-55 = 495 (руб) — стала цена в итоге. 4. Так как число после приписывания двух цифр должно делиться на 15, значит, оно будет делиться на 3 и на 5. По признаку делимости на 5 последняя цифра в числе может быть лишь 0 или 5. Используя признак делимости на 3, получим, что первая цифра может быть 3,6,9 (если последняя цифра — 0) или 1,4,7 (если последняя цифра — 5). Тогда ответом будут числа: 1155, 3150, 4155, 6150, 7155, 9150. 5. Так как Володя учится в 6 классе, а Герасимов в 5 классе, то Володя — не Герасимов. Так как отец Иванова — учитель, отец Володи — инженер, то Володя — не Иванов. Тогда Володя — Семенов, Миша — Иванов, а Петя — Герасимов. Можно для наглядности применить графы или таблицы. Вариант 5 1. Возможный вариант указан на рис. 54. 2. Возможный вариант: 25-!-7+(12Ц-4|У 25 + 125-357-0,008 = 25-348±|-254 4 357 = 25-3 + 8 - 25 + 13 + 357 = 22 + 200 + 370 = 592. 3. х = 7 или х = 1. 4. Пусть х — число страниц, которое было в книге. В первый день прочитали (0,2л: 4- 16) страниц; осталось прочитать во второй и третий дни (0,8л:-16) страниц; во второй день прочитали (0,3(0,8л:-16) 4- 20) = (0,24л: 4 15,2) страниц; в третий 144
Рис. 54 день прочитать осталось (0,56л:—31,2) страниц. Так как в третий день прочитали 0,75 остатка и еще 30 книг, то остаток будет составлять 120 страниц. В итоге получаем уравнение: 0,56л:-31,2 = 120, откуда находим х = 270. Ответ: 270 страниц. 5. Так как второе и третье сообщения ложны, то А является третьей планетой, а Б — не второй, поэтому Б — первая планета от звезды. Тогда В будет второй планетой, на которой живут инопланетяне. Вариант 6 1.46,02. 2. х = 10 или х = -4. 3. ,8126 8126 16252 4. Обозначим соответственно первую, вторую и третью цифру числа за a, Ъ> с. Тогда число можно записать 100 000а + 10 000& + 1000с + 100а + 10Ь + с = = 100100а + 10 010& + 1001с = ЮОЦЮОа + 10Ь + с) = = 71113(100а+10& + с). Данное число делится на 7, на 11, на 13. 5. Для доказательства составим таблицу зависимости числа набранных очков от числа решенных задач. 145
Число решенных задач Число набранных очков 10 20 9 17 8 14 7 11 6 8 5 5 4 2 3 0 2 0 1 0 Из таблицы видно, что существует всего 8 различных возможностей получения очков. А так как учеников было 9, то, по крайней мере, два из них получили одинаковое количество очков. Вариант 7 1..3930 3980 7910 2 1 1 l=i • 4 ' 4 " 4 64' Ответ: -^г. 64 5 3. В первом городе взыскали с купца ^ имущества, зна- о чит, осталось — всего имущества. Во втором городе взыскали 6 6 = 36 имУЩества> значит» осталось ё~зб = зб имуще' ства. Аналогично рассуждая, получим, что после третьего города у купца останется -^-= часть имущества. Так как это имущество стоит 1000 денежных единиц, то всего имущества было на 21 600 денежных единиц. Ответ: 21600. 4. Найдем дополнения каждой дроби до 1 и сравним их. i_JL = _L i_10 = i i_H = _L i_12 = _L 10 10' 11 11' 12 12' 13 13' Так как _I_ > _L > _L > i T0 A < io < и < 12 1ак как 10 > n > 12 > 18. то io < И < 12 13' Ответ- ±1 И IO A итвет. 13, 12, n, 1Q. 5. Число различных денежных сумм, которые можно составить из менее чем 1000 дукатов, меньше 1000, то есть меньше числа пиратов. Поэтому у 2 пиратов будет одинаковое число дукатов. 6. Сумма 2 чисел будет четной, если они оба четные или оба нечетные. Сумма 2 чисел будет нечетной, если одно из них будет четное, а другое — нечетное. Допустим, что сумма любых 146
2 соседних чисел нечетна, тогда четные и нечетные числа должны чередоваться. Значит, общее число чисел будет четным, а по условию чисел 2009, — нечетное количество. Значит, допущение сделано неверно, и на самом деле найдутся 2 числа, сумма которых будет четна. Вариант 8 1.17 кг. 2. 14286 14286 85716 114288 , 28572 57144 14286 204089796 3. Так как Наташа в зеленых туфлях, а Валя не в белых, то Валя в синих туфлях. Значит, Аня в белых туфлях. Так как цвет платья и туфель у Ани совпадает, то Аня в белом платье. Так как у остальных девочек цвет платья и туфель не совпадает, то Валя в зеленом платье, а Наташа — в синем. Ответ: Аня в белом платье и белых туфлях, Валя в зеленом платье и синих туфлях, Наташа в синем платье и зеленых туфлях. 4. 35—10 = 25 (учеников) — посещают кружки, 25—20 = 5 (учеников) — посещают лишь экологический кружок, 11—5 = 6 (учеников) — посещают оба кружка. О т в е т: 6 экологов увлекаются математикой. 5. Допустим, что во всех классах не менее 35 учеников, тогда во всей школе будет не менее чем 35-33 = 1155 (учеников), что противоречит условию задачи. Значит, в школе найдется класс, в котором менее, чем 35 учеников. Вариант 9 1. Так как в один конец Дима пешком тратит на 20 минут больше, чем на велосипеде, то в оба конца он потратит пеш- 147
ком больше на 40 минут. Значит, всего на путь туда и обратно пешком он потратит 1 час. 2. Возможный вариант показан на рис. 55. Рис. 55 3. Так как каждая грань большего кубика в 9 раз больше грани маленького, то и краски понадобится в 9 раз больше, то есть 18 г. 4. Решение лучше найти подбором. Пусть Маша за все покупки заплатила по 13 рублей, тогда покупок она сделала 18 и 5 рублей осталось (239 = 13 • 18 4- 5). Но 5 рублей остаться не может, так как разность в стоимости 1 блокнота и 1 тетради составляет 2 рубля. Денег должно остаться четное число. Значит, надо сделать 17 покупок, а 18 рублей доплатить за 9 блокнотов. Тогда тетрадей будет 8, а блокнотов — 9. Других решений не будет, так как следующее четное число после 18 будет 34. Оно получается при 15 покупках, а так как 34 : 2 = 17, то получается противоречие. Замечание: Задачу в старших классах можно было решить, применив метод решения линейных уравнений с 2 переменными в целых числах. Вариант 10 1. Так как знаменатель второй дроби в 20 раз больше знаменателя первой дроби, то корень уравнения можно найти устно: х= 12,3-20 + 4 = 250. 2. Разложив 3232 на множители, получим: 3232 = 32 • 101 = 2 • 2 • 2 • 2 • 2 • 101. Так как все двойки должны быть в одном числе, то эти числа будут 32 и 101. Так как наименьшее кратное двух взаимно 148
простых чисел будет равно их произведению, то оно будет равно 3232. 3. Из уравнения 13,5л: = 12,Ъу следует, что х < j/, если х и у — положительные числа; х = у, если х = 0 и у = 0; х > у, если х и у — отрицательные числа. 4. Так как верхние прямоугольники имеют общую сторону и площадь правого в 2 раза больше, то и его вторая сторона будет в 2 раза больше. Аналогично и вторая сторона правого нижнего прямоугольника будет больше стороны верхнего левого прямоугольника в 3 раза. А это означает, что площадь нижнего правого четырехугольника будет в 6 раз больше площади левого верхнего прямоугольника, то есть будет равна 12 см2. Поэтому площадь всего прямоугольника будет равна 24 см2. Вариант 11 1. За 1 день первая и вторая овцы съедят вместе (1 4- A J копны сена, а все остальные: i + T + ^ + ^ + ^ + ir (копны сена). о 4 о о 7 о Тя v wow 1-1-1-1-1 <r A -I- A -I- A — 1 ol-|-i-|-i<i-|-l-|-l — I rrn Гаккак -+-+-< g + g + g - l, a -+ ? + g < g + - + ---, то первые две овцы имеют большую скорость поедания, а значит, и съедят 1 копну сена быстрее. 2. Андрей и Борис менялись местами четное число раз, поэтому Андрей останется впереди Бориса. Андрей и Виктор менялись местами также четное число раз, поэтому Андрей останется впереди Виктора. Борис же и Виктор менялись местами нечетное число раз, поэтому Виктор придет раньше Бориса. Тогда порядок спортсменов на финише будет такой: Андрей, Виктор, Борис. 3. Разделим всех учеников на 2 группы: в первой — мальчики, во второй — девочки. Затем мальчиков, которые не любят математику, переведем во вторую группу, а девочек, которые любят математику, — в первую. Численности групп от этого не изменятся. Но в первой группе будут все ученики, которые любят математику, поэтому учеников, которые любят математику столько же, сколько и мальчиков. 4. Например: 20042004...2004 (цифры 2,0, 0, 4 повторяются 334 или 2004 раза). 149
7 класс Вариант 1 1. а) х = -9 при с = -1; х не может равняться нулю ни при 1 каком с; х = — при с = 45; 5 б) при с = 0; в) л: > 0 при с > 0. 2. а) (-а)2; З.На32%. 4. График состоит из 2 прямых: а) у = -3 и х = 2 (см. рис. 56а); б) I/ = -л: и х = 0 (см. рис. 566). б) -(-а)3. 1 О -3 О Рис.56 5. х 88 Х 88 , 88 88 9768 6. Пусть у Андреева первое утверждение верное, то есть он из Онеги. Тогда Григорьев живет не в Каргополе. Поэтому второе утверждение Данилова — ложное, значит, он из Вельска. Тогда первое утверждение Григорьева — ложно. Так как Андреев из Онеги, то первое утверждение Васильева ложно, поэтому Борисов — из Котласа. Так как Григорьев не из Каргополя, то остается, что он из Коряжмы, а Васильев из Каргополя. 150
Рассмотрим второй возможный вариант. Пусть у Андреева второе утверждение — правильное, тогда Григорьев приехал из Каргополя. Значит, Данилов приехал не из Вельска, а Андреев не из Онеги. Тогда у Борисова первое утверждение ложное (в Каргополе живет Григорьев), значит, Борисов прибыл из Коряжмы. Поэтому Андреев не из Коряжмы и получается, что Данилов из Вельска. Получили противоречие: Данилов из Вельска и не из Вельска. Значит, второй вариант невозможен. Ответ: Андреев из Онеги; Борисов из Котласа; Васильев из Каргополя; Григорьев из Коряжмы; Данилов из Вельска. Вариант 2 1.* = -2. 2. 24 и 32. З.х= 16,3 или х = -2,3. 4 46 ■ 95 + б9 ■ 120 _ 212 ■ З10 + 29 ■ З9 - 23 ■ 3 • 5 _ • 84.312_6П 212.312.2П.311 _ 212-310-(1 + 5)_ 2 6 _4 211-311-(6-1) 35 5 е 5. Решение задачи аналогично решению задачи 6 (вариант 7; 5 класс). Вариант 3 1. 9 + || = 10; 9 + 999"9 = 10; ^-| = 10; э| • | = 10; 9+ §-9 + 9 = 10. 2. Упростив левую часть, получим в ней 0. 3. х = 1,1/ — любое число; либо у = 1, х — любое число. 4. Применив формулу а2-Ь2 = (a—b)(a + b) к равенству а2-Ь2 = а + ft, получим а—Ъ = 1. Значит, все пары (a; ft), для которых а-Ъ = 1, будут являться решением уравнения a2_b2 = a + bt 5. Сначала квадрат 5x5 разрежем на 25 квадратов 1 х 1, затем каждый из полученных квадратов разрежем по диагонали на 4 треугольника, из которых, прикладывая большие стороны 151
2 треугольников друг к другу, можно получить по 2 квадрата (см. рис. 57). Рис.57 Вариант 4 1.420 га. 2. л: = ±1,6. 3. Так как четных четвергов три, то всего четвергов в месяце должно быть 5, причем первый четверг должен быть в начале месяца и в четный день. Значит, этот четверг приходится на второй день месяца, тогда следующие четверги будут 9,16, 23, 30. Если первый четверг попадает на четвертое число месяца, то в месяце будет всего 4 четверга. Если четверг попадает на первый или третий день месяца, то не будет 3 четных четвергов. Тогда 26-е будет воскресеньем. Ответ: 26-е будет воскресеньем. 4. Наливаем бензин в 5-литровый бидон и переливаем в бак мотоцикла. Затем вновь наливаем бензин в 5-литровый бидон, переливаем в 9-литровое ведро, наливаем еще раз в 5-литровый бидон и отливаем недостающие 4 л в 9-литровое ведро. Тогда в 5-литровом бидоне остается ровно 1 л, его и переливаем в бак мотоцикла. 5. а) 1 • (9-9) -9 = 0; б) 19-9-9 = 1; в) 1 + (9 + 9): 9 = 3; г) 1-9 : 9 + 9 = 9; д) 1 + 9-9 + 9= 10. Вариант 5 1. 160 яиц. 2.3. 152
3. Для упрощения вычислений в начале необходимо сократить дроби, а затем уже воспользоваться формулой (a+b)(a—b). Ответ: 16. 4. Так как АВ = ВС, то ABAC = Z.BCA (см. рис. 58). Далее, /ABE = 90°-/EBD, /CBD = 90°-/EBD. Отсюда /ABE = = /CBD. Итак, имеем: АВ = ВС, /.ВАС = /ВСА, /.ABE = = /CBD. Значит, ААВЕ = ABCD. Рис. 58 5. Так как у вас может быть всего 4 прабабушки и 4 прадедушки, а у каждого из прабабушек и прадедушек может в свою очередь быть по 2 бабушки и 4 прабабушки, то всего может быть по 16 бабушек и 32 прабабушки. Ответ: 16 бабушек и 32 прабабушки. Вариант 6 1.На44%. 2.54. 3. Пусть Мартышка села на первый пень, тогда вариантов сесть на три оставшихся пня у Осла, Козла и Косолапого Мишки будет 6: ОКМ, ОМК, КОМ, КМО, МКО, МОК (обозначили по первым буквам). Аналогично получится и в остальных случаях, когда на первый пень будут садиться Осел или Козел или Мишка. В сумме всего получится 24 варианта, поэтому пересаживаться придется 23 раза. Ответ: 23 раза. 4. Обозначим вторую и третью цифры трехзначного числа за х и у, тогда первоначальное число будет иметь вид: 900 + Юл: + у. После переноса цифры «9» в конец числа число 153
будет иметь вид: 100л: + 10у + 9. Учитывая, что разность чисел равна 216, после упрощения имеем: 90л: 4- 9у = 675. Разделив обе части на 9, имеем Юх + у = 75. Учитывая, что 0 < х < 10, 0 ^ у < 10, получим х = 7, у = 5. Ответ: 975. Вариант 7 1. Преобразуя числитель к выражению 522, и сокращая дробь на 522, получим ^. ^5 2. Так как положенное количество соли в 2 раза меньше того, которое нужно еще добавить, то оно составляет треть необходимого. Ответ: ^. 3. В выражении использованы десять различных букв, что соответствует 10 различным цифрам. Так как на 0 делить нельзя, то 0 будет входить в числитель. Поэтому дробь равна 0. 4. См. рис. 59. 5. Разделим 24 кг на две части, отмерив на весах по 12 кг гвоздей. Отложим одну кучу гвоздей, а вторую поделим поровну. Получим две кучи по 6 кг. Одну из них поделим пополам. Всего у нас будет четыре кучи гвоздей по 12, 6, 3, 3 кг. Сложим вторую и третью, получим ровно 9 кг. Вариант 8 1. Младшему 14 лет, а старшему 21 год. 2. 20 : 100 • 6 = 1,2 (т) — примесей в стали; 20-1,2 = 18,8 (т) — чистой стали; 40-18,8 = 21,2 (т) — примесей в руде; 21,2 : 40 • 100% = 53% — процент примесей в руде. Ответ: 53% примесей. 1 2 3. Первый фонтан за 1 час наполнит тг~к > т* е* ~Е бассейна, а второй фонтан за 1 час наполнит , т. е. — бассейна. о,75 15 154
Работая вместе, фонтаны за 1 час наполнят 2 + А= 10 = 2 5 15 15 3 (бассейна), поэтому весь бассейн будет наполнен за 1,5 ч. Ответ: 1,5 часа. 4. у 2х, если х ^ О, О, если х < 0. График — на рис. 60. У=2х Рис. 59 Рис. 60 5. х = -4,5 или л: = 1,5. 6. Так как ADBE = AECF = AFAD (по двум сторонам и углу между ними), то DE = EF = FD. Поэтому ADEF — равносторонний. Вариант 9 1. Существует 8 способов для выбора первой цифры такого числа (любая цифра, кроме 0 и 1) и 9 способов для выбора второй цифры (любая цифра, кроме 1). Тогда двузначных чисел, в записи которых не употребляется цифра 1, будет 8 • 9 = 72. Ответ: 72 числа. 2. Так как Петя, покупая на 3 бублика меньше, сможет купить на 1 пирожное больше, то цена 1 пирожного равна цене 3 бубликов. Тогда всего бубликов он купит 8 + 7 • 3 = 29. Ответ: 29. 3. Из признака делимости на 9 следует, что сумма стертых цифр должна быть равна 6. Так как больше то число, у которого цифр больше, то стирать надо две тройки. Останется число из 155
10 цифр. Чтобы это число было наибольшим, надо в старших разрядах иметь большие цифры, поэтому стираем 2 последние тройки. 4. На каждом из вырванных листов — две страницы. Номер одной из страниц — четное число, а другой — нечетное. Тогда в сумме всех номеров вырванных страниц будет 25 четных и 25 нечетных слагаемых. Поэтому сумма будет нечетной, а значит, она не может быть равна 2002. 5. Из условия задачи вытекает (см. рис. 61), что ААСК = = АВЕМ, поэтому Z.AKC = Z.BME иАК = ВМ. Тогда Z.MKD = = Z.KMD и AKMD будет равнобедренным. Поэтому KD = MD. Тогда АК + KD = ВМ + MD, а значит, AD = BD. D Вариант 10 1. Раскрыть скобки и привести подобные слагаемые. 2. х е {2,5; -6; 40}. 3. Обозначим ширину участка за х м, тогда площадь участка будет (Зх2) м2, а площадь, ограниченная оградой: Так как площадь, ограниченная оградой, на 128 м2 больше площади участка, то получим уравнение: 156
Решая его, находим х = 7, поэтому Зл: = 21; 21 м — длина участка. Ответ: 21 м. 4. а = 96; Ь= 120; с =168. 5.121„ = 1 • п2 + 2 • п1 + 1 = п2 + 2лг + 1 = (п + I)2, где п > 2. Вариант 11 1. Найдем сумму чисел -, -, ^, ^, -: 1 , 1 . 1 . 1 . 1 _ 459 3 4 5 6 7 420' 45Q :jf^r > 1, поэтому так жить нельзя. Ответ: нет, нельзя. о 2. Сложим кусок материи в ^ м пополам: имеем два куска о по \ метра. Сложив один из них пополам еще раз, получим три о куска по — м, — м и — м. Сумма длин первых двух равна — м. ODD Z 3. Рассмотрим сначала числа от 1 до 999. Добавим к ним число 0, а затем каждое из чисел дополним нулями слева до трехзначного числа: 000,001,002,..., 999. (При этом новых чисел не появится.) В этой записи имеются все возможные комбинации из трех цифр, следовательно, все цифры встречаются одинаковое число раз. Теперь рассмотрим числа от 1000 до 1999. Вычеркивая цифру «1» у всех, мы получим рассматриваемые выше числа: 000,001,..., 999. Так как в числах 2000, 2001, 2002 цифра «2» встречается 4 раза, а цифра «1» — 1 раз, то двоек использовано больше на три, чем единиц. Но при записи чисел от 1000 до 1999 цифра «1» использовалась больше, чем цифра «2» на 1000. Поэтому всего было использовано единиц больше на 1000-3 = 997. Ответ: единиц использовалось больше на 997. 4. Пусть в начале весны Аргус весил х кг, тогда к концу года он стал весить 0,75-1,2-0,9- 1,2л: = 0,972л:. Значит, Аргус потерял 0,028 своего веса. Ответ: похудел. 157
5. За 1 ч минутная стрелка проходит полный круг (360°), а часовая — в 12 раз меньше, то есть 30°. Значит, в 7 ч 00 мин минутная стрелка будет отставать от часовой стрелки на 210°. Че- 38 рез 38 минут минутная стрелка повернется на угол §^ • 360° = оО = 228°, а часовая на угол в 12 раз меньше, то есть на 19°. Тогда в 7 ч 38 мин угол между ними будет 210° 4-19° -228° = 1°. Ответ: 1°. Вариант 12 1. Так как длина ковровой дорожки складывается из горизонтальных и вертикальных отрезков, а сумма горизонтальных и вертикальных отрезков в обоих случаях будет равна соответственно длине основания и высоте лестницы, то дорожки хватит и для второй лестницы. 2. Обозначим за s (м) длину поезда, а за v (м/с) — скорость поезда. Используя первую часть предложения задачи, получим первое уравнение: ^ = 5. Так как с момента вхождения поезда на платформу до момента ухода с нее, «хвост» поезда проходит расстояние s 4 150 (м), то второе уравнение будет: -—у— = 15. Решая систему из данных двух уравнений, получаем, что s = 75 м, a v = 15 м/с. 3. Обозначим число 10а 4- ft, тогда получим по условию задачи уравнение: 10а 4- b = 6(а 4 Ъ) 4- 4, упростив которое, получим: 4(а—1) = ЬЪ. Так как аиЬ являются цифрами и число а—1 должно делиться на 5, то возможно 2 варианта: а = 1 или а = 6. В первом случае получится число 10 и тогда остаток (4) больше делителя (1), поэтому подходит только второй случай. Ответ: 64. 4. В 12.00 стрелки часов сходятся вместе. После этого за 20 минут минутная стрелка проходит ^ окружности, то есть о описывает угол в 120°. Часовая стрелка движется в 12 раз медленнее минутной (так как описывает круг за 12 часов). Поэтому она за 20 минут опишет угол в 120° : 12 = 10° и будет образовывать с минутной стрелкой угол в 120°—10° = 110°. 158
Вариант 13 1. Разделив на Б обе части равенства, получим АБ • А = 111. Так как 111 = 37 • 3, то А = 3, Б = 7. 2. Пусть П, Т, В — количество рыбы, которую поймали соответственно Петя, Толя и Ваня. По условию имеем два уравнения П + В = ЗТ,В + Т = 5П. Складывая их, получим В = Т + 2П. Если Петя поймал больше нуля (т. е. вообще у мальчиков был ненулевой улов), то В > Т 4- П. 3. Диагонали пятиугольника образуют замкнутый путь, при любом ходе фишки будут ходить как бы по «кругу», поэтому порядок следования их не изменится. А значит, позиции, когда одна фишка остается на месте, а две другие поменяются местами, не получить. 4. Треугольники ABC и CD A — равнобедренные и равны по трем сторонам. Поэтому /.ВСА = Z.CAD = ZACD. Значит, С А — биссектриса равнобедренного треугольника BCD, поэтому она является и высотой. Значит, отрезки АС и BD перпендикулярны. Проведем через точки А и С прямые, параллельные BD, a через точки BnD прямые, параллельные АС, получим прямоугольник, площадь которого в два раза больше площади ABCD. Поэтому Sabcd = \'BD AC = 75 (см2). Вариант 14 1. Так как периметр равен 16 см, то длина одной стороны квадрата будет равна 1 см, а значит, площадь 7 квадратов будет равна 7 см2. 2. Разобьем все числа, начиная с 2, на четверки. Всего четверок получится 501. Сумма чисел в каждой четверке равна 0. Тогда сумма всего выражения равна 1. 3. Пусть возрасты братьев в настоящий момент с и т. Старшему было столько, сколько младшему сейчас с-т лет назад. Тогда младшему было т-(с-т) = 2т—с. Так как старший был тогда в 3 раза старше младшего, то получаем уравнение т = 3(2/м-с), откуда Ът = Зс. Младшему будет столько лет, сколько старшему, через с—т лет. Тогда старшему брату будет 2с-т лет. Так как вместе им 60 лет, то получаем второе урав- 159
нение с + 2с—т = 60. Так как Зс = 5т, то из этого уравнения имеем Ът—т = 60, откуда т = 15. Следовательно, с = 25. Ответ: братьям 15 и 25 лет. 4. Применим несколько раз формулу для разности квадратов: а32-Ь32 = (а16 + &16)(а8 + &8)(а4 + Ь4)(а2 + Ь2)(а + Ь)(а-Ъ). Так как а-Ъ = 1 по условию, то получаем искомое тождество. 5. Закодируем карандаши тремя буквами, где первая буква будет означать цвет, вторая — длину, а третья — толщину. Тогда искомый набор может быть таким: АБА, ААБ, БАА, БББ. (Здесь БББ может означать: белый, самый длинный, самый толстый.) 8 класс Вариант 1 1в 5 • 2.1. 3. 48 км/ч. 4. График функции изображен на рис. 62. 5. ,1661 3148 4809 6. Рассмотрим ACKD (см. рис. 63). Так как СК и DK — биссектрисы внешних углов прямоугольника ABCD, то Z.KDC = = Z.KCD = 45°, a AKCD — равнобедренный и прямоугольный. Примем длины сторон СК и DK за с. Аналогично AN ВС, APAD, АМАВ являются равнобедренными и прямоугольными, причем AN ВС = APADy AKCD = = АМАВ. Обозначив длину NC за dy получим, что все стороны прямоугольника MNKP имеют длину с + dy поэтому MNKP является квадратом. Вариант 2 1.||1-2|-|4-8|-16| = 19. 2. Графиком уравнения являются 2 прямые, заданные уравнениями: у = Ои х = 1. 160
3. 1-3\/3 13' 1-Зх/З 1 + Зх/З 1-27 4. План построения (см. рис. 64). 1) Строим угол А. 2) Строим множество точек, находящихся на расстоянии, равном данной высоте от основания треугольника (прямую а). 3) Находим точку пересечения данного множества и второй стороны угла — В. Это будет вторая вершина треугольника. 4) Строим окружность с центром в вершине угла и радиусом в 2 раза большим данной медианы. Данная окружность пересечет построенную прямую а в некоторой точке М, принадлежащей внутренней области угла ВАС. 5) Соединяем данную точку М с вершиной угла А и делим полученный отрезок AM пополам. Полученную точку О соединяем со второй вершиной треугольника В, продолжаем прямую до пересечения с основанием треугольника и получаем третью вершину треугольника С. 5. Пусть такого класса в школе нет, т. е. во всех классах будет 33 и менее учащихся. Тогда во всей школе будет не более 33 • 30 = 990 учащихся, что противоречит условию задачи (в школе 1000 учащихся). Значит, наше предположение неверно, 161
\ в Рис. 64 Рис. 65 поэтому в школе есть класс, в котором не менее 34 учеников. Вариант 3 1. Так как Г + Г = Г, то Г = 0. Так как А + А = 0, тоА = 0 или 5, но 0 быть не может, так как Г = 0. Тогда А = 5, поэтому В = 1, значит Б = 2 и Д = 4. Ответ: ,5210 5210 10420 2. л:8 + х4 + 1 = х8 + х4 + 1 + х4-х4 = (л;4 + 1)2-(х2)2 = = (X4 + 1-Х2)(Х4 + 1+ X2) = (X4 + 1-Х2)(Х4 + 1 + X2 + Х2-Х2) = = (х4 + 1-х2)((х2 + 1)2-х2) = (л;4 + 1-*2)(х2 + 1-х)(х2 + 1 + х) = = (х4-*2 + Щх2-х + 1)(х2 + х + 1). 3. Допустим, что AM = 2AK (см. рис. 65). Тогда, так как АК = ? и отсюда 4 т. е. а = 0. Получим, что АВ = а = 0, чего не может быть. 4. При перестановке людей сохраняется четность номера места. Например, человек, стоящий вторым, может быть переставлен на четвертое, шестое, восьмое, ... две тысячи второе 162
место. Поэтому, если самый высокий человек стоит, например, вторым, то он никогда не станет первым. Ответ: не всегда. Вариант 4 1. Упрощая правую часть, имеем: у = х, где х ^ ±1. Таким образом, графиком указанной функции является прямая, заданная формулой у = х, без 2 точек: А(1; 1) и В(—1; -1). 3. В произведении всех чисел от 1 до 100 содержится 24 «пятерки»: по одной в числах 10, 20, 30, 40, 60, 70, 80, 90, 5, 15, 35, 45, 55, 65, 85, 95 и по две в числах 25, 50, 75, 100. Так как произведение цифры «5» на любое четное число оканчивается нулем, то произведение чисел от 1 до 100 оканчивается 24 нулями. Ответ: 24. 4. а) Пусть конфеты — это «кролики», а школьники — это «клетки». Так как 7 > 6, то по принципу Дирихле найдется школьник, который съел не менее 2 конфет, б) Нет, так как все конфеты мог съесть один школьник или один съесть 3, а второй — 4 конфеты. 5. Да. Пусть Z А — наибольший в А АВС (см. рис. 66). Проведем среднюю линию треугольника: MN. Она поделит высоту АН пополам. Загнем A ABC no MN, тогда А и Н совпадут. При этом: AM = HM,AN = HN. Тогда высоты треугольников ВМН и HNC: MD и NF будут и медианами, поэтому, загибая данные 163
треугольники по MD и NF, получим, что треугольники DMH и DM В; CNF и HNF совместятся. Вариант 5 х + 1 1. 2._1089708KL2 108 190809 _97 96 _108 108 0 3. Лев съедает за сутки ^ овцы, волк — ^ овцы, собака — ^ овцы. Тогда вместе за сутки они съедят ^4^4^ = 1 (овцу). о Zoo О т в е т: за один день. 4. Заметим, что х 4 2у 4 1 ^ 0, и упростим вид функции: х 4- Зу = х 4- 2у 4-1; получаем, что у = 1. Кроме того, л: + 2i/ +1 = = # + 2-1 + 1^ 0 или л: ^ —3. Строим график функции (рис. 67). Точка А(—3; 1) не принадлежит графику уравнения. У -з о Рис.67 5. Домножим числитель и знаменатель каждой дроби, стоящей в левой части равенства, на выражения, сопряженные знаменателям. В итоге получаем: у/2-1 ...+ v^-^ = v/l00-l Ответ: равенство верно. 164
6. План построения: 1) соединим точки А и Е; 2) отложим на прямой АЕ отрезок ЕВ = ЕА; 3) через точку F проведем прямую, параллельную прямой АВ; 4) отложим на данной прямой FC = АЕ и FD = АЕ; 5) точки А, В, С, D соединим отрезками. ABCD — искомый параллелограмм. Вариант 6 1 1\1\1\1\1\1\ 1 1 1 - WWW 20 32 42 56 72 90 110 132 5-6 67 7-8 I + ^ + ! + 1 - + + + ( + ( 8-9 910 1011 1112 U Ъ) V5 6 VII 127 4 12 6' 2.Z1 = |/А, Z3 = ±ZB (см. рис. 68). Zl + Z3 = 180°-125° = 55°, ZA+ZB = 55°-2 = 110°, ZC = 180°-110° = 70°. Ответ: ZC = 70°. Рис. 68 3. Так как ч/л*^ = |jc|, то задача совпадает с задачей 4 варианта 8 для 7 класса. 4. Последовательно освобождаемся от радикалов, возводя обе части в квадрат. В результате получаем: х = 49. 5. Приводя в правой части равенства дроби к общему знаменателю и учитывая, что знаменатели у дробей в левой и правой 165
частях равны, получим: Ъх + 31 = ах + 2а + Ьх—5х; Ъх + 31 = = (а 4- &)л: 4- (2а—5&). Откуда имеем: Решая данную систему, получаем: а = 8, & = —3. Ответ: при a = 8, Ъ = —3. Вариант 7 1. Выразим I/ через х: у = 20—2л:. График полученной зависимости — прямая. Из условия задачи следует, что хиу — натуральные числа. Поэтому график зависимости (см. рис. 69) состоит из 9 точек данной прямой, где хиу натуральные числа. 2. -—f = ——-— = 14- -^=7, поэтому исходное число с—4 с—4 с—4 будет целым, если 11 кратно с-4. 11 — простое число, значит, его делителями будут —11; —1; 1; 11. Решая 4 уравнения: с-4 = -11; с-4 = -1; с-4 = 1; с-4 = 11. Получаем с = -7; с = 3;с = 15. Ответ: -7; 3; 5; 15. 3. (х-у)(х 4 у) = 69 = 1 • 3 - 23 = 69 • 1 = 23 - 3; х > у. Тогда: или Решая данные системы, находим два решения: х = 35; у = 34 или х = 13, у = 10. 4. Треугольник с углами 60°, 30°, 90° (см. рис. 70). Треугольники: ААВС — прямоугольный, ААКС — остроугольный, АКВС — тупоугольный, ААКС — равносторонний, АСКВ — равнобедренный, А АС В — разносторонний. 5. (х2 4 х 4 1)(х2-х 4 1) — см. решение задачи 3 варианта 3 для 8 класса. 9 класс Вариант 1 1.44,4 : 4-4,4 : 4 = 10. 166
У\ к Рис.69 Рис. 70 2. С помощью прямоугольного треугольника, в котором один катет в 2 раза больше другого, строим угол в 30° так, чтобы он содержал внутри себя данный угол в 19° (см. рис. 71). Таким образом получаем угол в 11°. Теперь откладываем этот угол внутри данного угла и получаем угол в 8°. Деля полученный угол пополам (3 раза), получим угол в 1°, который и составляет — часть исходного угла. 1У Другое решение — см. задачу 8.16 (6 раздел). Рис. 71 3. Скорости автомобилей равны 50 км/ч и 40 км/ч. 167
4. Существует всего один способ, и число 2009007 является искомым. Если бы существовали другие способы, то числа отличались бы от этого числа, по крайней мере, на 2007, то есть первые четыре цифры не совпадали бы с 2009. 5. Пусть эти шестеро: А, В, С, £>, Е, М\ А находится в одном из двух отношений: «знаком» или «не знаком» хотя бы с тремя из них. Пусть это будут В, С, D. Если какие-то 2 из них находятся в том же отношении друг с другом, то они вместе с А образуют искомую тройку. В противном случае искомая тройка В, С, D. Вариант 2 1. Применяя формулу (х—у)(х + у) = х2-у2 последовательно для последних 2 множителей, в результате получим: При а = 2009 получим 1-а = -2008. Ответ: -2008. 2 ' *2 + 3*-10 (х + 5)(х-2) х-2 = х + 2, где х f -5 и х f 2. 3. Введем новые переменные: и = Зл: 4- у, и = х-у; решим систему уравнений относительно переменных и и и. Затем найдем хиу. Ответ: (3;-1); (-3; 1). л х2 + 5х-6 (х-1)(х + 6) ^ а , л 4. у = ——[—; у = -—-jti—> у = х + 6>где х * 1 (график — на рис. 72). 5. Вместе первый и второй мальчики купили пенал, 2 ластика и карандаш, заплатив 52 рубля за всю покупку. Так как третий мальчик заплатил 50 рублей за пенал, 2 тетради и карандаш, то ластик стоит дороже тетради на 1 рубль. Тогда так как пенал и ластик стоят 40 рублей, то пенал и тетрадь будут стоить 39 рублей. О т в е т: 39 рублей. 168
Рис. 72 Вариант 3 1. 103 [48 96 |2" 2. Так как сумма всех чисел от 1 до 12 равна 78, то сумма четырех чисел, записанных в вершинах каждого из прямоугольников и вершинах квадрата, будет равна 26. Возможный вариант записи может быть такой: 12, 9, 1,4 — в вершинах квадрата; 11, 8, 2, 5 и 10, 7, 3, 6 — в вершинах прямоугольников. 3. Найдем последнюю цифру выражений хг + хихг-х: х оканчивается на х3 оканчивается на хг + х оканчивается на х3—х оканчивается на 0 0 0 0 1 1 2 0 2 8 0 6 3 7 0 4 4 4 8 0 5 5 0 0 6 6 2 0 7 3 0 6 8 2 0 4 9 9 8 0 Из таблицы видно, что хотя бы одно из чисел хг + х и Xs—х делится на 10. 4. Больше будет закрытая часть, так как Si = S2, S3 = S4 (см. рис. 73). 5. Обозначим у = х + 1, тогда данное уравнение примет вид 169
Рис. 73 которое после упрощения примет вид: у4 + 6#2-40 = 0. Данное биквадратное уравнение имеет решения y\t2 = ±2. Следовательно, х\ = 1; Х2 = —3. Ответ: х\ = 1; Х2 = -3. Вариант 4 1. Антон — плавает, Борис — футболист, Владимир играет в теннис. 2. Обозначив за х пистолей стоимость лошади и учитывая, что при продаже было потеряно х%, имеем следующее уравнение: 24. х— X ■ X 100 Решая его, получаем х = 40 или х = 60. Ответ: лошадь купили за 40 или 60 пистолей. 3. Пусть х = 3g+ri, у = Зр+Г2, где г\ и Г2 остатки от деления на 3, то есть числа 0,1,2. Тогда х2 + у2 = (3g + ri)2 4- (Зд + г2)2 = = 3(3д2 + Зр2 + 2gri + 2рг2) + т\ + г|. Так как х2 + i/2 делится на 3, то г2 + г| тоже делится на 3. Но г\ + г| может принять значения 0, 1, 2, 4, 5 или 8; из них только 0 делится на 3, откуда г\ = г2 = 0. Следовательно, jchj/ делятся на 3. 4. Пусть ABCD — данный четырехугольник. Тогда (все данные приведены на рис. 74): Si &csin(1800-*) = ab sin x. be sin x 170
Поэтому сс со abed sin2 x oi • Ьз — ^2 • О4 j , что и требовалось доказать. 5. Пусть Х\ — общий корень данных трехчленов, тогда х\ + ах\ + 1 = 0 и х\ + х\ + а = О, то есть х\ + a#i + 1 = #2 + л:х + а <£► ал:х + 1 = jci + а & 1) = х\ — 1 4=> (х\ — 1)(а—1) = 0. Тогда а = 1 или х\ = 1. Если а = 1, то трехчлены оба имеют вид я2 + х + 1 и не имеют действительных корней. Если #i = 1, то I2 + а • 1 + 1 = 0 и I2 + 1 + а = 0. В обоих случаях а = -2. Ответ: а = -2. Вариант 5 х х = 12±2УЛ1 • 5 2. Домножаем числитель и знаменатель каждой дроби на выражение, сопряженное знаменателю дроби; упрощаем полученное выражение (см. задачу 5 варианта 5 для 8 класса). Ответ: \/2009-1. 171
3. S = ^ab • sin С и S = j(a2 4 b2). Выразим из данных Z 4 формул sin С: sin С = 1 + - Так как sin С < 1, то n , < 0. Но о , ^ 0, поэтому Zao Zao / = 0. Тогда sin С = 1, значит, Z.C = 90°. Так как а = ft, то = 45°. Ответ: ZC = 90°, ZA = ZB = 45°. 4. Обозначим первое число за дг, последнее за п + fe. Тогда сумма запишется следующим образом: п + (я 4- 1) + (п + 2) + ... 4- (п + fe) = 100. Найдя сумму членов арифметической прогрессии в левой части и упростив, получим: (2n+fe)(fe+l) = 200, 200 = 2100 = 1-200 = 4-50 = 5-40 = 8-25. Так как первый множитель больше второго и один множитель — четный, а второй — нечетный, то получим: или Решая данные системы, получим: п = 9, k = 7 или п = 18, fe = 4. О т в е т: 9 4 10 4 11 4 12 4 13 4 14 4 15 4 16 = 100 или 18 4 4 19 4 20421422 = 100. 5. Нет, так как 15 нечетных слагаемых в сумме дают нечетное число, а 50 — четное число. Вариант 6 1.(х2-х-1)-5-хг = 0 4* (х2-х-1)-4-хг-1 = 0 ^ & (*2-*-1-2)(*2-*-142)-(*341) = 0 & (х2-х-3)(х2-х+1)- -(х+1)(*2-*41) = 0 & (х2-х+1)(х2-2х-4) = 0& х2-х+1 = = 0;или*2-2л;-4 = 0. 172
Первое из данных уравнений корней не имеет, корнями второго уравнения будут: Х\д = 1 ± л/5. Ответ: х\^ = 1 ± \[Ъ. 2. 600 : 2 = 300 (км) — половина пути; 300 : 100 = 3 (ч) — время, затраченное на первую половину пути; 300 : 60 = 5 (ч) — время, затраченное на вторую половину пути; 3 4- 5 = 8 (ч) — время движения автомобиля; 600 : 8 = 75 (км/ч) — средняя скорость движения автомобиля. Ответ: 75 км/ч. A D Рис. 75 3. Так как Z.B > 90°, то Zl = Z2 (см. рис. 75). Но ВС || AD, АС — секущая, значит, Z.CAD = Z2. Так как Z3 ^ Z2 (иначе Z А = Z.Cy чего не может быть), то Z3 = Z.D. Но Z.D = Z А, поэтому Z3 = Z14 Z2, тогда Z3 = 2 • Z1 = 2 • Z2. В результате имеем: Z2 4 Z3 4 Z3 = 180°; Z2 + 2 • Z2 + 2 • Z2 = 5 • Z2 = 180°; откуда: Z2 = 36°. Тогда углы трапеции будут 72°, 108°, 108°, 72°. Ответ: 72° и 108°. 4. 45 • 513 = (210 • 510) • 53 = 1010 • 53 = 1 250 000 000 000. Ответ: 13 цифр. 5. Перепишем данное уравнение в виде у = х(у-1). При у = 1 решений нет. Пусть у ^ 1, тогда можно поделить обе части уравнения на у—1. Получим: FT = х ** у-1 =х^1 + ^г = х' 173
Так как х — целое число, то и —=-т — целое число. Но делителями единицы являются лишь числа 1 и -1. В итоге получаем: у = 2, х = 2 или у = 0, х = 0. 10 класс Вариант 1 2. у = у/2 + sin4 х- cos 2# + \/2 + cos4 л: + cos 2x, у = у/2 + sin4 #-1 + 2 sin2 # + %/2 + cos4 # + 2 cos2 л:—1, i/ = V(sin2x+1)2 + v^Ccos^cTIj, i/ = | sin2 jc + 1| + | cos2 л: 4- у = sin2 x + 1 + cos2 л: + 1, i/ = 3. График: прямая у = 3. 3. 27 9 3 1 54 18 6 2 108 36 12 4 216 72 24 8 4. Преобразуем левую часть: 2•2009 1 + 2 1 1 + 2 + 3 1 , 2 "* ■*■» 1 4 1 12 1 + 2 + 3--- + 2•2009 iii + ю + - 2009 , 1 , "*" 20 "*" '" 2009 2009 , 1 Г 1005- 2010 2009 •2009 I + I _ I + I _ I + + _i L 22 33 4 '" 2009 2010 2009 1- 2010. 2010 = 2010. 2010 Ответ: верно. 5. Рассмотрим ААМВ (см. рис. 76). КН — средняя линия, поэтому КН = \МВ = 8,5 (см). Проведем КЕ || AD, КЕСО — параллелограмм (КЕ \\ AD, AD || ВС), поэтому КЕ || ОС; КЕ = ОС = \AD. Применяя Li 174
271 теорему косинусов для ADMC и АЕМС, находим Z.DMC = и ЕС = 9,5 см. Значит, и КО = 9,5 см. Оставшиеся два расстояния от точки К до сторон AD и DC будут по 8,5 см и 9,5 см. Они находятся аналогично тому, как нашли КН и КО. Вариант 2 1. л:4-4л:3 + 12л:2-24л: + 24 < 0 & х4 - 4л:3 + 4л:2 - 4л:2 + + 12л:2 - 24л: + 24 < 0 & (х2-2х)2 + (8л:2-24л: + 24) < 0. Но (л:2-2л:)2 ^ 0 для любых л:, а 8л:2-24л: + 24 > 0 для любых л:, так как 8 > 0, а дискриминант — отрицательный. Следовательно, (л:2-2л:)2 + (8л:2-24л: + 24) > 0. Ответ: решений нет. 2. Воспользумся формулой для синуса двойного угла: sin х • cos х = ^ sin 2л:, ^ sin 2л: cos 2x = j sin 4л:, тогда получим уравнение: \ sin 4л: • cos 8x= \ sin 12л:, 4 4 175
равносильное уравнению: sin 4x • cos 8л: = sin 12л:, sin 4л: • cos 8л: = ^(эт(-4л:) 4- sin 12л:), ^-4л:) 4- sin 12л:) = sin 12л:, sin 12л: 4 sin 4л: = О, Lt 2 sin 8л: cos 4л: = 0. Тогда sin 8л: = 0 или cos 4л: = 0. Решениями этих уравнений будут х = ^ или х = £ + ^ где т е Z; n e Z. В итоге о о 4 получим л: = ^^, где т Е Z. о Ответ: ^,где/м е Z. о 3. Уравнение л:г/ = л:—*/ преобразуем к виду что равносильно совокупности следующих систем уравнений: \х + 1 = -1, Данные системы имеют решения: (—2; 2) и (0; 0). Ответ:(-2;2),(0;0). 4. Сложив все три уравнения системы, получим уравнение (х + у + z)(2x 4 2у 4- 2г) = 288, из которого найдем л: 4 у 4 г = 12 или х + у + z = -12. Подставляя вместо (л: 4 у 4 z) числа 12 и -12, получим в первом случае: х = 2, у = 4, z = 6, а во втором: х = -2,i/ = -4, г = -6. Ответ: (2;4;6); (-2; -4; -6). 5. Пусть ^АпВ : ^АтВ : ^ВрС = 2:5:17 (см. рис. 77). Положим ^АдгВ = 2л:; ^АтВ * 5л:; ^ВрС = 17л:. Тогда 2л: 4 5л: 4 17л: = 360°, откуда л: = 15°, значит, ^-АпВ = 30°; ^АтВ = 75°; ^ВрС = 225°. Поэтому углы треугольника ABC будут соответственно 15°; 37,5°; 127,5°. Из теоремы синусов следует, что АС = 2i?sinB = 2i?sin 37,5°, ВС = 2Д sinA = 2i?sin 127,5°. о АС ВС . п 2Д8т37,5°-2Д8т127,5о-8т15' Saabc = о— sine = о 176
Рис. 77 Так как sin37,5° • sin 127,5° = 0,5(cos90°- cos 165°) = 0,5cos 15°, TO Saabc = R2 cos 15° • sin 15° = Щ-. 4 p2 Ответ: SAAbc = ^r- 4 Вариант 3 1. Обозначив cos x = у, получим уравнение 21/2 4- у = 1, корнями которого будут — 1 и ^. Решая соответствующие уравне- ния cos л: = -1, cos л: = ^, получим х = тг 4- 2тгп; л: = ±^ 4 2тгА?; Z о n, fe G Z. Объединив решения, найдем л: = ^ 4 ^/, / Е Z. о о 2. Обозначив за л: дней — время, за которое второй рабочий смог бы выполнить всю работу один, получим уравнение: 7 • —Цт 4 5-^ = 1. Решая его, найдем время работы второго л: 4 4 х рабочего: 10 дней. Тогда время работы первого рабочего будет 14 дней. О т в е т: 14 и 10 дней. 3. График функции изображен на рис. 78. = f 2*2 4 3* 4 2, где х ^ 0, I 2*2-Зл; 4 2, где*<0. 177
у = 2х2+3\х\+2 О Рис. 78 4. Преобразуем исходное выражение: Это число будет рациональным, когда k является полным кубом. Итак, k= 133 = 2197. 5. Разложим —Зху на два слагаемых — ху и —2ху. Тогда получим: х2—ху—2ху + 2у2 = 7. Сгруппируем и вынесем за скобки (х—у) и получим: (х—у)(х—2у) = 7. Учитывая, что 7 = = 1-7 = 71 = ~1(—7) = -7 (-1), получим следующие четыре системы уравнений: (*-i/ = I, (*-i/ = 7, Гл:—у — —1, [х-у = -1у \x-2y = 7, \x-2y = 1, \*-2i/ = -7, \*-2i/ = -1. Решая данные системы, найдем решения уравнения: (-5;-6);(5;6);(13;6);(-13;-6). Ответ: (-5; -6); (5;6); (13;6); (-13; -6). Вариант 4 1. Перенесем 16 в левую часть неравенства и воспользуемся формулой разности квадратов: 2 24*-4) ^ 0 <* (х-2)2(х2-4х + 4) ^ 0. 178
Так как (х2-4л:-4) = (х-2-у/8)(х-2-\-у/8), то неравенство примет вид: (х-2)2(х-2-\/8)(х-2 4 \/8) ^ 0. Его решением будет: (-оо; 2->/8] U {2} U [2 + у/8; +оо). Ответ: (-оо; 2-\/8] U {2} U [2 + у/8; Н-оо). 2. Возведем обе части равенства х 4 — = 5 в квадрат: л:2 + 4 \ 4 2 = 25. Откуда *2 4 4т = 23. 3. Неравенство будет верно, если D < 0. Найдя дискриминант и учитывая, что он должен быть отрицателен, получим неравенство k2-6k 4- 8 < 0, которое будет иметь решения при 2 < k < 4, то есть при fe = 3. Ответ: k = 3. 4. Преобразуем данное уравнение к следующему виду: (*-2009)(i/-2009) = 20092. тогда х = у = 4018. 5. На 2010 треугольников разделить можно. План построения: 1) Отложите AM = ^ АВ (см. рис. 79). 2) Проведите MN, параллельную АС. 3) На АС отложите 1005 отрезков, равных AM. 4) Из точек Ai,A2, ...,Аюо4 проведите прямые, параллельные прямой АВ. 5) В полосе AMNC (см. рис. 79) получается 2009 треугольников, они все равносторонние и равны между собой, а 2010-й треугольник — это AMBN. Вариант 5 1. В случае, когда sin x < 0, числитель получается положительный и решений не будет. Если же sin х ^ 0, то 1-2 sin2 x = = 0. Решениями данного уравнения будут числа х = j 4 ^дг, п G Z. Учитывая, что л:(8—х) > 0 и sin* ^ 0, получаем ответ: т; %; % (так как тг < 3,2, то т^г < 7,2 < 8 и так как тг > 3, то 4 4 4 4 ^тг > 8,25 > 8). 4 179
M А А\ А2 Аюо4 С Треугольники АМА\, А\М\А29 ...,Аюо4^С — равносторонние Рис. 79 2. Обозначим за М, N, К, L середины отрезков АВ, CD, AC и BD (рис. 80). В треугольнике ABC отрезок МК является средней линией, поэтому МК \\ ВС и МК = \ВС. Аналогично, LN — средняя линия в треугольнике DBC> поэтому LN || ВС и LN = -zBC. Тогда MKNL — параллелограмм. Но так как KL = MN9 то MKNL — прямоугольник. Тогда угол между прямыми ВС и AD равен углу между параллельными им прямыми МК и ML, a Z.KML = 90°, значит, и угол между прямыми ВС и AD есть прямой. 3. Таккак*3+Зл;2+Зл;-3 = (*+1)3-4,тоу = (*+1)3-4. График же этой функции получается из графика функции у = Xs параллельным переносом. Но у графика функции у = хг есть центр симметрии — О(0,0), поэтому центр симметрии есть и у исходного графика, а именно в точке (-1, -4). 4. Любую сумму денег большую 10 рублей, можно составить из монет в 10 и 1 рубль. Четность числа монет можно по- 180
Рис. 80 менять с помощью размена 10 рублей на 2 монеты по 5 рублей. А сумма денег меньшая 11 рублей может быть представлена с помощью монет по 1 рублю, четность же легко поменять, заменив 2 монеты по 1 рублю на монету в 2 рубля. Таким образом, любая сумма денег, большая 1 рубля, может быть уплачена как четным, так и нечетным числом монет. 5. Перельем всю краску из первой банки в остальные. Затем перельем в первую банку по ^ остальных банок, тогда в о первой банке красок будет поровну. Перельем из второй банки все содержимое в третью и четвертую банки, а затем из них по половине банки обратно, тогда во второй банке красок будет поровну. Из третьей банки перельем все в четвертую, и там красок станет поровну. Вариант 6 1. Умножив обе части формулы на 2/г, получим: 2ап = 2S-(n-l)dn. Выразим S и получим 2a + d(n-l)^ О = z: П. 181
2. Так как sin4 a— cos4 a = (sin2 a+cos2 a) (sin2 a- cos2 а) = 1—2 cos2 а, a cos а = 4 -447 то найдя cos a = — —, получим sin а— cos a = — —. о Аи 3. Пусть #0— общий корень уравнений. Тогда х^+Ьхо = -1 иХц+Ьхц = -1. Разделим почленно второе уравнение на первое, тогда получим: 3 Сокращая в левой части уравнения дробь на Xq 4- &л:о, получим х0 = 1. Тогда Ь = -2. 4. Площадь трапеции ABCD можно найти, как сумму площадей 4 треугольников (см. рис. 81). Площадь каждого из треугольников можно найти по формуле S = -ab sin а. Обозначив ВО = х, получим 2 5— -. Так как л: — Рг/с. Si 5. Выразим л: через у: х = ^ целое, то у + 1 — делитель 11, то есть у может быть 0; —2; 10 или -12. Находя соответствующие значения х> получим следующие решения уравнения: (-6; 0), (16; -2), (4; 10), (6; —12). 182
Вариант 7 1. Так как левая часть уравнения при любом целом значении переменной х является четной, а правая — нечетной, то число 17 не является корнем данного уравнения. 2. Введем систему координат с центром в вершине квадрата А и единичными векторами, равными стороне квадрата (см. рис. 82). Тогда, учитывая, чтоХЙ = (1; -0,5), А$ = (0,5; -1), найдем cos /.MAN 0,5 + 0,5 AM AN = = 0,8. Тогда /MAN = arccosO,8. У А Рис. 82 3. Пусть дана арифметическая прогрессия, содержащая четное число членов: а\^а^а^^ ...,02л- Найдем суммы членов первой половины и второй половины: +ап а2п Учитывая,чтоап = а\+ dn—d, an+\ = a\+dn, о 2ai 4- Sdn-d ^ получим О2 = —f—ъ * п* Тогда Li S2S1 = f (2ai 4 3dn-d-2ai-dn + d 4. Так как 0 < sin | < 1, 0 < cos | < 1, то о о ctg ^ = cos ^ : sin - > cos ^ = а\ 4-2dn—d, = d-n2. 183
а а а а а и tg - = sin - : cos - > sin -. Поскольку - > £, то 0 0 0 0 О О Л Л f\ Таким образом, порядок чисел будет таким: cos ^ ctg ^, sin ^, 0 0 0 *!• 5. 2х2+у2 = 2ху+4х & х2-4х+4:у2-2ху+х2 = 4 <£> (х-2)2+ + (*-*/)2 = 4. Если сумма двух квадратов целых чисел равна 4, то один из квадратов равен 4, а другой — 0. Если х—2 = О, то х = 2 и (2-i/)2 = 4 «=> 2-1/ = ±2, что дает первые два ответа. Если (х-2)2 = 4, то х—2 = ±2 и х—у = 0, что дает третий и четвертый ответы. Таким образом, ответом будут следующие 4пары:(2;0),(2;4),(0;0),(4;4). Вариант 8 1. Так как (cosx)°v I— sin2 x = |cosa:| для всех значений переменной х> при которых cos* ^ 0 (х ^ ^ 4- тгдг, дг Е Z), то получим следующий график функции у = (cos л:)0 у/1 - sin2 x (см. рис. 83). — 7Г Рис. 83 2. Оценим приближенно число ^125-^0,2. Так как уТгб G (3;5), ^2 G (0,6; 0,7), то ^125-^2 е (2,3; 4,4). Учитывая, что в правой части стоит рациональное число, то и в левой части будет число рациональное. Из интервала (2,3; 4,4) только при х = 4 может получиться рациональное число в левой части. Подставив х = 4 в уравнение, получим, что х = 4 является его корнем. Ответ: х = 4. 184
3. Для нахождения радиуса круга воспользуемся тем, что £ = tg30° = ^ (см. рис. 84). Откуда г = 2\/3-2. Тогда 'круга = 8тг(2-л/3)(см2). Рис. 84 4. Величины углов образуют арифметическую прогрессию, поэтому обозначим эти углы в порядке возрастания: ху х + dy х + 2dy x + 3d, x + 4d. Так как сумма углов пятиугольника равна 540°, а больший угол не может быть больше 180°, то получим систему: Выражая из первого уравнения системы d = 54°-0,5л; и подставляя в неравенство, после упрощений получаем х > 36°. 5. Допустим, что (0; 0) — центр симметрии, тогда функция f(x) = х4 + ахг + Ъх2 + сх + d является нечетной. Но тогда f(x) = -f(-x) и х4 + Ъх2 + d = 0 является тождеством. Получили противоречие, значит точка (0; 0) не является центром симметрии. Вариант 9 1. Преобразуем уравнение к виду 12х + 5х = 13х. Разделим обе части уравнения на 13х и применим свойства степеней, тогда получим Щ 13 185
Корнем этого уравнения является х = 2. Других корней уравнение не имеет, так как в левой части функция убывает на всей числовой прямой. 2. Пусть в треугольнике ABC угол А равен 30°, а О — центр описанной окружности. Поскольку вписанный угол ВАС вдвое меньше центрального угла ВОСУ опирающегося на ту же дугу, то /.ВОС = 60°. Поскольку ОВ = ОС и /.ВОС = 60°, то треугольник ВОС — равносторонний. А это означает, что сторона ВС равна радиусу описанной окружности. Заметим, что сторону ВС можно было найти и по теореме синусов: ВС = 2flsin30°= R. Так как ВС < АВ + АС, то ^ < 4г + 4г> откуда получим: Л Л Л означает, что АВ+АС ВС< Li 3. Так как аиЬ — положительные числа, то а + Ъ > 0, поэтому аЪ + 1 < а + Ь. Перенося все выражения в левую часть и группируя, получим, (1—а)(1—Ь) < 0. Последнее же неравенство имеет 2 решения: или А это и требовалось доказать. 4. Преобразуем выражение: пг + 23/г = пг + Ъп + 18дг = = п(п2+5)+18п = А1((д12-1)+6)+18д1 = п(п+1)(п-1)+6п+18п = = (п-1)п(п + 1) 4- 24дг. Первое слагаемое делится на 2 и на 3, поэтому делится и на 6. Второе слагаемое также делится на 6, поэтому сумма делится на 6, а значит и выражение дг3 + 23п делится нацело на 6. 5. Пусть приобретается х, у, z путевок соответственно. Тогда 24000л: + 40000i/ + 60000z = 2000000 — уравнение с 3 переменными, a 27y+15x+45z = f(x,y,z) — исследуемая функция. Аналитическая модель f 24000л: + 400001/ + бООООг = 2000000, 186
не приводит к решению. Необходимо подойти к задаче иначе, то есть исходить из того, какова стоимость одного дня отдыха в каждом случае: 24000 1ЛПП 40000 14Й11 60000 = 160° = 1481 Так как 1333^ < 1481 ^ < 1600, то покупать сначала надо путевки на 45 дней, а затем на 27 дней. Первых можно купить 32 (32 • 60000 = 1920000 р.), а на оставшиеся 80000 р. купить путевки на 27 дней. Итак, 32 • 60000 + 2 • 40000 = 2000000. Вариант 10 1. Преобразуем уравнение таким образом, чтобы выделить куб суммы или разности двух выражений. Для этого умножим обе части на 3 и затем перенесем 1 в левую часть уравнения. Тогда получим уравнение: Зл:—Зх2—6л:3—1 = 0. Представим -6л:3 как сумму хг и -7л:3, тогда получим л:3-Зл:2 + Зл:-1-7л:3 = 0. Откуда (л:-1)3 = 7л:3. Извлекая корни кубические из обеих ча- 1 стеи уравнения и выражая х получим, х = —. 1-V7 2. Обозначим за/(л:) = ал:2+&л:+с.Таккак/(1) = a+b+с < 0, то график функции f(x) = ах2 + Ъх + с будет находиться ниже оси абсцисс, а значит, /(0) = с < 0. 3. Возведем обе части уравнений в квадрат и сложим, после упрощений получим уравнение 2 + 2 cos(a—/3) = а2 +1, из которого получаем а2 = 2 + 2 cos(a-/3). Учитывая, что cos(a—/3) < 1, получаем а2 < 3. Значит, а < л/3. 4. Треугольники ACD и ВСЕ равны (первый из них переходит во второй при повороте на 60° против часовой стрелки). Следовательно, отрезки СМ и СР, как медианы этих треугольников равны и угол между ними 60°. Значит треугольник СМР — равносторонний. 5. Победит Баба Яга с помощью следующей стратегии. Каждым своим ходом она уравнивает число грибов в кучках, имеющееся к ее ходу. 187
11 класс Вариант 1 l.i/= \/4sin4;t-2cos2;t + 3 4- \/4cos4;t4 2 cos 2л: + 3, у = у/4 sin4 x-2 4 4 sin2 л: 4- 3 4 >/4 cos4 л: 4 4 cos2 л: 4 1, I/ = у 4 sin4 х + 4 sin2 л: 4 1 4 \/4 cos4 л: 4 4 cos2 л: 4 1, I/ = 2 sin2 л: 4 14 2 cos2 л: 41, у = 4. Поэтому графиком функции будет прямая, заданная уравнением у = 4. 2. Найдем сумму квадратов корней уравнения: х\+ х\ = = (#1 4 x2)2-2;ti;t2 = (2-а)2 4 2(а 4 3) = а2-4а 4 4 4 2а 4 6 = = а2—2а 4 10 = (а—I)2 4 9. Значение данного выражения будет наименьшим при а = 1. При этом значении а дискриминант левой части уравнения положителен, поэтому корни существуют. Ответ: а = 1. 3. СО = \/10 см (см. рис. 85). В Рис. 85 4. Найдем производную функции f(a) = 2а 4 —-: 1 а1 188
при а е (0; 1). Значит, f(a) убывает на (0;1), а поэтому /(0) > > /(1), где /(1) = 3 т. е. 2а + \ > 3 при а е (0; 1). 5. Пусть DMNE — данная трапеция, а АВ — искомый отрезок (см. рис. 86). Тогда ADME и АСМА; ADNE и ABNC; AMNE и А АСЕ являются подобными; поэтому АС = Л1. CB = hi. AC _hx + h я Л ' я Л ' b h ' Из первых двух равенств следует, что АС = СВ; а из первого и третьего — что АС = -Щ-. Так как АВ = АС + СВ и АС = СВ а—о получим: АВ а—о (если а > Ь). Рис. 86 Вариант 2 2. Наибольшее значение равно 2-^=\ наименьшее равно 1. 3. у = x/tga; • vctg^c, i/ = \/tg# • ctgar, i/ = 1, где x ^ — • n, л Е Z. Учитывая область определения функции, построим график (см. рис. 87). 4. Данная фигура существует. Ее можно получить из 2 равных треугольников ABC и BCD, приложенных друг к другу по стороне ВС под некоторым углом (см. рис. 88). 189
Зтг 2 — 7Г 1 _ж о 2 Ч Рис.87 Зтг о 5тг Т 27Г Т 5. При указанной операции не меняется четность количества минусов. Поэтому последний знак — «+», если было написано четное число минусов, и « — », если — нечетное. Вариант 3 1. sin 9х + (| cos Зх + ^ sin 3*) = О <Ф 4* sin9* + cos (|-3*) = 0 4* 2cos (f§-6*) cos (j7j-3*) = 0; xi = -^-| fe; x2 = -|f-|^, где MGZ, 2. Построим указанную фигуру (см. рис. 89). -±x2-x} dx -2 з f(0, Ответ: 10-ttj kb« еД* 190
у=1,5х+3 Рис. 89 3. Выразим из первого и второго уравнений |x+2j/| и найдем 1 у из уравнения 2-У = у: получим у = 1. Тогда из уравнения \х + 2| = 1 находим Х\ = —1; х<ь = —3. 4. ПустьАВСО — данная трапеция (см. рис. 90),ВКиСМ — высоты. В Рис. 90 Тогда ; АК" = DM = 6 см; BD = VBK2 + KD2; ^. Отсюда R = ю| см. 4o о BD = 17 см; АВ = \/АК"2 + ВК2; АВ = 10 см; SabD = \AD ■ ВК; Sabd = J • 8 • 21 = 84 см2; R g Ответ: R = 10~ см. о 5. Преобразуем данное уравнение к виду (х+2у)2+(i/+1 )2 = = 0. Его решением будет пара (2; —1). Ответ:(2;-1). 191
Вариант 4 1. Имеем f ttcostt* = (-1)*£ + тг/г, fe G Z «» costt* = (-1)*^ + f J О О ID О Так как costtjc e [-1; 1], то найдем k. Решая неравенство 1 Qb получим fe = 0; 1; -1. Решая уравнение costtx = (-l)kzrp: + — 1U о при fe = 0; 1; -1, получим: а) при k = 0, х = ±^ arccos ^- 4- 2лг, б) при k = 1, л: = ±\ + 2/, / G Z, о в) npnfe = -1,* = ±|: arccos (-тт>) + 2р,р Е Z. Ответ: i^arccos ^г+2дг,±^+2/,/ Е Z,±^arccos ( — — 1 + -h 2p, n,i,p e z. 2. Надо вершины шестиугольника взять в серединах двух соседних сторон верхнего основания, двух противоположных сторонах нижнего основания. Полученное сечение будет правильным шестиугольником, так как его стороны будут равны половине диагонали грани, а углы все по 120°, так как его стороны параллельны сторонам равностороннего треугольника, состоящего из трех диагоналей граней. 3. См. рис. 91. ;, где* > 0, --, гдел;<0. 4.1 = [0F]; 2 = [0F+ >/SF]; 3 = [тг]; 4 = [тг + 0П; 5 = [тг^]; 6 = [тг + тг]; 7 = [тг + V5F] + [тг]; 8 = [(тг • Tr)-0F]; 9 = [(тг • тг)]; 10 = [0F] + [тг • тг]. 5. 6+66+666+...+666...6 = |(9+99+999... 9) = |((10-1)+ -Н10*-1> + _ | ( 192
I I I I I I О I I I I I I 1. а) Нули функции: Рис. 91 Вариант 5 -2±уД6 б) Промежутки монотонности: функция возрастает на (-оо; -2] и [1; +оо) убывает на [-2; 0) и (0; 1]. в) {/max = 2| при х = -2; ymin = -^ при х=1. Схематически график функции У= изображен на рис. 92. 193
2. | > 2; */| > л/2; | > >/2;2lo*23 = 3 > 2\/2 = 22/3;log23 > > ^ > \/2, значит log2 3 > л/2, что и требовалось доказать. Li 3. Так как а = 1-2&, то аЪ = (1-2Ь)Ь = -2Ь2 + Ъ = = -2&2 + &-I + I = -2 (b-i)2 + |, откудатах(а&) = |. I + I 4. л: > 2. 5. Дискриминант квадратного уравнения D = 4 sin2(xj/) - — 4^0, отсюда |sin(xj/)| = 1 и х = — sin(xy), x = ±1, у = Li Ответ: (±1; -| 4- 2nk\ k G Z. Вариант 6 1. Подбором найдем одно из решений уравнения: х = 1. Выражение в левой части уравнения определено только для х ^ 1 и является возрастающей функцией, поэтому при х > 1 оно будет больше постоянной величины справа. 2. Возможны два варианта: основаниями трапеции являются стороны АВ и CD или AD и ВС. Рассмотрим первый случай. Тогда должно выполняться A AM В ~ ACMD, откуда Ж = Ш и DM = М^М = 8. Во втором случае подобными треугольниками будут AMD и СВМ. Тогда откуда DM = 0,5. Ответ: 8 или 0,5. 3. Находя производную функции!/' = -3+2е~*иприравни- о вая ее нулю, получим критическую точку Xq = In —. Определяя о знак производной слева и справа от этой точки, получим, что Хо является точкой максимума. Найдем этот максимум: Утвх = -3 In f-2e"ln § « -3 In |-3 = -3 (in | + l) . 4. а) Раскрывая скобки, получим сумму целых слагаемых, б) Сгруппируем дроби в скобках парами: 1 4- , о + о<\по > •" и приведем их к общему знаменателю. Z Z00 3 194
Все числители будут делиться на 2005, а знаменатели «исчезнут» после раскрытия скобок. Поэтому указанное число будет делиться на 2005. 5. В качестве контрпримера рассмотрим многогранник, состоящий из куба и трех четырехугольных пирамид, основания которых можно приложить к граням куба разными способами. Значит, Саша мог склеить и другой многогранник. Вариант 7 1. Так как lim sin — не существует, то функция / в точке х = 0 терпит разрыв, следовательно, не является в этой точке дифференцируемой. Производная функции Ах sin ^- 1 g(x) = lim т—— = lim sin -£- Ах->о Ах дх->о Ал: не существует, поэтому в точке х = 0 нет производной. В част- ности, при Ах = -jf—^—г отношение -г-^- = 1, а при Ах = ~^- Ag Л отношение -г— = 0. Ах 2. Умножим обе части уравнения на выражение, сопряженное левой части уравнения. Получим уравнение /лО /л i О Сложив данное уравнение с исходным и упростив, получим уравнение 2\/1 + х + х2 = 2 + х. Возведя в квадрат обе части данного уравнения, получим х = 0. Выполнив проверку, убеждаемся, что х = 0 — единственный корень этого уравнения. 3. Воспользуемся геометрическим смыслом определенного интеграла. Изобразим график подынтегральной функции (см. рис. 93). Площадь фигуры под графиком изображенной функции равна 5, поэтому и з -з 195
-3 -2 -1 Рис. 93 4. Рассмотрим уравнение 2х2—2ух + (у2-3у) = 0. Его дискриминант равен 4(6у-у2). Он неотрицателен при 0^1/^6. Заметим, что исходное уравнение можно привести к виду (у—х)2 + х2 = Зу. Поскольку квадраты целых чисел при делении на 3 могут давать только остатки 0 и 1, числа у—х и х, а, значит, и сами числа х и у кратны 3, т.е. из соображений делимости, получаем, что х делится наЗи{/ делится на 3. Осталось перебрать случаи у = 09у = 39у = б.Ъ итоге получаем следующие решения: (0; 0), (0; 3), (3; 3), (3; 6). 5. Рассмотрим отношение: 20112009 20102010 20102010 /201П2010 V2010y = [1 + 2010/ ч 2010 1 1 -20Ц-1. Так как (l + 5) < 3, то ■ 1 А 2010У 2010 -1 2011 < 1. Значит, 20112009 < 20102010. Вариант 8 1. Применив симметрию относительно прямой i/ = -x, найдем площадь полученной новой фигуры, как 2 x2dx=l 196
2. 0<*< 1. 3. Учитывая, что х > О, после упрощений получим уравнение 2х • In х + х = О, решением которого будет х = -^=. 4. Обозначим радиусы нижнего и верхнего оснований конуса соответственно за г\ и г2, радиус шара — Д, искомый угол — а (см. рис. 94). Тогда, используя условие задачи, получим 2г1 + 2г2 = 5Д. (1) Так как углы АВО и ОВС равны (О — центр шара), то А АВО = = /.ОВС = §. Тогда rx = -Rctgf, г2 = -Rtgf. Учитывая это, уравнение (1) примет вид: которое приводится к виду: 2tg2 ^—5tg^ + 2 = 0. Из этого уравнения находим tg ^ = 2 или tg ^ = ^. Так как a = 2arctg2>|, то искомый угол будет а = 2 arctg -. Г2 5. В соответствии с условием задачи, составим систему уравнений: (а + Ь + с= 13лг, а + Ь + с= 13т. 197
Здесь a, b, с — цифры, при этом а^О; л = 1,2,..., 76; т = 1 или 2. Выразим а из второго уравнения и подставим в первое уравнение, после преобразований получим: 9(11а + &) = 13т(п— 1). Тогда 11а 4- b кратно 13. Рассматривая а = 1,2,..., 9, находим соответствующие значения для бис. В результате найдем все трехзначные числа, удовлетворяющие условию задачи: 247; 364; 481; 715; 832. Вариант 9 1. Рассмотрев 4 случая: а) х ^ 0, у > 0; б) х ^ 0, у < 0; в)л;<0,1/^0;г)л;<0,1/<0; получим следующий график уравнения (см. рис. 95) 2. Изобразив в прямоугольной системе координат подынтегральную функцию, и применяя геометрический смысл интеграла, устанавливаем, что 3. Используя условия равенства и перпендикулярность векторов, доказывается, что ABCD — прямоугольник. Искомая прямая проходит через точку С и середину АВ — точку К. (Координаты этих точек подставим в уравнение прямой). Далее находим площадь треугольника и трапеции. Они равны соответственно 12,5 и 37,5. 198
4. Так как (х2 + х)2 = \х2 + л:|2, то получим уравнение: Обозначим \х2 + х\ = t ^ 0. Тогда получим уравнение t2 + £—2 = = 0. Корнями данного уравнения будут t\ = 1, t<i = -2 (не подходит, так как £ ^ 0). Решая уравнение \х2 + #| = 1, получаем, что уравнение х2 + л: + 1 = 0 не имеет действительных корней, а уравнение х2 + х-1 = 0 имеет корни л: = ^ , которые являются и корнями исходного уравнения. 5. Распилим куб на 1000 кубиков со стороной jjr м. Тогда найдется кубик, в котором как минимум сидят 3 таракана. Вычислим радиус сферы, описанной вокруг такого кубика: г = — м. Так как — > -^-, то найдется сфера радиуса ^-, которая будет содержать кубик с 3 тараканами. Вариант 10 1. Подынтегральная функция не является непрерывной на [0; тг], поэтому применять формулу Ньютона-Лейбница нельзя. 2. Рассмотрим функцию f(x) = хь + хг + 1. Ее производная f(x) = 5л:4 4- Зл:2 > 0, то есть / — возрастающая функция. Поэтому, если она имеет действительный корень, то он единственный. Так как /(-1) = -1 < 0, а /(0) = 1 > 0, то на [-1; 0] существует единственный действительный корень этого уравнения. _^^ х + у х + 2 У + 2 г*, 3. Обозначим —-— = —з— = —л— = *• Тогда второе урав- Z о 4 нение системы примет вид: 4t2 + 9t2 H- 16t2 = 29. Решая данное уравнение, получим: t = ±1. Решая две системы при значениях t = 1 и t = -1, получим две тройки решений исходной системы уравнений: (0,5; 1,5; 2,5) и (-0,5; -1,5; -2,5). 4. Для решения задачи необходимо рассмотреть три случая. В двух случаях решение очевидно: О(0; 0);А(1; 0)иО(0; 0); А(—1; 0), где О — точка касания, а А — центр окружности (см. рис. 96, 97). 199
Рис. 97 Для нахождения координат центра окружности и координат точек касания в третьем случае обозначим центр окружности А(0, а), а точку касания М\(х^\ х%) (см. рис. 98). Координаты центра окружности и точек касания находим, решая систему уравнений: где у = f(x) и у = (р(х) являются уравнениями окружности и параболы соответственно. 200
Решением системы являются jcoi = 0, jco2 = ±^т>~- Находя соответствующие значе- La ния для а: а = 1 (при #oi = 0), а = - (при xq2 = ±-9-). При а = 1 и #oi = 0 получаем первый случай (рис. 96). Итак, окружность с центром в точке А (0; -), касается параболы в точках Mi ( -=-; j ) и М2 ( —^п~1 т ) (рис. 98). Таким образом, ответом будут: 1)О(0;0);А(1;0); 2)О(0;0);А(-1;0); 5. Преобразуем уравнение к виду: (H-jc2)(1-I-jc) = 2У, откуда 1 -I- х2 = 2т> где дп — целое неотрицательное число и1 + х = 2д, где п — целое неотрицательное число. Так как х = 2п—1, то х2 = (2"-1)2 = 22"-2 • 2" + 1. Учитывая, что 1 + х2 = 2ту получим 22л-2 • 2п Л- 2 = 2т. Разделив обе части уравнения на 2, получим: 22л-1—2п + 1 = 2m-1, которое равносильно уравнению 2Л(2Л~1-1) -f 1 = 2Ш-1. Так как т — целое неотрицательное число, то при т > 1 число 2т~1 является четным, поэтому 2Л(2Л~1-1) — число нечетное. А это невозможно. Значит, осталось рассмотреть т = 0 и т = 1. При т = 0 имеем 1 -I- я2 = 2°, откуда jc = 0, у = 0. А при т = 1 получим, что jc2 = 1 и, учитывая, что jc — число натуральное, получаем х = 1, у = 2. Таким образом, ответом будут: (0; 0), (1; 2). 201
Задачи для подготовки к математической олимпиаде (6 раздел) 5 класс 5.1. а) 99 + 99 : 99; б) 91 + 5742 : 638. 5.2. Рассуждая с конца, имеем: 2 • 7 = 14, 14 -I- б = 20, 20 : 4 = 5, 5 • 9 = 45, 45—5 = 40. Ваня задумал число 40. 5.3. Уменьшив втрое количество орехов в большей части, мы получим их столько же, сколько в 4 меньших частях. Значит, большая часть должна содержать в 3 • 4 = 12 (раз) больше орехов, чем меньшая, а общее число орехов должно быть в 13 раз больше, чем в меньшей части. Поэтому меньшая часть должна содержать 130 : 13 = 10 (орехов), а большая: 130-10 = = 120 (орехов). 5.4. 555 + 55 + 55 + 55 + 55 + 55 + 55 + 55 + 55 + 5. 5.5. Задача имеет различные способы решения. Например, для того, чтобы пройти через ворота с а яблоками, перед воротами у крестьянина должно быть 2(а +1) яблок. Поэтому перед последними воротами у него должно быть 4 яблока (а = 1), перед вторыми 10 яблок (а = 4), перед первыми 22 яблока (а = 10). Значит, надо взять 22 яблока. 5.6.12111 = 11000 + 1100 + 11. 5.7. Равносторонний треугольник дает угол в 60°, правильная четырехугольная пирамида, у которой все 8 ребер равны 1 спичке, дает в основании квадрат с углом 90°. Два равносторонних треугольника, имеющих общую сторону, дают угол в 120°. 5.8. 45. 5.9. 3 кг и 4 кг. 5.10. На 50, если разбить на пары все числа. 5.11. Перевернуть обои часы. Когда пройдет 3 минуты, в семиминутных часах останется 4 минуты. Поставить яйцо в данный момент вариться. Когда 4 минуты закончатся, перевернуть семиминутные часы обратно. Получим 4 + 7 = 11. 5.12.4. 5.13.6. 5.14. Иван поймал 21 рыбу, его сына звали Николай. 202
5.15.1957. 5.16. Петя и Ваня к шапке прибегут одновременно. 5.17. Дешевле обойдется совместная работа землекопов с 2 сторон. 5.18. За 30 секунд; за 10 секунд. Продавец может отсчитывать из пачки не 70, а 30 конвертов, тогда в пачке остается 70 конвертов. Аналогично и с 90 конвертами: отсчитать 10 конвертов. 5.19. Ошибка допущена в вынесении общего множителя за скобки в левой и правой частях тождества 4:4 = 5:5. 5.20. Да, если это было сказано 1 января, то 30 декабря Пете было 10 лет, 31 декабря исполнилось 11 лет. 31 декабря этого же года ему будет уже 12 лет, а в будущем году — 13. 5.21. (9999 : 9-999 : 9)((9 + 9): 9). 5.22. План действий должен быть такой: 1) сначала переправляются двое легких, 2) один из них перегоняет лодку обратно, 3) самый тяжелый садится в лодку и переплывает один, 4) второй легкий садится в лодку и перегоняет ее обратно, 5) двое легких садятся в лодку и переправляются на остров. 5.23. Так как каждый из поросят съел по 4 пирожка, то деньги все достанутся Нуф-Нуфу, который свои 4 пирожка отдал Наф-Нафу. 5.24. Общая сумма возрастов 11 игроков равна 11-22 = 242. После удаления игрока эта сумма стала 10-21 = 210. Тогда, найдя разницу, получим, что удаленному игроку 32 года. 5.25. С = 1, так как складываются 2 однозначных и 1 двузначное число. В = 8 или 9, так как в сумме получается трехзначное число. Рассмотрим оба случая. а) В = 8, значит, С — четное, но С = 1. Поэтому данный случай невозможен. 6) В = 9, значит, 2А = 11-9 или 2А = 21-9. А = 1 не удовлетворяет условию, значит, А = 6. Проверим: 6 + 99-1-6= 111 — верное равенство. 5.26. Митя, Толя, Сеня, Костя, Юра или Митя, Толя, Костя, Сеня, Юра. 203
5.27. Воспользуемся таблицей подсчета очков. Решено задач Не решено задач Набрано очков 6 0 42 5 1 32 4 2 22 3 3 12 2 4 2 1 5 -8 0 6 -18 В результате получим 3 задачи, 2 задачи, 5 задач. 5.28. Таких чисел 19: 5, 15, 25, 35, 45, 50, 51, 52, 53, 54, 55, 56, 57, 58, 59, 65, 75, 85, 95. 5.29. Имя девочки начинается с буквы В. 5.30. 6. 5.31. Вова в четвертом классе, Петя в третьем классе, Юра во втором классе, Коля в первом классе. 5.32. 6 кг. 5.33. Два арбуза разрезать пополам, а один — двумя разрезами на 4 части. Тогда выполним всего 4 разреза. 5.34. Через 7 лет. 5.35. 5. 5.36.102. 5.37. Наф-Нафу достался домик из камней. 5.38. Возможный вариант: первые 5 дней дежурят 30 богатырей по б в день; на шестой день дежурят оставшиеся 3 богатыря и первые 3. В следующие 5 дней дежурят оставшиеся 30 богатырей по б в день. 5.39. 45. 5.40. б апреля. 5.41.13. 5.42. 5. 5.43. См. рис. 99. 5.44. См. рис. 100. 5.45. У пяти учеников. 5.46.13. 5.47.16-1-18-15 = 19 (голубей) — наблюдали Малыш и Карлсон. 5.48. Да, так как 1000 х 1000 х 1000 = 1000 000 000 (мм) = 1000 (км). 5.49. См. рис. 101. 204
Рис. 99 Рис.100 Щ ill Рис.101 5.50. 3 333 333 : 3-333 333 : 3 = 1000 000 или 333 333-3 + 3:3 = 1000000. Нет, так как всегда будет число, кратное трем. 5.51.22. 5.52. Пересечением может быть точка, отрезок, треугольник, четырехугольник, пятиугольник, шестиугольник, семиугольник или пустое множество (см. рис. 102). 5.53. Почта — 1 — 3 — почта —7 — 1 — 2 — 3 — 4 — 5 — б — 7 — 5. Федор живет в доме № 5 (см. рис. 32 на стр. 106). 5.54.84 см2. 5.55. См. рис. 103. 5.56. В 5 раз. 5.57. ^115 Х 98 , 920 1035 11270 205
5.58. Так как сумма чисел увеличилась на 99-75 = 24, причем каждое из чисел увеличилось на 3, то всего чисел было 24 : 3 = 8. 5.59. 9: 21000, 20100, 20010, 20001, 12000, 10200, 10020, 10002,30000. 5.60. 5 способов, они изображены на рис. 104. Рис.104 5.61. 270 + (120 + 390): 3 • 5 = 1120. 5.62. V = 248-8 = 153б(см3). S = 4-8-24+288 = 89б(см2). 5.63. 3; 4; 5; 6; 7. 5.64. 9 мин. 5.65. От 32 до 20. 5.66. Сначала проконсультировать Борю, затем Аню и Свету. Тогда общее время нахождения учеников в кабинете составит 2x3+5x2+7= 23 (мин). 5.67. 62 + 55-17 и (3 + 22) • (3-2) • 4. 207
5.68. 16 чисел: 10000, 10001, 10010, 10011, 10100, 10101, 10110, 10111, 11000, 11001, 11010, 11011, 11100, 11101, 11110,11111. 5.69.1) Налить молоко в пятилитровый бидон и перелить в восьми литровый. 2) Снова налить молоко в пятилитровый бидон и долить восьми литровый бидон. Тогда в пяти литровом бидоне останется 2 л молока. 3) Вылить молоко в цистерну из восьмилитрового бидона. 4) Перелить 2 л молока из пятилитрового бидона в восьми литровый бидон. 5) Налить молоко в пятилитровый бидон и перелить его в восьми литровый. В результате в восьмилитровом бидоне получим 2-1-5 = 7 (л) молока. 5.70. Через 15 минут, так как половину пути от школы до дома старший брат проходит за 15, а младший — за 20 минут. 5.71. 6 12 = 72 (см). 5.72. 1) Поджарить первый и второй куски с одной стороны. 2) Перевернуть первый кусок, а второй кусок заменить третьим. 3) Обжаренный с 2 сторон первый кусок заменить вторым куском, обжаренным с одной стороны, и перевернуть третий кусок. 4) Обжарить второй и третий куски. 5.73. 8: четыре сверху и четыре снизу. 5.74. Муравей затратил на обратный путь времени больше, так как на первую половину обратного пути он затратил времени столько же, сколько и пешком. 5.75. Мать и сын. 5.76.111-11 = 100; 5-55-5-5= 100; 33 3 + 3 : 3 = 100. 5.77. Ваня шепнул: «Подскажи ответ», то есть прочитал слова не с начала, а с конца. 208
6 класс 6.1. За один год лошадь съест 12 возов сена, коза — б, а овца — 4 воза сена. Всего за год они вместе съедят 22 воза сена. г* Тогда один воз сена они съедят все вместе за 12 : 22 = — (месяца). Л9 14. 15 6-2-19'19- 6.3. У числа А =1-2-3 2001 • 2002 • 2003 на 5 делится 2000 : 5 = 400 чисел, из них на 25 делится 2000 : 25 = 80 чисел, на 125 делится 2000 : 125 = 16 чисел, на 625 делятся 3 числа. 400 + 80 + 16 + 3 = 499. Тогда число А делится на 5499. Так как среди множителей числа А содержится 1001 четных чисел, то А делится и на 2499, а значит, А делится на 10499, то есть оканчивается 499 нулями. 6.4. Так как мужчин не больше б, то самое большее их может быть 5. Значит, мужчины несут 10 буханок. Остается 2 буханки, поэтому 2 женщин не может быть, значит, женщин — 1, а детей — 6. 6.5. На девятый. 6.6. См. рис. 105. V 209
6.7. На рис. 106 показан ковер-самолет после того, как его испортил Змей Горыныч. На рис. 107 показан ковер-самолет, который разрезала Василиса Премудрая. На рис. 108 показан ковер-самолет, который сшила Василиса Премудрая. \S \\ Рис. 106 Рис.107 y/ // У/ 'A У/ 7/ // у, // v< У/ '/ Рис.108 6.8. В гулливерском спичечном коробке должно поместиться 12 лиллипутских коробков в ширину, 12 — в длину, 12 — в высоту. Всего 12 • 12 • 12 = 1728 (коробков). 6.9. Фигура В, смещенная вниз на \ длины флага и влево на о 2 полосы. В результате получается флаг, состоящий из 10 полос и имеющий ту же площадь (см. рис. 109). 6.10. 2; 2; 3. 6.11. Рассмотрим на плоскости треугольник, все стороны которого равны по 1 метру. По принципу Дирихле, из трех вершин треугольника две каких-то будут окрашены одинаково. 6.12. Будем первым ключом открывать по очереди все чемоданы. Если один из чемоданов открылся — прекрасно, от- 210
I ш А У/ I I 1 Рис.109 i ставляем в сторону этот чемодан с этим ключом. Если среди первых четырех чемоданов ни один не открылся, то этот ключ непременно будет соответствовать пятому чемодану. Таким образом, мы использовали не более четырех попыток. С оставшимися четырьмя чемоданами и четырьмя ключами поступаем аналогично. Таким образом, мы получим, что понадобится 4 + 3 + 2 + 1 = 10 (попыток). 6.13. В избушке живут Говорящие Коты, Мудрые Совы и Усатые Тараканы. Из того, что, кроме двух — остальные Говорящие Коты, значит, что Мудрых Сов и Усатых Тараканов вместе двое. Аналогично, из того, что, кроме двух, в избушке — остальные Мудрые Совы; Усатых Тараканов и Говорящих Котов — тоже двое. Эти два условия будут выполняться лишь в случаях: 1) Тараканов — 2, Сов и Котов — нет или 2) всех — по одному. Но 1-й случай не подходит, т. к. в условии сказано, что Совы и Коты живут в избушке. Поэтому у Бабы Яги поселились по 1 Говорящему Коту, Мудрой Сове и Усатому Таракану, т.е. всего 3. 6.14. Рассмотрим, что будет получаться в каждом из различных случаев. Если сорвать банан, на дереве снова вырастет банан. Если сорвать апельсин, снова вырастет апельсин. Т. е. 211
если срывать по одному плоду, ничего не меняется. Сорвем 2 банана, тогда на дереве будет 1 банан и 5 апельсинов, т. е. плодов стало на 1 меньше, бананов уменьшилось, апельсинов увеличилось. Больше по 2 банана не сорвать. Сорвем 2 апельсина, на дереве останется 3 банана и 3 апельсина. Сорвем банан и апельсин, тогда на дереве будет 3 банана и 3 апельсина. Таким образом, срывая по 2 плода, мы получаем, что число плодов уменьшится на 1, причем число бананов остается все время нечетным. Можно предложить такой вариант для ответа на 1 вопрос: срывать 4 раза по банану и апельсину вместе, в итоге останется лишь 1 банан. Так как на яблоне всегда остается один плод, то не сделать так, чтобы ничего не осталось. 6.15. Представим число 121 в виде произведения двух сомножителей: 121 = 1 • 121 = 121 1 = 11- 11. Так как ребятне может быть по 1 и рыбок по 1, то на рыбалку ходили 11 ребят, которые поймали все по 11 рыбок. 6.16. Изобразим путь пассажира отрезками (см. рис. 110). A BCD Рис.110 Обозначим за S длину отрезка CD, тогда ВС = 2S (пока спал), всего BD = 3S, но АВ = BD, значит, AD = 6S. Бодрствовал он на АВ и CD, АВ + CD = SS + S = 4S. Пассажир бодрствовал — = - = - (пути). 6.17. Сметаны в банке с вареньем окажется столько же, сколько варенья в стакане со сметаной. 6.18. Да, так как 5 руб—1 руб 80 коп = 3 руб 20 коп = 320 коп; а 320 не делится на б нацело. 6.19. Так как число учеников, получивших ту или иную оценку, всегда натуральное, то для решения задачи надо найти такое натуральное число, меньше 30, одновременно делящееся на 5, 4 и 2. Возможным единственным ответом является число 20. Тогда в классе получили «пятерки» — 4 ученика, 212
«четверки» — 10 учеников, «тройки» — 5 учеников. Значит, двойку получил 1 ученик. 6.20. Последняя цифра уменьшаемого оканчивается на 1, поэтому разность оканчивается 0. Значит, число делится на 10. 6.21. Так как за решение каждой задачи девочки получали вместе б конфет, значит, сумма всех полученных ими конфет должна обязательно делиться на б, но 33 на б нацело не делится. Значит, девочки Витю обманули. 6.22. За 2 часа пассажирский поезд пройдет 108 км, разность скоростей поездов 18 км/ч, значит, скорый поезд догонит пассажирский поезд за 108 : 18 = б (ч). Расстояние от станции А будет 6 • 72 = 432 (км). 6.23. Так как Дима и папа двигались со скоростью каждый по 1,5 м/с, то скорость сближения будет равна 3 м/с и они встретятся через 300 м : 3 м/с = 100 с. Т. к. скорость собаки 5 м/с, то за 100 с она пробежит 500 м. 6.24. Из условия задачи следует, что Витя купил пирожков больше всех, значит, больше, чем третью часть от 14, т. е. 5 или больше. Так как число пирожков у него в 2 раза больше, чем у Коли, то это может быть б, 8, 10, 12, 14. Проверим. Возьмем б, тогда Коля купил 3 пирожка, а Женя — 5. Эти числа удовлетворяют условию задачи. Возьмем 8, тогда Коля купил 4, Женя — 2, что меньше Коли. Поэтому данный вариант не подходит. Аналогично можно доказать, что других вариантов нет. 6.25. Спереди — К, справа — А, сверху — Ф (см. рис. 111). Рис.111 6.26. Найдя значения знаменателей и сократив первую дробь на б б, а вторую на 7-7, получим: 111 1112-111 1112 = 0. 213
6.27. Так как у числа есть треть и семнадцатая часть, то оно делится на 51, т. е. имеет вид 51л:. Тогда треть его будет 17л:, а семнадцатая часть — Зл:. По условию задачи составим уравнение: 17л: = Зхр + 100. Выразим х: х = ~ ^ - Учитывая, что хир натуральные, подбором найдемр = 5. Тогда х = 50. В итоге получим, что число будет 2550. 6.28. 2002 натуральных чисел могут быть все четными; все нечетными; или четными и нечетными. Два первых случая не могут быть, так как сумма — нечетное число. Значит, среди 2002 натуральных чисел есть четные и нечетные числа, поэтому произведение будет числом четным. 6.29. Разрежем ковер на 16 ковриков размером 1x1 метр. Так как 16 > 15, то один из ковриков будет без дыр. 6.30.15. 6.31.1. 6.32. -90-89-88- ... -1 + 0 + 1 + 2 + ... + 98 + 99 + 100 = = (91+ 100)-5 = 191-5 = 955. 6.33. 75 га. 6.34. 48 км/ч. 7 11 6.35. Так как 7:12=т^ = ± + ^,то надо разделить 3 ябло- \Л 4 о ка на 4 части, а 4 яблока каждое на 3 части и каждому человеку дать — по — и яблока. 6.36. В шестеричной системе счисления, так как 4(6 -I- 3) = 2 = 102. 6.37. Пусть п — число сторожей в бригаде, к — число бригад, т — число ночей, которое проспал один сторож. Тогда тпк = 1001. Так как 1001 = 7 • 11 • 13 и п< т < к, то п = 7, дп=11,й = 13. Значит, сторожей в бригаде будет 7. 6.38. Так как половина яблок составляет треть объема банки, то половина оставшихся яблок составляет шестую часть 12 1 первоначального объема. Так как - от - будет составлять ±, о о 4 то уровень компота понизится на четверть. 6.39. См. рис. 112. 214
\ \ \ / / \ \ \ Рис Л12 6.40. Обозначим длину стороны самого большого квадрата за х. Тогда, двигаясь от большого квадрата по часовой стрелке, последовательно выразим через х стороны других квадратов: х—1, х-2, х-3. Из равенства верхней и нижней сторон прямоугольника получаем уравнение х + (jc-1) = (х-2) + (х-3) + (х-3). Корнем данного уравнения будет х = 7. 6.41. Пронумеруем борцов в соответствии с их силой (больший номер побеждает меньший). Интуитивно ясно, что если такие команды существуют, то их общая сила примерно равна. Разобьем борцов сначала на три группы (1, 2,3); (4, 5,6); (7,8,9) и в каждую команду направим по одному представителю от каждой группы. Тогда команды (1, 5,9); (3,4,8); (2,6, 7) будут удовлетворять условию задачи. 7 класс 7.1. Пусть такие 20 учащихся найдутся. Тогда сумма возрастов оставшихся 5 учеников будет не больше 10 лет. То есть средний возраст этих «учеников» должен быть не больше 2 лет, что противоречит возрасту учащегося школы (меньше 5 лет возраст ученика быть не может). Таким образом, таких учащихся не найдется. 7.2. а) См. задачу 4 варианта 1 для 8 класса. 0, если х ^ 0, б) у = \x\-x = еслил:<0 (график на рис. 113). 215
Рис.113 7.3. Так как а = Ъ • q + г, то За = ЗЬ • q + 3 • г. Значит, остаток увеличился в 3 раза, а частное не изменилось. Например, 5 = 3-1 + 2, 5-3 = 15, 3-3 = 9, 15 = 91 + 6 — изменился остаток. 7.4. К концу первых суток улитка поднимется на 1 м, к середине вторых суток — на б м, к середине 3-х — на 7 м..., к середине 5-х — на 9 м, к середине б-х — на 10 м. Таким образом, ответом будет 5,5 суток. 7.5. Условию отвечает положение стрелок в 3 часа, в 9 часов, в 15 часов, в 21 час. Также условие будет сохраняться через поворот часовой стрелки по циферблату на jj полного угла. Таким образом, всего за сутки таких положений будет 11-4 = 44. 7.6. а = 2, Ъ = 9. Другие варианты не подходят. 7.7.11. 7.8. Надо взять 21 трубу длиной по 7 м и 4 трубы длиной по 5 м. 7.9. Колодец надо вырыть в центре окружности, проходящей через три указанные точки. 7.10. Василиса Прекрасная не может быть в первой темнице, значит она во второй или третьей темнице. Так как темница 3 — пустая, то Василиса Прекрасная будет во второй темнице. 7.11.13 + 132 + 133 + 134 +... +132001 +132002 = 13(1 +13) + + 133(1 + 13) + ... + 132001(1 + 13) = 14(13 + 133 + ... + 132001). Так как 14 делится на 7, то и само число делится нацело на 7. 7.12. а) 4 • 12 + 18 : (б + 3); б) (4 12 + 18): (б + 3); в) 4(12 + 18: 6 +3). 216
7.13. a) 376 45 , 1880 1504 б) V239 X 54 , 956 1195 16920 12906 7.14. Докажем задачу методом от противного. Пусть в каждой конюшне находится четное число лошадей, тогда сумма четных чисел — число четное. А по условию всего лошадей 101 — число нечетное. Таким образом, получили противоречие. Значит, хотя бы в одной конюшне будет нечетное число лошадей. 7.15. Можно, см. рис. 114. Рис.114 7.16. Пусть ежегодно выпуск продукции снижался на х %. Примем первоначальный объем продукции за 1. Тогда через год продукции будет выпущено 1~twx> a через два года — х \2 51 —ттЬ\) • По условию это число равно 1 —-г^т> = 0,49, откуда 100/ 100 7.17. Обозначим число подъездов в доме, квартир на этаже и число этажей соответственно за х, у, г. Разложим 60 на простые множители: 60 = 2 • 2 • 3 • 5. Так как подъездов больше одного и меньше, чем число квартир и число этажей, то их может быть 2 или 3. Составляя произведение из трех множителей, каждый из которых больше предыдущего, а произведение равно 60, получим следующие варианты: 60 = 2-310 = 2-56 = 3-4-5. А это означает, что в доме может быть 5, б или 10 этажей. 217
7.18. Разложим 1995 на множители: 1995 = 3 • 5 • 7 • 19. Так как искомое число не может быть ни однозначным, ни трехзначным, то оно является двузначным. Рассматривая возможные варианты для двузначного числа, получаем ответ: 57-5-7 = 1995. 7.19. Так как один сапфир и два топаза равны по стоимости 3 изумрудам, то 8 сапфиров и 16 топазов будут равноценны 24 изумрудам. Так как семь сапфиров и топаз равны по стоимости 8 изумрудам, то 21 сапфир и 3 топаза будут равноценны тем же 24 изумрудам. А тогда, получается, что 13 сапфиров равноценны 13 топазам. А это означает, что сапфир и топаз равноценны. 7.20. Обозначим сторону самого большого квадрата зал:, тогда, двигаясь от большого квадрата по часовой стрелке, последовательно выразим через х стороны других квадратов: х-1> х- 2, х- 3. Обозначив сторону искомого квадрата за у, получим два выражения для длины верхней стороны фигуры: х + х-1 = = у + х - 2 + х-З.Из данного равенства находим у = 4. 7.21. Разность между числом и суммой его цифр всегда делится на 9. Значит, все числа, кроме исходного, должны делиться на 9. Тогда, 0 получится из 9, 9 из 18, 18 из 27 и т. д. В результате получим такую цепочку: 0 <- 9 <- 18 <- 27 <- 36 <- 45 <- 54 <- 63 <- 72 <- 81. Число 81 может получиться из 90 или 99. Но 90 ни из какого числа не получить. Поэтому следующим числом в цепочке будет 99, которое может получиться из 100, 101, 102,..., 109. Таким образом, начинали вычитания с любого числа от 100 до 109. 7.22. Да, например, п = 10, (23 + 25)2 = 26 + 2 • 28 + 210 = = 26 + 29 + 210. 7.23. Выполним дополнительное построение: отложим на стороне АВ отрезок BD = ВС и соединим точки D и С (см. рис. 115). Тогда треугольник BCD будет равнобедренным. Поэтому углы DCB и CDB будут равны 80°. Пусть СЕ — биссектриса треугольника ABC. Так как Z.C = 120°, то /LACE = 60°. Поэтому Z АЕС = 80°. Таким образом, в треугольнике DEC два угла равны 80°, поэтому он будет равнобедренным. Тогда угол 218
при вершине С данного треугольника будет равен 20°, поэтому ZACD = 60°-20° = 40°. Значит, треугольникАС!) также является равнобедренным, поэтому СЕ = CD = AD = AB-BC = 4. A DE В Рис.115 7.24. Обозначим за х часов — время горения свечей. Тогда сгорело j толстой свечи и ^ тонкой свечи. Так как огарок толстой свечи длиннее огарка тонкой свечи в 3 раза, то получим уравнение: Решением данного уравнения будет jc = 1,6 (часа). Таким образом, свечи горели 1 час 36 минут. 7.25. Определим наименьшее и наибольшее число городов в данном государстве. Если в государстве 4 города, то, соединяя все города, получим б дорог. При меньшем числе городов дорог будет еще меньше. Таким образом, наименьшее возможное число городов может быть 5. Теперь оценим число городов сверху. Так как каждая дорога соединяет 2 города, а дорог — 7, то больше 14 городов быть не может. Таким образом, в этом государстве может быть от 5 до 14 городов. Возможные расположения дорог между городами показаны на рис. 116) (цифрами обозначены города). 7.26. Так как учеников всего пять, а цветов маек — три, то какой-то один ученик будет носить майку такого цвета, который не носят другие ученики. Этим учеником не может быть ни Саша, ни Гриша, ни Витя, так как среди оставшихся будут ученики в красной, синей и зеленой майках. Значит, ученик, который носит майку, которой нет у других учеников — Дима или Миша. Значит, у Димы и Миши майки разных цветов. 7.27. Обозначим числа, записанные в вершинах за х, у, z. Тогда учитывая условие, получим уравнение x + y + z + xy + xz + yz + xyz = 1000. 219
12 3 4 5 6 7 8 12 3 4 5 6 7 8 Oil 8 10 9 10 1 • ••••• i i 99999 ill 12345679 12345678 9 10 1112 L2 13 9 10 11 rtn III l ггз Шп 12 3 4 5 6 7 8 9 10 11 12 13 14 Рис.116 Прибавим по 1 к левой и правой частям полученного уравнения и сгруппируем слагаемые в левой части, тогда получим: (1 + х) + (у + ху) + (z + xz) + (yz + xyz) = 1001. Преобразуем левую часть уравнения: (1 + х) + (у + ху) + (2 + xz) + (yz + xyz) = Так как 1001 = 7 • 11 • 13 и ни одна из скобок не равна 1, то получаем, что числа х, у, z будут равны б, 10 и 12. 7.28. Пусть ABC — данный треугольник, ZA = 30°, ZB = = 15°, CD — его медиана (см. рис. 117). Выполним дополнительное построение: построим /.ABE = 30°. Тогда треугольник ABE будет равнобедренный с основанием АВ. В этом треугольнике медиана ED будет являться высотой и биссектрисой. Поэтому /DEB = ZDEA = 60°. Продолжим отрезок BE за точку 220
Е, тогда ЕА — биссектриса угла FED. Следовательно, точка С — точка пересечения биссектрисы внешнего угла FED и биссектрисы угла ABE, а значит, она равноудалена от прямых EF, DE и AD. Следовательно, точка С лежит на биссектрисе внешнего угла ADE. Так как угол ADE — прямой, то Z.CDE = = \LADE = 45°. Поэтому ZCDA = 45°. La 7.29. Используя условие, получим: (10А + М)(10Э + М) = (ЮМ + Э)(10М + А). Раскроем скобки: 100АЭ + ЮАМ + ЮМЭ + М2 = 100М2 + 10ЭМ + ЮМА + ЭА. После упрощения получаем: 99АЭ = 99М2. Так как двузначное число не может начинаться с нуля, то А ^ 0 и М ^ 0. Разделим обе части равенства на 99АЭ, получим 1 = -т-^, откуда следует равенство, которое требовалось доказать. 7.30. Рассмотрим прямоугольник ABCD и четыре треугольника ABC, BCD, CDA и DAB, которые удовлетворяют условиям задачи (см. рис. 118). Тем не менее, одной, общей вершины для всех четырех треугольников не существует. Поэтому неверно. В A D Рис.118 221
7.31. За б часов с момента начала боя часовая стрелка пройдет половину циферблата и окажется между 16 и 17 часами, а минутная стрелка обойдет циферблат б раз, поэтому угол между ними изменится на 180°. Тогда часовая и минутная стрелки совпадут. А так как между 16 и 17 часами дважды стрелки совпасть не могут, то математический бой продолжался ровно б часов. Замечание. Задачу можно решить, составив уравнение. 7.32. На месте последних цифр могут быть цифры 1, 2,..., 9, всего 9 цифр. Так как Вася забыл 2 последние цифры, то всего вариантов набора номера может быть 9-9 = 81. 7.33. Пусть утверждение 2) — ложно. Тогда утверждения 1) и 3) будут оба истинными, что невозможно (четное число не может быть простым). Значит, утверждение 2) — истинно. Поэтому это число 7. 7.34. Выиграет второй. Он должен брать каждый раз столько конфет, чтобы сумма числа конфет, взятых первым семиклассником и им, равнялась 5. 7.35. Так как 1002 > 50 • 150, то первое число больше. 8 класс 8.1. Возможный план построения: 1) Строим вспомогательный треугольник АОО\ такой, что АО = |me, AOi = |дпь, ООг = |дпс. 2) Проведем медиану из вершины А, удвоим ее и получим АВ — основание исходного треугольника. 3) Удвоим отрезок О\О, получим О\С. Точка С будет третьей вершиной треугольника ABC. 8.2. Один из возможных вариантов построения: 1) Строим биссектрисы двух других углов треугольника. Полученная точка их пересечения будет первой точкой искомой биссектрисы. 2) Берем любую точку на одной из сторон острого угла треугольника и соединяем ее отрезком с противолежащей вершиной треугольника. У нового треугольника так- 222
же строим точку пересечения биссектрис. Данная точка принадлежит биссектрисе острого угла. 3) Соединяем полученные две точки прямой, которая является биссектрисой острого угла. 8.3. Необходимо поставить знак «=». 8.4. Пусть А АВС разрезан на 2 равных треугольника (см. рис. 119). Тогда Z ADB в AADB должен быть равен одному из В D С Рис А19 углов ЛАОС. Но ZADB не может равняться /LACD или ZCAD, так как внешний угол треугольника всегда больше внутреннего угла треугольника, не смежного с ним. Если же Z ADB = = ZADC, то ZADB = ZADC = 90°, значит, AD является высотой. Но в равных треугольниках против равных углов лежат равные стороны, значит, АВ = АС, что противоречит тому, что Л АВС разносторонний. Поэтому разносторонний треугольник разрезать на 2 равных треугольника нельзя. 8.5. Тремя способами: проведя 3 медианы (высоты, биссектрисы). 8.6. Пусть А = y/k+ 1 + Vk-1,B = 2\/й(гдей = 2010). Тогда А2-В2 = 2k + 2y/k2-l-4k = 2 U/k2-l-k\ < 0. Поэтому А < В. 8.7. Стрелки совпадут снова в 1 ч 5уу мин. Возможный вариант решения может быть такой. До совпадения часовая стрелка опишет угол х градусов, а минутная — (х + 360) градусов. Так как минутная стрелка движется в 12 раз быстрее часовой стрелки, то получим уравнение 12л: = х + 360, из ко- 223
о торого находим х = 32^-. Переведя угол в часы и минуты, получим 1 ч 5— мин. 8.8. Не всегда (см. рис. 120). Рис.120 8.9. Из условия следует, что 20 черных коров и 15 рыжих дают за 1 день столько же молока, что и 12 черных и 20 рыжих коров. Тогда 8 черных коров дают столько же молока, сколько 5 рыжих. Поэтому у рыжих коров удои больше. 8.10. Так как нас интересует последняя цифра результата, то определим последнюю цифру числа 92011. Число 9 при возведении в степень дает 2 варианта последней цифры — 9 (если показатель степени нечетный) и 1 (если показатель степени четный). Так как 2011 — число нечетное, то 92011 оканчивается цифрой 9. 8.11. Объединяя 2 человек, сидящих друг напротив друга в пары, получим 10 пар. Так как юношей больше 10, то в одной из пар будут только юноши. 8.12. Если в каждый месяц родилось не более 3 учеников, то всего учеников будет не больше 36. А по условию их 37, значит, такого быть не может. Поэтому найдется 4 ученика, отмечающиеся день рождения в один месяц. 8.13. Черноволосой не может быть отличница, так как отличница подтвердила слова черноволосой. Поэтому черные волосы у Рыжовой. Тогда Белова будет иметь рыжие волосы, а Чернова — белые. 224
• И) ■ (Ч) ■ (1-& -J.^ Л ^JL^ =132435 141б = 11б = _8_ 15/Л 15/ 2 2 3 3 4 4 '" 15 15 2 15 15* 8.15. Находя значения степени 81, 82, 83, 84, 85 и т. д., замечаем закономерность: последней цифрой являются 8, 4, 2, 6, а далее они повторяются. Так как 2011 = 502 • 4 + 3, то 82011 оканчивается той же цифрой, что и 83, то есть 2. 8.16. Так как 1 = 19- 19-360, то для построения угла 1° надо 19 раз отложить угол 19° таким образом, чтобы все время одна из сторон у 2 соседних углов была общей. При девятнадцатом откладывании первый и девятнадцатый углы в пересечении дадут 1°. 8.17. Через I2 + 22 + З2 + ... + 132 = 819 (дней) Кощей зарыл клад на глубину 13 м. Зарыть клад на глубину 14 м он не успел, так как 819 + 142 = 1015, а 1015 > 1001. Но отрыть клад и выложить его на землю Кощей успел, так как 819 -I-132 = 988 и 988 < 1001. Значит, клад будет на глубине 0 м, то есть на земле. 8.18. а) п2 + п + 1 = п(п + 1) + 1. Так как п(п + 1) — число четное, то п(п + 1) -I-1 — будет нечетным числом; б) Ближайшие к числу п2 + п +1 квадраты находятся среди натуральных чисел п2 и (п + I)2, но п2 < п2 + п+ 1 <(п+ I)2. Так как п2 и (п + 1) — квадраты последовательных натуральных чисел, а число п2 + п + 1 находится между указанными квадратами, то оно само не может быть квадратом натурального числа. 8.19. Разделим монеты на три кучки по 27 монет. Взвесим первую и вторую кучки. Если весы в равновесии, то фальшивая монета в третьей кучке. Если весы не в равновесии, то фальшивая монета в той кучке, которая легче. После этого разбиваем кучку из 27 монет (в которой есть фальшивая монета) на три кучки по 9 монет и вторым взвешиванием определяем более легкую кучку. Третьим взвешиванием определяем наиболее 225
легкую тройку монет. И наконец, четвертым взвешиванием определяем фальшивую искомую монету. 8.20. Разделим монеты на три кучки: 27, 27 и 25. Если фальшивая монета окажется среди 27, то решение аналогично выше изложенному. Если фальшивая монета среди 25 монет, то разделим 25 монет на три кучки по 9, 9 и 7 монет. Взвешиваем кучки по 9 монет. Если весы не в равновесии, то фальшивая монета среди 9 монет. В этом случае поступаем аналогично предыдущей задаче. Если же весы в равновесии, то фальшивая монета среди 7 монет. Тогда делим 7 монет на кучки: 3, 3 и 1 монету. Взвешиваем кучки по 3 монеты. Если весы не в равновесии, то находим фальшивую монету среди 3 монет аналогично предыдущей задаче. Если весы в равновесии, то фальшивая монета — оставшаяся. В этом случае хватило трех взвешиваний. 8.21. Равенство а2+Ь2 = с2 умножим на с: а2с+Ь2с = с3. Так как с > аис> Ь, то а2с > as,b2c > ft3. Тогда а2с + Ъ2с > а3 +Ь3. Но а2с + Ъгс = с3, значит, с3 > а3 + Ь3. 8.22. 3(Ь-а)(а-с)(Ь-с). 8.23. Прямоугольные треугольники с гипотенузами АЕ и СК, FB и DL равны по двум катетам, значит, АЕ = С К и FB = = DL. Так как EF = KD> как стороны квадрата, то АЕ + EF + FB = СК + KD + DL. Значит, оба маршрута будут одинаковы. 8.24. Да, возможный вариант приведен на рис. 121. Рис. 121 226
8.25. Разложим левую часть на множители: а) (х-у)(х + у) = 2011. Учитывая, что число 2011 — простое, а х и у — натуральные числа, получим х—у = 1, х + у = 2011. Откуда х = 1006, у= 1005. В случаях б) и в) решений нет. 8.26. х = jj . Необходимо проделать одинаковые операции с левой и правой частью: вычесть 1; сократить на 8; перевернуть дроби; вычесть 1; и т. д. 8.27. Воспользуемся неравенством треугольника: каждая сторона треугольника меньше суммы двух других, но больше их разности. Если длина диагонали равна 15, то из оставшихся чисел (длин сторон) можно было бы составить две пары, дающие в сумме больше, чем 15. Однако это не так. Поэтому 15 быть не может. Аналогично не подходит и 10. Если длина диагонали равна 2, то из оставшихся чисел можно было бы составить 2 пары, разность которых меньше, чем 2. Однако это не так. Аналогично не подходит и 4. Тогда остается, что диагональю будет отрезок длиной 5,5, который подходит. 8.28. Так как -х > 0, -у > 0, то х = -(у/^)2, у = -(\/-y)2- Тогда 8.29. При рубке сосен количество деревьев других пород не изменится. Вначале эти деревья составляли 1% леса, а после рубки составят 2% леса. Но если доля прочих деревьев возрастет в 2 раза, то это означает, что лес уменьшится в 2 раза. Поэтому бригада вырубит половину леса. 8.30. Точки пересечения отрезков СЕ и CF с диагональю BD являются точками пересечения медиан треугольников ABC и ADC. Поскольку медианы треугольника делятся в точке пересечения в отношении 2:1, считая от вершины, а треугольники ABC и ADC равны, то расстояния между точками пересечения медиан равно расстоянию от них до вершин В и D. 8.31. Обозначим число abed, тогда получим: abed > а000 = = а-10-10-10> abed. 227
8.32. Пусть х и k — первоначальная цена хлеба и кваса. Тогда х + k = 1,2(0,Ъх + k), откуда х = 0,5fe. Так как вначале денежки хватало на х + k, то ее хватало на l,5fe. После второго повышения цен квас будет стоить l,44fe. Поэтому денежки на квас хватит. 8.33. Найдем разность двух соседних чисел вида 1001...17, она будет равна 9010...0. Числа такого вида делятся на 53. Также делится на 53 и число 1007. А значит, все числа вида 1001... 17 делятся на 53. 8.34. Пусть 1аЪс = 7000 + 100а + 106 + с — исходное число, тогда новое число будет abcl = 1000a -I- 1006 + 10с + 7. Так как разность этих чисел равна 864, то получим: 7000 + 100а + 106 + с = 1000а + 1006 + Юс + 7 + 800 + 60 + 4. Так как число единиц слева и справа совпадает, то с = 1. Число тысяч, сотен и десятков тоже должно совпадать. Имеем: 7000 + 100а + 106 = 1000а + 100(6 + 8) + 10 • 8, откуда 6 = 8 и, соответственно, а = 6. Ответ: 7681. 8.35. Изобразим модель часов (рис. 122); имеем: /LCOB = = ZAOB + ZAOC; ZAOB = 180°-^^ = 150°; ZAOC = = 17,5°; поэтому /.СОВ = 167,5°. Рис. 122 228
9 класс \х + 2\ = х + 2, если* (Jjc-2| = 2-ху если х Ответ: -2 < х ^ 2. -2, 2. 9.1. 0. 9.3. Иван родился в 1980 году. Ему будет 45 лет в 2025 году. 9.4. Умножим на 2 обе части уравнения и сгруппируем. В итоге получим: (х2 + 2ху + у2) + (х2-4х + 4) + (i/2 + 4j/ + 4) = 0, (х + у)2 + (х-2)2 + (у + 2)2 = 0, х = 2; у = -2. Ответ:(2;-2). 9.5. В точке пересечения диагоналей четырехугольника, вершинами которого являются домики друзей ослика Иа. 9.6. На 25%. 9.7. Нет. 9.8. См. рис. 123. Рис. 123 9.9. Возвести обе части равенства а + Ь + с = 5в квадрат. Ответ: 15. 9.10. Предположим, что солгал Алексей. Тогда получается, что он был первым или последним. Значит, солгали еще Владимир или Григорий. А это противоречит тому, что солгал всего один из ребят. Пусть солгал Борис. Тогда он был последним. Но Григорий также утверждал, что он был последним. Значит, данного случая также не может быть. Пусть солгал Владимир. Тогда он был не первым. В этом случае все получается и первым тогда будет Борис. Последний случай, когда 229
солгал Григорий, быть не может, так как тогда последним никто из ребят не был. Ответ: правду сказали Алексей, Борис, Григорий. Первым был Борис. 9.11. Из того, что Вика стоит впереди Сони, но после Аллы, порядок девочек следующий: Алла, Вика, Соня. Так как Денис не находится рядом ни с Аллой, ни с Викой, то Алла стоит первой, Вика — второй, а Денис может стоять лишь крайним справа. Но так как Алла и Боря не стоят рядом, а Борис не находится рядом с Денисом, то место Бориса — после Вики. Тогда порядок будет такой: Алла, Вика, Борис, Соня, Денис. 9.12. Так как (2 \ \/l6 л/5), то Да, верно. 9.13. а) Обозначим а = \/бб, Ь = у[Ч + \/30, тогда а2 = 66, Ь2 = 37 + 2^2Т0. Сравним а2-37 = 29 и Ь2-37 = = 2\/210. Возведем в квадрат снова а2-37 и Ь2-37. Тогда получим 292 = 841, а (2\/21б)2 = 840. Так как 841 > 840, то и \/бб > л/l + л/30, б) Обозначим а = у/в+у/14, Ъ = 2\/3+>/10. Рассуждая аналогично, в итоге получим: (а2-22)2-409 = = 4\/102 и (Ь2-22)2-409 = 71. Так как второе выражение больше первого, то у/Ъ + у/14 < 2>/3 + \/10. 9.14. Не существует, так как тп2—п2 нечетно или кратно 4, а 2010 — нет. 9.15. Ошибка в том, что 916 >_5\2 = ' 2) 2-1 = 0,5, ане—0,5. 2(\/2-1)(\/3 230
9.17. Обозначим показания учащихся через «Б», «Г», «не Г», «не В». Одно из 2 утверждений «Г» и «не Г» истинны, поэтому утверждения «Б» и «не В» ложны, т. е. журнал похитил не Борис, журнал похитил Виктор. 9.18. a + b = ab = j-&a = O, ft ^ О или ft = 1 или Ъ = -1. 1)О + Ь = О-Ь = т:, решения нет, т. к. Ь f 0; о 2)a+l=al = Y> решения нет; 3) а-1 = -а = -^г, решение а = ^; ft = -1. 4 • \/2 > 4, значит, 2^2 + v^l8 > 4. 9.20. См. рис. 124. 9.21. Пусть двузначное число имеет вид 10j/ -I- x. Так как 10г/ + л; = 2ху — четное число, то ил: — цифра четная. Разделив обе части уравнения Юу + х = 2jcj/ на 2j/, получим 5 -I- -£- = л:, откуда х > 5. Учитывая, что jc — четная цифра, получим х = б или jc = 8. Если х = 8, то5 + — = 8иг/ = §, что невозможно. ^j/ о Если же jc = б, то у = 3. Тогда искомое число будет 36. 9.22. Выделим полный квадрат: х4 + 4 = х4 + 4л:2 + 4—4л:2 = = (х2 + 2)2-(2х)2 = (jc2 + 2-2х) • (л:2 + 2 + 2л:). 9.23.(jc2-jc-1)2-jc3 = 5 ^ (*2-л:-1)2-4-(х3 + 1) = 0 & & (л:2-л:-1-2)(л:2-л:-1 + 2)-(л: + 1)(л:2-л: + 1) = 0 & & (х2-х + 1)(*2-*-3-л:-1) = 0. Корнями данного уравнения будут х\ = 1-у/Ь> Х2 = 1 231
9.24. Допустим, что все ученики получили жетонов разное количество. Тогда 10 учеников получили не менее 0 + 1 + 2 + + 3 + 4 + 5 + 6 + 7 + 8 + 9 = 45 (жетонов), что противоречит условию. Значит, найдутся ученики, получившие одинаковое число жетонов. 9.25. аЪаЪаЪ = 100 000а + 10 000b + 1000а + 100Ь + 10а + Ь = = 101010а + 10101b = 10 101(10а + Ъ). Так как 10 101 делится на 21, то и аЪаЪаЪ делится на 21. 9.26. Пусть АС = 49 см, АВ = 99 см, /ВАС = 59° (см. рис. 125). Опустим из вершины В высоту BD на сторону АС. Тогда AD = АВ cos /.ВАС = 99 см • cos 59°. Так как косинус убывает на отрезке [0; тг], то cos 59° > cos 60° = ^. Таким обра- зом, AD > 49,5 > АС и, поэтому основание высоты будет на продолжении стороны АС, значит, А АВС — тупоугольный. 9.27. Проведем 2 разреза, центрально симметричные уже сделанным (см. рис. 126). Куски 1,2,6, 9 достались Малышу, а симметричные им 7,8,4 и 3 — Карлсону, которому отошла еще и середина 5. Поэтому Карлсону досталось не менее половины торта. А ( 4 \7 ^ ^ 2 5 ^ттттттттттттттттттттттш 8 ^ ^ з\ 6 ) / У Рис. 126 9.28. Преобразуем правую часть: \а2 + ЗаЬ + 2Ь2 = \а2 + а2 + 2аЬ + 2 • \ • аЪ + Ъ2 + Ъ2 4 4 I + 2 • | • ab + (а2 + 2аЬ + Ъ2) = 232
Поскольку слагаемые — неотрицательные числа, то и их сумма является числом неотрицательным, поэтому 9.29. Диагональ делит исходный прямоугольник и два внутренних прямоугольника на равные треугольники (рис. 127). Отнимая от равных треугольников равные, получим фигуры равной площади. N. а а N. ^N Ъ Ъ N. Рис. 127 Рис. 128 9.30. Обозначим угол А за а, и пусть АО = а, АВ = Ь (см. рис. 128). Тогда найдем стороны треугольника DMN, применяя теорему косинусов к треугольникам AMD, BMN, CDN: MD2 = а2 + b2-2abcos(a + 60°) = MN2 = DN2. Так как квадраты сторон треугольника DMN равны, то и сами стороны будут равны, т.е. треугольник будет равносторонним. 9.31. Чтобы доказать требуемое неравенство, достаточно доказать, что а2 > -За2-15+13а. Рассмотрим функции Д(а) = = а3 и /2(#) = —За2 -I- 13а—15. Так как дискриминант трехчлена -За2 -I- 13а-15 отрицателен, и -3 < 0, то соответствующий график (парабола) расположен ниже оси абсцисс. А так как график функции f\(x) = а3 при а ^ 0 расположен выше оси абсцисс, то этим доказано, что А (а) > /2(0)- 9.32. Уравнение имеет вид у(2х + Зу) = 24. Легко видеть, что оба сомножителя у и 2х + Зу имеют одинаковую четность, и потому оба четные. Кроме того, очевидно, что если пара (х, у) — решение уравнения, то пара (-х, -у) — также решение. Таким 233
образом, надо рассмотреть случаи, когда оба сомножителя равны четным положительным делителям числа 24, т. е. у равно 2, 4, б или 12. Рассматривая все эти случаи, получаем 8 пар решений: (±3, ±2), (±3, ^4), (±7, ^6), (±17, ^12). 9.33. Так как скорость минутной стрелки больше в 12 раз скорости часовой, то обозначив за х — время, пройденное часовой стрелкой, тогда 12л; — будет время минутной и, учитывая, что первоначально между ними было 20 минут, получим уравнение: 12jc = л:+ 20, из которого х = уу = 1-Д-. Тогда минутная 9 9 догонит часовую через 20+1^ = 21:рг минуты. 9.34. Пусть данное уравнение имеет решение (хо, у о), тогда имеем 2009jcq-2010i/q = 2011, которое равносильно уравнению 2009(jCq-1)-2010i/q = 2. Из данного уравнения следует, что Xq-1 число четное, значит xq = 2k + 1. Но тогда Xq-1 делится на 4. В итоге получили, что в равенстве левая часть делится на 4, а правая только на 2 — противоречие. Значит, уравнение решений в целых числах не имеет. 9.35. Разложим многочлен на множители: Xs + 9хь + 8х2 = = х2(хг + 1)(хг + 8). Так как х четно, то х2 делится на 4, а (xs + 8) — на 8. Пусть х = 3/г, тогда х2 кратно 9. Если х = = Зп ± 1, то Xs + 1 или Xs + 8 также делятся нацело на 9. Так как 4 и 9 взаимно простые числа, то при любом четном х рассматриваемое число делится на 4 • 8 • 9 = 288. 10 класс 10.1. На рис. 129 показано, как нужно сложить трапеции. Snap. = (а + Ь)Л, значит, STp. = ^_ . 10.2. у 1-27^26 + 9v/262 +^26 = \/27-26-27\/26 + 9^262 -3 • З2 • ^26 + 3 • 3 • = ^(3- \/26)3 + ^26 = 3-^26+^26 = 234
10.3. Пусть SAABc = 2SAMnb (рис. 130); Л ABC ~ AMNB; = \f2\ АВ = \/2МВ; AM = AB-MB = \f2MB-MB = Ответ: -l): 1. N a Рис. 129 Рис.130 10.4. Так как из каждой вершины можно провести по 4 диагонали, а всего вершин 7, но по 2 диагонали совпадают, то всего можно провести 14 диагоналей. Ответ: 14 диагоналей. 10.5. Нет, пример приведен на рис. 131 (площадь Л ABC меньше площади AMNK). М_ N К Рис.131 10.6. Заменить х на 1. Получим, что стертое число равно 2. 10.7. Обозначим а = у/2 + \/3 и Ь = л/2-\/3 и учтем, что /Zg a2 /2 i_9 Умножая неравенство на 2, имеем: a + b ^ 2^/g ^ или 235
4- у/2-у/З < 2у/2. Равенство быть не может, так как a f Ъ. Значит: \Ji + ^3 + у/Ъ-Щ < 2\[2. 10.8. Так как оставшийся кусок имеет форму правильного восьмиугольника, а отрезанных кусков — пять, то они могут иметь не больше одной общей стороны со стороной восьмиугольника. Значит, минимум три стороны восьмиугольника принадлежат квадрату. Поэтому форма искомой стенгазеты будет квадрат со стороной, равной расстоянию между противоположными сторонами восьмиугольника. Вырезанные же многоугольники будут: пять треугольников или четыре треугольника и один четырехугольник, причем два треугольника (один треугольник и четырехугольник) будут в сумме составлять один из оставшихся трех треугольников. 10.9. Да, возможные варианты показаны на рис. 132. Рис.132 10.10. а) 3\/5 б) 4тг-10. 10.11. Разность прогрессии может быть равной лишь 1,-1, 2,-2. Так как четной цифрой число заканчиваться не может, то начинаться оно должно с четной цифры (при разности 1 или -1) или нечетной цифры (при разности 2 или -2). Рассматривая возможные варианты: 2345, 4567, 6789, 8765, 6543, 4321, 1357, 3579, 9753, 7531 — находим единственное простое число: 4567. 10.12. Нет, так как д/2 f 1+^ и 10.13. Пусть в первом куске процентное содержание свинца будет а%, а во втором — Ь%. Пусть от первого и второго кусков отрезали по х кг. Тогда получим процентное содержа- 236
ние свинца в первом сплаве: ((6-х)-а , х.ь\ ,л V ЮО lOoJ ' а во втором: (12-*) & 100 Так как они равны, то получим уравнение: (6-х)-а Х.Ь\ /(12-х)-Ь х.а\ "Too— + loo) '6 ~ V—1оо~~ + loo ) ■12- Произведя упрощения, получим: Зх(а-Ь) = 12(а-Ь). Так как (a-b) ^ 0, то х = 4 (кг). Массы отрезанных кусков равны 4 кг. Ответ: 4 кг. 10.14. Обозначим радиус данного круга за г. Тогда радиус искомого круга R = y/T6r = у/г2 + 9r2. Его можно найти как длину гипотенузы прямоугольного треугольника с катетами г иЗг. 10.15. Можно. Начнем с 1. Соседними с ней могут быть только 3 и 7. Соседним с 7 могут быть 4 или 6 или 9. Выберем 4, тогда вторым соседним числом с 4 может быть только 9. Рассуждая далее аналогично, получим такое расположение чисел: 3-1-7-4-9-2-6-5-8-3. 10.16. Обозначив корни уравнения за х\ и х2* получим: где а ^ 0. Выражая х\ и х% из 1-го и 3-го уравнений и подставляя их во второе уравнение системы, получим после упрощения уравнение 9а2-8а-1 = 0. Решая его, найдем а\ = 1; Ответ: при а = 1 или а = — ^. 10.17. Обозначим число килограммов краски, которое надо купить, за N. Тогда N будет иметь вид 3/г, Зл + 1, или Зл -I- 2. Рассмотрим числа вида Зп. Тогда надо брать число банок п, все массой по 3 кг. 237
Рассмотрим числа вида Зп + 1. Минимальное число килограммов краски этого вида будет 10 кг. Тогда 10 = 5-1-5. Если Зп + 1 > 10, то берем 2 банки краски по 5 кг и (п-3) банки краски по 3 кг (З(л-З) + 10 = Зп + 1). Рассмотрим числа вида Зп + 2. Тогда минимальное число такого вида, большее 7, будет 8 = 3-1-5. Если же Зп + 2 > 8, то берем 1 банку по 5 кг и /i-l банку по 3 кг (3(п-1) + 5 = = 3/1-3 + 5 = 3/1 + 2). 10.18. Достичь требуемого невозможно. Следуя со скоростью 60 км/ч, мотоциклист проезжает за минуту 1 км, а за 4 мин — 4 км. При скорости 30 км/ч он проезжает подъем протяженностью 2 км за 4 мин. Поэтому на спуск ему времени не остается. 10.19. Воспользуемся формулой tg ^ = 1 _ZL = ДА"6 = 2_ 12 l + cos£ л , , о 1 ~г - Но л/3 + 1 V^3 + l'\/3 + l 4 + 2v/3 2 + >/з' Значит, tg£ = 10.20. #2-2j/2 = 1 <ь х2 = 2у2 + 1. Так как правая часть уравнения нечетное число, то и левая часть является числом нечетным. Обозначим х = 2п + 1. Тогда 2 *2-1 что означает, что г/ — число четное. Но простым четным числом может быть только 2. Подставляя у = 2 в уравнение, находим решение уравнения: jc = 3, у = 2. Ответ: х = 3, у = 2. 10.21. Графики j/ = sinx и j/ = ал: симметричны относительно начала координат и проходят через начало координат. Поэтому число корней уравнения sin х = ах — нечетно. 238
10.22. Найдем сумму членов арифметической прогрессии: Но последняя цифра п2 может оканчиваться на 0, 1, 4, 9, б, 5. Поэтому данная сумма не может равняться 2007. 10.23. Преобразовывая выражение в левой части неравенства, получим sin6 х + cos6 x = 1 -1 sin2 2x. 4 Так как sin2 2x < 1, то -- sin2 2x > --, а значит, 1-? sin2 2x^0,25. 4 10.24. Так как х4 > 0, 2x2 > 0, то x4 + 2x2 + 8 > 8. Поэтому уравнение jc4 4- 2л:2 -f 8 = а не будет иметь корней при а < 8. 10.25. Петя сможет это сделать следующим образом. Необходимо положить в каждую чашку по 50 монет. Если Петя взял фальшивую монету, то на чашках весов фальшивых монет останется 49. Тогда стрелка укажет разницу в весе монет на чашках, равную нечетному числу граммов. Если же Петя взял настоящую монету, то фальшивых монет на чашках останется 50, и стрелка покажет разницу в четное число граммов. 10.26. Приведем пример, подтверждающий указанную возможность: 30, 30, 30, 30, -130, 30, 30, 30, 30, -130, 30, 30. Числа означают разность между доходом и расходом (сальдо) за каждый месяц (в тысячах рублей). Как видим, сумма любых последовательных чисел отрицательна (и равна -10), а в целом за год сумма всех чисел положительна (равна 40). 10.27. Так как а = 2Ь • sin 10°, то а3 + Ь3-ЗаЬ2 = = 863 sin3 10° + Ь3-6Ь3 sin 10° = Ь3-2Ь3(3 sin 10°-4sm310°) = = b3-2b3sin30° = 0. 10.28. Выпишем подряд несколько первых членов этой последовательности: 1; 2000; 1999; 1; 1998; 1997; 1;.... Заметим, что если в последовательности встречаются два рядом стоящие числа п + 1,л, где п > 0, то дальше она продолжается следующим образом: 1; п-1; п—2\ 1;... . Таким образом, после 239
тройки чисел п + 1, /г, 1 в последовательности будет идти тройка п-1; /1-2; 1. Тогда 670-я тройка будет иметь вид 1; 662; 661, то есть на 2010-м месте будет стоять число 661. 10.29. Так как 13! = 1 • 2 • 3 •... • 13 = 1 • 2 • 3 •... • 11 • (12 • 13) = = 11!.12-13,то 131-11! = 11! (12 13-1) = 11! 155 = 11! 31 5, которое кратно 31. 10.30. Заметим,что47 + 716 = 214 + 716 = 214+2-27-78 + 716- -2 -27-78 = (27 + 78)2-148 = (27 + 78 + 78)-(27 + 78--78),которое является составным. 10.31. \/10 + \/24 = у/2 + 3 + 5 + 2У1ГЗ \/60 = 2у/2Т5 + 2\/3~Г5 10.32. Так как дуги содержат разное число градусов, то отрезки АВ и CD не могут быть боковыми сторонами трапеции, поэтому АВ и CD являются основаниями трапеции. В зависимости от взаимного расположения точек А, В, С, D можно рассмотреть следующие 4 случая (см. рис. 133). 1) 3) 2) 4) Рис.133 240
В первом случае угол между прямыми АС и BD равен 50° + 30° = 40о во втором случае — 2 50°-30° = 10°. В третьем случае — 310°-30° = 140°, а в четвертом, так как основаниями трапеции будут отрезки DB и АС, должны быть дуги AD и ВС равны, что невозможно (обозначив ^DB = х> то wLD = 50°—х, ^ВС = 30° -х> которые не могут быть равны). Таким образом, угол между прямыми может быть 10°, 40° или 140°. 11 класс 11.1. а) 1; б) sinl. 11.2. a) sin(arcsin х) = х> где -1 < х < 1 (график изображен на рис. 134). б) График — на рис. 135. У = sin(arcsinjc) У 7Г -1 Рис А 34 Рис А 35 11.3. Построив графики функций, стоящих в левой и правой части уравнения: у = \х\иу = х—а (см. рис. 136), получим, что при а > 0 графики не пересекаются; при а = 0 пересечением является луч у = х при х ^ 0; при а < 0 — пересекаются в а одной точке х = -. 241
При а < О решений нет. При а = О решением является множество чисел [0; +оо). При а > О уравнение имеет единственное решение. х = Vx + 4; vx + 4 = —x—1. Решая данные уравнения, находим корни первого урав- „ _ l-v/17 нения: хх = — g , ^ g корни второго уравнения: хз = — (посторонний корень) и (посторонний корень), Ответ: xi = = 1— 11.5. Данное уравнение не имеет решений, так как левая часть — число нечетное, а правая — четное. 1-cosx 1-cosx 11 a sm х cos2x 1-sinx (I-sinx)(l-l-sinx) 1-sinx = 0 =*► 1-cosx = 0 или cos x—sin x = 0. Решением данной совокупности уравнений является: xi = 2тгл, х2 = т + я"£> n,keZ. 4 Данные решения являются и решением исходного уравнения. 242
11.7. Правая часть при четном k делится нацело на 4; при нечетном же k дает остаток 1. Левая же часть при п > 1 при делении на 4 дает остаток 3. Поэтому уравнение при п > 1 не может иметь решений. Рассмотрим п = 1, тогда 21 + 7 = fe2, fe = 3. Ответ: л = 1, k = 3. 11.8. Обозначим искомое число за 1000а + 100b -f 10c + d. По условию задачи имеем: 4(1000а + 100b + 10с + d) = 1000d + 100c + 10b + а. Так как левая часть — число четное, то и правая часть — число четное, поэтому а — четная цифра. Тогда а = 2, так как в других случаях получим в левой части пятизначное число. Так как 4 • d оканчивается на 2, то d = 8. В итоге имеем: 4(1000 • 2 + ЮОЪ + Юс + 8) = 1000 • 8 + 100с + 10Ь + 2. Тогда 4(10Ь + с) + 3 = Юс + Ъ или 40Ь + 4с + 3 = Юс + Ъ. После упрощения получим: 13Ь -I-1 = 2с. Решением данного уравнения будут: Ъ = 1, с = 7. Тогда искомое число будет 2178. Ответ: 2178. 11.9. Так как левая часть уравнения является возрастающей функцией на множестве действительных чисел, то любое свое значение она принимает один раз. Значение — функция о примет при х = -1. Ответ: х = -1. 11.10. Обозначим число за х> тогда х = 100 000 -I- а. Если в числе х единицу переставить в конец, то в полученном числе у число а будет показывать число его десятков, т.е.г/ = 10а + 1. По условию: 10а + 1 = 3(100 000 + а), поэтому 7а = 299 999 и а = 42 857. Тогда х = 142 857. 11.11. Возьмем от первой машины один мяч, от второй — два, от третьей — три и т. д., от десятой — десять. Найдем их общую массу. Это взвешивание будет единственным. Если бы все мячи были массой по 10 г, то весы показали бы 10 • (1 + 2 + 3 + 4 + ... + 10) = 550 (г). 243
Если первая машина допускает брак, то общая масса станет меньше на 5 г, если вторая, то на 10 г, и т. д., если десятая, то на 50. Таким образом, по массе 55 мячей можно узнать, какая машина испортилась. 11.12. Нетрудно получить (проверьте!), что 3 < log3 72 < 4 и 3 < Iog736 < 4. Умножая на 2, имеем 6 < log3 74 < 8 и 6 < log7 312 < 8. Так как З7 = 2187, 74 = 2401, то 7 < log3 74 < < 8. Учитывая, что 77 = 823 543, З12 = 531 441, имеем б < log7 312 < 7. В результате имеем: | < log3 7 < 2, 1,5 < log7 27 < |, поэтому log3 7 > log7 27. 11.13. jci = 5-\/24, j/i = 5-\/24, гх = 5-\/24, хх = 5 + л/24, i/i = 5-h >/24, 2i = 5-h y/ti, 11.14. Так как объем, а значит, и масса картофелины пропорциональны кубу ее радиуса, а поверхность — квадрату радиуса (если картофелины считать шарообразными). 11.15. Так как lg ^ < 0, то на последнем шаге получаем не 2 > 3,а2 <3. 11.16. Пусть Ig3 — число рациональное, т. е. Ig3 = ™у причем (т. к. Ig3 > 0) т е N, п е N. Тогда: 10^ = 3 и 10т = 3". Получили, что в левой части последнего равенства стоит четное число, а в правой — нечетное, что невозможно. Значит, lg 3 — число иррациональное. 11.17. Выгоднее купить большой арбуз, так как его объем (с\3 125 т ) = ^тгг раза (т. е. почти в два, а не в полтора) больше, 4/ Ь4 чем объем другого арбуза. 11.18. При п > 3 имеем 10"-1000 = 103(10"-3-1) = 25 • 40 • (10"-3-1); (10д"3-1) делится на 9, значит, (10"-1000) делится на 360. Поэтому все члены последовательности, начиная с четвертого, 244
совпадают с sin 1000° = sin(3 • 360°-80°) < 0. Таким образом, в последовательности 3 положительных члена. 11.19. Разложим левую часть на множители: 2х2 + 5ху-12у2 = 2х2 + 8ху-3ху-12у2 = (х + 4у)(2х-3у). Так как 28 = 1 • 2 • 2 • 7, то решениями уравнения будут являться решения систем уравнений: 2х-3у = 14; Только решение последней системы (8; 5) удовлетворяет условию задачи. 11.20. Ответ: 45°. Указание: вычислите скалярное произведение векторов АС и BD. 11.21. Поступаем следующим образом. Сначала положим на две чашки весов по 13 монет, потом по 11 из еще не бравшихся монет, затем — по 9, затем по 7; 5; 3; 1. Если во всех случаях было равновесие, то фальшивая монета — оставшаяся. Если при каком-то взвешивании одна из чашек перевесила, то фальшивая лежит в другой чаше. Рассмотрим, как ее определить. Если это случилось при первом взвешивании, то разбиваем 13 монет на пары и за б взвешиваний определяем фальшивую монету, взвешивая по одной монете из каждой пары. Если при каком-то взвешивании равновесие нарушается, то более легкая монета и будет фальшивой, если же ни в одном взвешивании равновесие не нарушится, то оставшаяся без пары монета — фальшивая. Аналогично поступаем и в остальных случаях: когда равновесие нарушилось во второй, третий или шестой раз. Если равновесие нарушилось в седьмой раз, когда на чашах лежало по одной монете, то более легкая монета будет фальшивой. 11.22. Так как рыцарь говорит всегда правду, то он ответил утвердительно на 1 из 3 вопросов. Так как лжец — врет, то он ответил утвердительно 2 раза. Известно, что общее число утвердительных ответов равно 150% от числа жителей острова. Поэтому 50% населения острова будут составлять лжецы. Утвердительно на принадлежность ко второй партии ответили 245
рыцари из второй партии и лжецы, которые не состоят во второй партии. Обозначим за п% рыцарей, состоящих во второй партии. Тогда (50-л)% населения острова будут составлять лжецы не из второй партии. Соответственно, во второй партии лжецов будет (50-(50-л))% = п%. Поэтому рыцарей и лжецов во второй партии будут поровну. 11.23. 360 раз. Сначала найдем число всех возможных перестановок из 7 элементов — Ру = 71 = 5040. Но это не будет ответом на вопрос задачи, так как это число возможных вариантов рассаживания на скамейке, а не вокруг круглого или прямоугольного стола. Так как при пересаживании всех гостей одновременно на один стул влево или вправо относительное положение гостей не изменяется, а пересаживаться гости так могут б раз вправо и б раз влево, то искомое число будет меньше в 14 раз, то есть равно 360. 11.24. Разложим на множители пъ-п = п(п2-1)(п2 + 1). Рассмотрим два случая: а) п — четное число; б) п — нечетное число. а) п = 2fe, тогда п2 ± 1 = 4fe2 ± 1. Если п делится на 5, то все доказано. Пусть п не делится на 5. Тогда п = 5т ± 1 или п = Ът±2. Значитп2 даст 25т2± Ют +1 или 25т2±20т + 4. А поэтому п2-\ или п2 + 1 делится на 5. б) п = 2k + 1, тогда п2 + 1 = 4fe2 + 4fe + 2, п2-1 = 4fe2 + 4А. Если п делится на 5, то все доказано. Пусть п не делится на 5. Тогда п = 5ап±1 или п = 5дп±2. Значит, п2 даст 25т2±10т + 1 или 25т2 ± 20т + 4. А поэтому п2-\ или п2 + 1 делится на 5. 11.25. Обозначим тогда получим sine* = 0, решением которого будет а = тг/г, п Е Z. Учитывая, что \а\ ^ ^, получим при п = 0 а = 0. Тогда Li исходное уравнение будет равносильно уравнению sin ^x = 0, L* решением которого будет х = 2/г, п Е Z. 11.26. Предположим, что такое возможно. а) Пусть ребра в 1 см и в 11 см опираются на концы одной стороны квадрата. Тогда сторона квадрата будет больше 10 см. В этом же случае ребра в 1 с,м и б см будут опираться на концы 246
другой стороны квадрата или концы диагонали квадрата. Но та и другая будут не меньше 7 см. А это невозможно. б) Пусть ребра в 1 см и 11 см опираются на концы диагонали квадрата, тогда диагональ квадрата должна быть больше 10 см, а сторона меньше 7 см, что опять же невозможно (7\/2 = >/98 < 10). Таким образом, основанием пирамиды не является квадрат. 11.27. Преобразуем уравнение к виду: которое равносильно уравнению: х2 + х-4 , о х2 х-4 х2-4 Вынося за скобки общий множитель, получим: ^ - -1±\/Г7 Решением первого уравнения будут х = , а второго 11.28. Так как 4 + х2 > 4, a 8~sin * ^ 4, то уравнение 2 + JC4 будет иметь решения только в случае, когда и левая и правая части уравнения равны 4, а это будет лишь тогда, когда х = 0. 11.29. Применяя формулу для синуса двойного угла 2 раза, преобразуем уравнение к виду: 4 sin 4л: • cos 8л: = ^ sin 12л:, 4 4 которое после применения формулы sin a • cos /3 и упрощения преобразуется к уравнению sin 12л: -I- sin 4л: = 0, решением которого будет х = ^fe, k G Z. о 247
11.30. Функция у(х) определена на (-оо;-1) и (-1;+оо) и на каждом из этих промежутков непрерывна. Вычислим tg(y(x)) по формуле тангенса суммы, получим 1. Функция у(х) постоянна на каждом из промежутков области определения. При х = 0 значение у = j, а при х = -\/3 значение у = — ^?. Таким образом, функция г/(лг) имеет вид: '-^ при* е (-оо;-1), прил: G (-1;-1-оо).
Литература 1. Бабинская И,Л. Задачи математических олимпиад. — М.: Наука, 1975. 2. Балаян Э.Н. 1001 олимпиадная и занимательная задачи по математике. — Ростов-на-Дону: Феникс, 2007. 3. Вакульчик П. А. Нестандартные и олимпиадные задачи по математике. — Мн.: УниверсалПресс, 2004. 4. Введенская Т. В., Лященко Е.И., Радченко В. П. Математика, 5 класс: Учимся решать задачи / Ред. Т. Н. Муравьева, О. А. Богомолова. — СПб.: Дидактика, 1995. 5. Внеклассная работа: математические олимпиады по лигам. 5-9 кл. / Автор-составитель: А. Н. Павлов. — М.: Изд-во НЦ ЭНАС, 2007. 6. Дориченко С. А, Ященко И. В. Московская математическая олимпиада: Сборник подготовительных задач. — М., 1994. 7. Епишева О. Б., Крупич В. И. Учить школьников учиться математике: Формирование приемов учебной деятельности: Книга для учителя. — М.: Просвещение, 1990. 8. Канель-Белов А. Я., Ковальдин А. К., Васильев Н.Б. Подготовительные задачи к VII Московской математической олимпиаде 1994 года для 8-11 классов. — М.: Trade Publishers, 1994. 9. Канин Е.С. Учебные математические задачи: Учебное пособие. — Киров, 2003. 10. Козлова Е.Г. Сказки и подсказки: Задачи для математического кружка. — М.: Мирос, 1994. 11. Лоповок Л.М. Тысяча проблемных задач по математике: Книга для учащихся. — М.: Просвещение, 1995. 12. Математическая олимпиада школьников «Юные дарования» (1996-1997). Решения и указания по проверке, оценке и разбору задач олимпиад для 5-8 классов. — Чебоксары: Клио, 1998. 13. Математическая олимпиада школьников «Юные дарования». Решения и указания по проверке, оценке и разбору задач районного, республиканского и зонального туров для 9-11 классов 1996-1997 учебного года. — Чебоксары: Клио, 1998. 14. Материалы для проведения школьных олимпиад по математике / Авторы-составители: Н. И. Мерлина, А. В. Мерлин. — Чебоксары: Клио, 1997. 15. Медников Л.Э., Мерзляков АС. Математические олимпиады. — Ижевск: Свиток, 1997. 16. Мерлин А. В., Мерлина Н. И. Нестандартные задачи по математике в школьном курсе. — Чебоксары: Клио, 1998. 249
17. Московские математические олимпиады 60 лет спустя / Под ред. Ю. С. Ильяшенко и В. М. Тихомирова. — М.: Бюро Квантум, 1995. 18. Московский Интеллектуальный марафон. Сборник заданий. 9-11 класс. / Л. Б. Огурэ. — М.: Интеллект-Центр, 2002. 19. Нагибин Ф.Ф., Канин Е.С. Математическая шкатулка: Пособие для учащихся 4-8 классов средних школ. — 6-е изд. — М.: Просвещение, 1988. 20. О проведении школьного и районного туров олимпиады по математике для учащихся школ г. Москвы. — М.: Изд-во МГИУУ, 1984. 21. Петраков И.С. Математические олимпиады школьников: Пособие для учителей. — М.: Просвещение, 1982. 22. Подходова Н.С. Геометрия. 5 класс: Учебное пособие / Ред. Т. Н. Муравьева. — СПб.: Дидактика, 1995. 23. Попов Г.Н. Сборник исторических задач по элементарной математике. — М.: КомКнига, 2006. 24. Программно-методические материалы: Математика. 5- 11 классы: Сборник нормативных документов / Сост. Г. М. Кузнецова. — М.: Дрофа, 1998. 25. Рассудовская М.М., Сырнев Н.И. Организация и методика проведения математических олимпиад учащихся средней школы: Учебное пособие. — М.: Изд-во МОПИ, 1984. 26. Русанов В.Н. Сборник задач математических олимпиад младших школьников: Пособие для учителей и родителей. — Рос- стали-на-Каме: Оса, 1995. 27. Сборник задач для факультативных и внеклассных занятий по математике: Книга для учителя / Сост. В. Н. Березин, Л. Ю. Березина, И. Л. Никольская. — М.: Просвещение, 1985. 28. CueauiuHCKuu И.Х. Задачи по математике для внеклассных занятий (9-10 классы). — М.: Просвещение, 1968. 29. Ткачева М.В. Домашняя математика: Книга для учащихся 7 классов средних школ. — М.: Просвещение, 1993. 30. Шапиро А. Д. Зачем нужно решать задачи?: Книга для учащихся. — М.: Просвещение, 1996.
Содержание Предисловие к восьмому изданию (От автора) 3 Введение 4 Раздел первый. Подготовка математической олимпиады в школе 6 Раздел второй. Проведение математической олимпиады; проверка, оценка заданий; выявление победителей 11 Раздел третий. Основные направления по подготовке учащихся к математическим олимпиадам 18 I. Работа учителя математики на уроке 20 5 класс 35 6 класс 37 II. Внеклассная работа по математике 42 III. Внешкольная работа по математике 46 IV. Заочная работа 47 Раздел четвертый. Примерные тексты школьных олимпиад 49 5 класс 49 6 класс 58 7 класс 66 8 класс 74 9 класс 77 10 класс 81 11 класс 86 Раздел пятый. Особенности проведения математических олимпиад в сельских школах с малой наполняемостью классов 92 Примерные тексты школьных олимпиад для учащихся 5-6 классов 93 Примерный текст школьной олимпиады для учащихся 7-9 классов 95 Раздел шестой. Задачи для подготовки к математической олимпиаде 100 5 класс 100 6 класс 109 7 класс 115 8 класс 119 251
9 класс 122 10 класс 125 11 класс 127 Раздел седьмой. Ответы, указания, решения 131 Примерные тексты школьных олимпиад (4 раздел) 131 Задачи для подготовки к математической олимпиаде (6 раздел) 202 Литература 249
По вопросам оптовых закупок обращаться: тел./факс: (495)785-15-30, e-mail: trade@airis.ru Адрес: Москва, пр. Мира, 104 Наш сайт: www.airis.ru Вы можете приобрести наши книги с 1100 до 1730, кроме субботы, воскресенья, в киоске по адресу: пр. Мира, д. 104, 4 этаж, тел. (495) 785-15-30 Адрес редакции: 129626, Москва, а/я 66 Издательство «АЙРИС-пресс» приглашает к сотрудничеству авторов образовательной и развивающей литературы. По всем вопросам обращаться по тел.: (495) 785-15-33, e-mail: editor@airis.ru Методическое пособие Фарков Александр Викторович МАТЕМАТИЧЕСКИЕ ОЛИМПИАДЫ В ШКОЛЕ 5-11 классы Ведущий редактор В. В. Мелентьева Редактора. Г. Галан Художественный редактор А. М. Драговой Иллюстрации Н. Г. Рысьева, Н. В. Матушевский Оформление О. Е. Баурина, О. А. Сторожевских Технический редактор С. С. Коломеец Компьютерная верстка Е. Г. Иванов Корректор 3. А. Тихонова Подписано в печать 16.12.08. Бумага газетная. Формат 60x90 ]/\в. Гарнитура «Ньютон». Печать офсетная. Печ.л. 16 Усл.-печ. л. 16. Тираж 8000 экз. Заказ № 6106. ООО «Издательство «АЙРИС-пресс» 113184, Москва, ул. Б. Полянка, д. 50, стр. 3. Отпечатано в ОАО «Можайский полиграфический комбинат» 143200, г Можайск, ул. Мира, 93
1111Ш ШКОЛЬНЫЕ ОЛИМПИАДЫ ■ 1111 Внеклассная работа по МАТЕМАТИКЕ Обл., 2 88 с. В пособии рассматриваются вопросы организации и методики проведения внеклассной и внешкольной работы по математике для учащихся 5-11 классов. Представлен материал для факультативов, кружков, олимпиад, различных соревнований, недель математики, школьной математической печати и т. д. Предложены примерные разработки для указанных форм внеклассной и внешкольной работы. Книга адресована учителям математики, в первую очередь начинающим. Пособие будет полезно студентам педвузов, а также преподавателям вузов, работникам Центров дополнительного образования, Домов детского творчества для организации внешкольной работы по математике, <ц/ издательство
ШКОЛЬНЫЕ ОЛИМПИАДЫ щтттттт ими и ■iiiiiiiiiim тттттттштш шшшш А. В. Фарков УЧИМСЯ РЕШАТЬ ОЛИМПИАДНЫЕ ЗАДАЧИ ГЕОМЕТРИЯ У Обь, 128 с. В пособии рассмотрены основные методы и приемы решения олимпиадных задач по геометрии. В нем представлены задания, которые встречались на олимпиадах разного уровня. Учителя геометрии найдут в книге дополнительные материалы для проведения уроков. Пособие предназначено для учащихся 5-11 классов, желающих самостоятельно подготовиться к олимпиадам по геометрии, и учителей геометрии. a <l/ издательство л Айрис Ач
ШКОЛЬНЫЕ ОЛИМПИАДЫ ш ш II А. В. Фарков МАТЕМАТИЧЕСКИЕ КРУЖКИ В ШКОЛЕ Пособие посвящено методике проведения занятий школьного математического кружка. В нем разобраны вопросы организации и планирования этой формы работы с детьми. В книге предложены примерные разработки 17 кружковых занятий с учащимися 5-8 классов, приведены тексты более 200 задач, большая часть которых решена. В качестве приложения даны варианты городских олимпиад в 5-8 классах. Книга адресована учителям математики, руководителям межшкольных кружков и факультативов. Она также может быть полезна студентам педвузов и учащимся 5-8 классов. А с/ издательство А Аирис!^
■ Школьным учителям математики и руководителям математических кружков ■ Подготовка олимпиады, составление и отбор задач ■ Проведение олимпиады, проверка и оценка заданий, выявление победителей ■ Задачи для подготовки и образцы олимпиадных заданий для 5-11 классов ■ Ответы и решения